Nothing Special   »   [go: up one dir, main page]

Pedia Pracs + Exit Exam Merged

Download as pdf or txt
Download as pdf or txt
You are on page 1of 316

| DISCLAIMER: The department did not post the ANSWER KEY.

NICU 7. Which blood vessels transport blood rich in


1. What are the major causes of neonatal early- oxygen and nutrients to the fetus?
onset sepsis? a. Ductus arteriosus
a. Entero-viruses & HSV b. Umbilical artery
b. Grp B-beta hemolytic streptococci, c. Ductus venosus
Escherichia coli & Listeria monocytogenes d. Umbilical vein
Klebsiella pneumonia & Enterobacter
a0

Candida albicans, coagulase-negative 8. A newborn on the 36% HOL developed poor


staphylococci; Staphylococcus aureus feeding, lethargy, cyanosis and temperature
instability. Respiratory symptoms include
Which of the following is NOT a condition grunting and tachypnea. Which among the
associated with infants of diabetic mothers? following is the initial management for the
a. hyperglycemia patient?
b. Hypocalcemia a. Obtain CBC, Blood culture, CRP,
c. Congenital anomalies Urinalysis, CXR and start Ceftriaxone
d. RDS b. Obtain CBC, Blood culture, CRP,
Urinalysis, Urine Culture, CXR, Lumbar
One must observe the following to help a tap and start Ampicillin and
newly born baby successfully breastfeed Gentamicin
a. Remove the baby from the Mom after the c. Obtain CBC, Blood Culture, CRP,
first breastfeed Urinalysis, CXR and start Ampicillin and
b. Let the baby feed when he is ready Gentamicin
Let the baby feed from one breast only d. Obtain CBC, Blood culture, CRP,
Give the baby to the Mom when he is Urinalysis, urine Culture, CXR, lumbar tap
thermoregulated and start Ceftriaxone
Which of the following is/are mechanism/s by
which early feeding can decrease serum 9. What is the single and most effective initial
bilirubin? step in newborn resuscitation which should be
a. increase liver enzymes for conjugation established with the first minute of life?
b. increase gut motility a. Ventilation
c. All of the choices are correct b. Circulation
d. feeding introduces bacteria to the gut c. Fluid resuscitation
d. Perfusion
What is the most common cause of anemia ina
four-day-old infant? 10. A 37 weeker infant of a diabetic mother may
a. Hemorrhage develop RDS due to:
b. Hemolysis a. Surfactant deficiency
c. Iron deficiency b. Pulmonary insufficiency
d. Folic Acid Deficiency c. Prematurity
d. Neonatal deficiency
Which of the following is NOT an important
predisposing cause of neonatal sepsis?
a. Premature onset of labor
b. Prenatal or perinatal maternal infection
c. Prolonged rupture of membranes
d. Eclampsia
11. Which of the following steps in Essential 17. Which of the following is NOT a recommended
Intrapartum Newborn Care (EINC) has been newborn practice?
proven to be an intervention for provision of a. BCG and Hepatitis B Vaccination after the
warmth and contribute to immunoprotection first full breastfeed
of the newborn? Dry Cord Care

oF
a. Immediate and thorough drying Immediate breastfeeding after bathing
b. Initiation of breastfeeding d. Use of steroids for preterm labor
c. Early skin to skin contact
d. Properly timed Cord clamping
18. An infant is delivered to a mother whose
12. Which of the following disorders are included membranes were ruptured for 24 hrs. He is
in the Newborn Screening Program? well at birth and is breastfeeding well,
a. Conditions with obvious signs at birth however the next day, you are concerned
b. Conditions with no readily available because infant is hypotonic and his exam
treatment reveals a heart rate of 200 beats per minute,
c. Conditions that are life threatening or respiratory rate of 80 per minute and weak
cause severe mental and physical central pulses. Which of the following orders
disability detected and treated early would be appropriate?
d. Conditions that are rare and difficult to a. CBC with differentials, CRP, and blood
diagnose culture
b. Type and cross match, then give 10
13. Which is a consistent manifestation of mL/kg of packed RBCs
intestinal obstruction in the newborn? Give oxygen and observe

ao
a. Polyhydramnios Apply an ice pack to the face to treat
b. Constipation supraventricular tachycardia
c. Abdominal distention
d. Vomiting 19. What is the APGAR score of a newly born baby
with irregular breaths, acrocyanosis, HR
14. When are newborn interventions such as 100/min, some movement of extremities and
weighing, physical examination, eye grimace?
prophylaxis and intramuscular injections best a. 4
done? b. 6
a. After the full breastfeed c 7
b. As soonas possible after birth d 5
c. While the baby is breastfeeding
d. Before the baby attaches to the breast 20. Following a planned home birth, a term male
infant is brought to the emergency department
15. Infants of diabetic mothers are at increased at two hours of life because of respiratory
risk for becoming hypoglycemic due to the distress, The right hand O02 saturation reading
following: is 70%. When she is undressed for the exam,
a. Decreased glycogen stores she begins to cry and her saturation quickly
b. Increased levels of insulin increases to 96%. When she stops crying, her
c. Decreased placental transfer of glucose saturation decreases again to 70% range. This
d. Increased utilization of glucose pattern of saturation change is typical for
which of the following conditions?
16. Which of the following is a recommended a. Cyanotic congenital heart disease
newborn practice? b. Choanal atresia
a. Early bathing c. Diaphragmatic hernia
b. Immediate and thorough drying d. Persistent Pulmonary hypertension
c. Early vitamin K administration and eye
prophylaxis before feeding
d. Immediate cord clamping
21. A newborn developed vesicopustular papules WARD
in face, trunk and extremities 48 hours after Acyclovir is recommended in which of the
birth. If a gram stain of the lesions was done to following conditions:
confirm the diagnosis of erythema toxicum, a. A/7 year-old child with allergic rhinitis
which of the following would be seen? b. A11/2 -year old child infant exposed to
a. Gram negative bacilli a brother with varicella 7 days ago
b. Eosinophils c. 14-year old child with asthma on topical
c. Gram positive bacilli steroids
d. Neutrophils d. An otherwise normal 3-year old child
Which among the following vaccines can
prevent diarrhea and its complications?
22. The following describes the pathophysiology a. Measles vaccine
of Respiratory Distress Syndrome? b. Inactivated Polio vaccine
a. Microatelectasis c. Acellular Pertussis vaccine
b. Ball-valve effect d. Hepatitis A
c. Pneumothorax A child diagnosed with asthma should be given
d. Granular membranes line the bronchioles a medical recommendation in school that
he/she should not join sports activity
a. True
23 . Which of the following is/are true about the b. False
Expanded Newborn Screening?
a. It is done beyond 24 hours of life Heart failure and development of pulmonary
b. Performed ona voluntary basis artery hypertension are observed in
c. All choices are correct infants /children with:
d. Aconfirmatory test is also done among a. Lesions with pressure overload
the 1st degree relatives b. Lesions with obstruction to pulmonary
blood flow
24. A 32 week old boy was born to a 25 year old c. Lesions with left to right shunting
primigravid. The pregnancy was d. All choices are correct
uncomplicated but on the day of unexpected
vaginal delivery, the amniotic fluid was clear The most common etiologic agent for urinary
and normal in volume. The infant cried tract infection is
immediately, but shortly after, the baby a. Klebsiella spp
developed mild grunting and flaring of the alae b. S.aureus
nasi. The RR was 60/min and cardiac exam c. Proteus spp
was normal. BW-1725 grams. What is the most d. E.coli
probable diagnosis?
a. Neonatal pneumonia A child with right pleural effusion has a
b. Meconium Aspiration Syndrome decreased breath sound on the right chest,
c. Transient Tachypnea of the Newborn with right dullness on percussion and
d. Respiratory Distress Syndrome increased tactile fremitus
a. True
25. The probable cause of hematemesis in a b. False
previously well three day old infant is:
a. Intraventricular hemorrhage
b. Bleeding stress disorder Rescue medications for asthma, such as
c. Hemolytic disease of the newborn bronchodilators, may be taken on a daily basis
d. Vitamin K deficiency bleeding to control frequent symptoms
a. True
b. False
8. A 10-year old boy came in to ER due to 13. A patient with severe classic hemophilia will
difficulty of breathing. He has a 1-month benefit from the following treatment
history of low-grade fever and productive modalities, EXCEPT:
cough. He lives with his father who has a. Factor 9 concentrate
pulmonary TB. On PE, the patient is b. Factor 8 concentrate
tachycardic, BP of 80/60, with distended neck c. Cryoprecipitate
veins, muffled heart sounds. Chest xray d. Fresh frozen plasma
showed cardiomegaly with water bottle
configuration. ECG showed the total amplitude 14. The efficacy of ORS in treating dehydration in
ofR +S is <Smm in Limb leads and <8mm in acute watery diarrhea is due to:
the Chest Leads. He will likely need what a. Glucose-linked chloride reabsorption
procedure? b. Glucose-linked potassium reabsorption
a. Endomyocardial biosy c. Glucose-linked sodium reabsorption
b. Emergency pericardiocentesis d. Glucose-linked bicarbonate reabsorption
c. Cardiac catheterization
d. Aggressive treatment with diuretics 15. This is often the first sign of systemic lupus
erythematosus:
This is true of Bronchiolitis a. Joint pains
It has acute onset dyspnea b. Hematuria
2

b. Clinical triad of fast breathing, retractions, c. Malar rash


and wheezing d. Anemia
It is common in school-aged children
ig

d. The most common etiologic agent is 16. Important information to ask in determining
Influenza virus risk factors in seizures with fever is/are
a. Neurodevelopmental milestones
10. A 3-year old boy was seen at the clinic fora b. History of the same illness in the family
check-up. He was 14kg and was 95 cm tall. He c. Age of parents
had good appetite without frequent cough and d. Age of the patient
colds. However, his grandmother was newly
diagnosed to have PTB with a positive sputum 17. A patient with measles may be brought out
AFB smear. The child’s tuberculin test showed from isolation room how many days from the
10mm induration with a normal chest x-ray. appearance of rashes
What is your clinical impression? a.
fw

a. Clinically-diagnosed PTB b.
b. Bacteriologically-diagnosed PTB c.
WO

c. Latent TB d
d. TB exposure
18. The cardinal feature of nephrotic syndrome is:
11. Which is the most common infectious cause of a. Hypertension
upper airway obstruction b. Edema
a. Epiglottitis c. Hypercholesterolemia
b. Subglottic abscess d. Oliguria
c. Laryngomalacia
d. Laryngotracheobronchitis 19. A 14-year old girl is being evaluated for fatigue
and left knee pain. Over the past several
12. The most common cause of urinary tract weeks, she has been complaining of easy
infection in the pediatric age group is: fatigability. Her left knee was tender, red and
a. Double collecting tubules swollen over the past several days. A similar
b. Vesicoureteral reflux event occurred with the right knee 2 weeks
c. Posterior urethral vakve ago that resolved on its own. On PE, she hasa
d. Ureteropelvic junction obstruction 3/6 holosystolic murmur at the apex. Her left
knee is erythematous and tender to palpation.
CBC showed leukocytosis with neutrophilic
predominance and ASO was 800 todd units. Of 23. This spectrum of Iron deficiency is
the following, in addition to cardiology characterized by limitation of erythropoiesis,
referral, which is the MOST appropriate next modest fall in the hemoglobin and little or no
step in management for this girl? change in the red cell morphology
a. Penicillin and glucocorticoids orally a. Irondeficiency anemia
b. Aspirin orally and intravenous b. Iron store depletion
immunoglobulin c. Iron deficient erythropoiesis
c. Penicillin orally and intravenous
immunoglobulin 24, Of the following, which structure in the fetus
d. Aspirin and penicillin orally has the least saturated blood?
a. Inferior vena cava
20. A child was seen for multiple, bilateral b. Superior vena cava
enlarged, but painless cervical nodes c. Ductus venosus
measuring around 2-2.5 x 2.5 cm. CXR showed d. Ascending aortic arch
interstitial pneumonitis with hilar
lymphandenopathies. Tuberculin test showed 25. A 1 year old child, with cough and fever, mild
10mm induration. What is the recommended dehydration, RR: 65 cpm, symmetrical chest
treatment regimen for this case? expansion, no retractions, crackles on both
a. 2HRZ+10HR lung fields, but with equal breath sounds was
b. 2HRZE+4HR referred to you. Your most likely impression
c. 2HRZ+4HR would be:
d. 3HRZE+10HR a. PCAPC
b. PCAPB
21. A 14-month old male had sudden onset of c. PCAPD
jerky movements of the left upper and lower d. PCAPA
extremities. When the mother carried the
child, he was noted to be hot. He was OPD
immediately rushed to the emergency room What parameter in the urinalysis will be
where there was recurrence of the said indicative of UTI?
episode lasting for 15 minutes. After the event, a. Pus cells
he was noted to be drowsy and hold Babinski b. Nitrites
on the left. What diagnostic test would you c. Esterase
result for? d. Bacteria
a. Fasting blood sugar Parameter/s in CBC that will indicate bacterial
b. Serum electrolytes infection include/s
c. Liver function a. Leukocytosis
d. Lumbar puncture b. Neutrophilia
c. Stab cells
22. An 8-year old female was noted to have d. All choices are correct
episodes of bizzare movements of both upper
and lower extremities. There was frequent What is your diagnosis for 4 year old Ana with
twisting of the trunk when walking. While history of fever, cough, colds, coryza and
examination there was note of repetitive conjunctivitis followed by maculopapular rash
episodes of scooping of the hands with starting on face, spreading on forehead,
protrusion of the tongue. All of the symptoms behind the ear, neck to the extremities, leaving
are not seen when patient is asleep. Your fine desquamation?
diagnosis is: a. Measles
a. Chorea b. Erythema infectiosium
b. Tourette syndrome c. Rubella
c. Tics d. Rosela
d. Stereotypies
An 8-month-old infant receiving vaccines from An adolescent taking medications for a
the health center had completed the following: prolonged period may be particularly
BCG-1 dose; DPT /HepB/HiB- 3 doses OPV-3 concerned about
doses. Which of the following vaccines would a. Who will administer the medication
you recommend at this visit? b. Frequency of intake
a. IPV vaccine c. Over-all cost of treatment
b. JE vaccine d. Effect of medications of body appearance
c. Rotavirus over time
d. TdaP vaccine
10. Among the psychosocial developmental goals
A 10-month old infant was brought to the OPD of an adolescent, this is the most affected in an
with acute diarrhea. He had sunken eyeballs adolescent dealing with a chronic illness
and seemed eager to drink. The following will a. Establishing intimacy
be the objective/s of your treatment b. Establishing autonomy
a. Reduce the duration and severity of the c. Establishing one’s identity
diarrhea d. Being comfortable with one’s sexuality
b. Prevent nutritional damage 11. Haley is a 5-year old who likes to draw and
Prevent dehydration color. What shapes should she'll be able to
ao

Treat dehydration copy easily?


a. Circle, square, cross
b. circle, square, diamond
Among patients with acute c. Circle, rectangle, triangle
glomerulonephritis, microscopic hematuria is d. Circle, square, triangle
expected to resolve in
a. 7-10 days 12. Among neonates, this is the recommended
b. 2-3 months urine collection method for urine culture and
c. 6-12 months sensitivity
d. 2-3 weeks a. midstream clean catch
b. catheterized urine
Term, well, newborns discharged on the 48th c. sterile bag collection
hour of life may be followed up after: d. suprapubic tap
a. 4 weeks
b. 3 weeks 13. A 3-year old boy was seen at the OPD for
c. 1 week routine check-up. You would expect him to:
d. 2 weeks a. normally be able to draw a diamond
b. normally weigh about 14 kg
A 2 month old infant was brought to the OPD c. have received 3 doses of DPT and polio
because mother wanted a second opinion on vaccine
her baby who had cyanotic spells and d. all choices are correct
interrupted feeding. She showed the chest
Xray film taken from a government hospital 14. In the management of acute gastroenteritis,
and revealed a boot-shaped heart with Racecadotril may be used to:
decreased pulmonary vascular markings. This a. All choices are correct
is a case of: b. Reduce the hypermotility of the intestines
a. PS c. Reduce the hypersecretion of water and
b. TOF electrolytes into the instestinal lumen
c. MVP d. To treat the bacteria or virus causing the
d. COA diarrhea
15. Juan, 9 y/o complains of inability to sleep due 20. A 13 y/o boy was brought to the OPD because
to anal pruritus. Adhesive cellophane test of rhinorrhea and paroxysmal sneezing lasting
revealed a 1 cm white worm. Juan is suffering for 3 days in the past 3 weeks. No other signs
from: and symptoms. Mother has bronchial asthma.
a. Trichuriasis The family has pet dog at home. The most
b. Ascariasis likely diagnosis is:
c. Enterobiasis a. Sinusitis
d. Toxocariasis b. Allergic rhinitis
c. Idiopathic rhinitis
d. Common colds
16. True regarding Zinc except:
a. Zinc has a crucial role in neuronal 21. This is true regarding acidic stools:
communication and memory a. Itis expected in exclusively breastfed
b. Studies have shown that is beneficial in babies
the management of diarrhea b. Itcan be normal in an asymptomatic child
c. Itis vital for a healthy immune system It can indicate lactose intolerance

a0
and is considered an immunomodulatory All of the choices are correct
d. Zinc is the treatment of choice for
common cold.

17. This radiologic finding is readily seen as


abnormal free air between the liver and
diaphragm
a. Pneumatosis intestinalis
b. Pneumoperitonium
c. lTleus
d. Portal vein gas

18. Poppy demonstrates stranger anxiety by


crying when unfamiliar people pick her up.
She is only happy when her parent carries her.
This is seen by what age?
a. 9 months
b. 6 months
c. 8 months
d. 7 months

19. In the HEADSS risk assessment, the first “S”


stands for
a. Safety
b. Social media
c. Sexuality
d. Suicide
* PEDIATRICS BCA AUG 2019 + EXIT EXAM+PRACS (MAY 2020
FEU-NRMF PGI 2019-2020
PEDIATRICS COURSE AUDIT GREEN DRAGONS
FEU-NRMF PGI ME, PEDIA
atch 2019-202Q EXIT EXAM FEU-NRMF PGI Speat!
PEDIATRICS , AUGUST 2020

Disclaimer: The 100% exact exit exam we had last August 2019 was never* returned because there were identification questions at the end. Also, they
said that the exam topics would be “ANYTHING UNDER PEDIA.” Sadly, the exam questions only revolved on the ones discussed during the course
audit: Growth & Development, Hematology, & Acquired Heart Diseases. What the fruit, right? So This samplex is just basically a mock samplex. It
will comprise the following:
Part I- Pediatrics Green Dragons/Sans Peur Exit Exam May/June 2020
Part Il- Pediatrics Green Dragons/Sans Peur Practical Exam May/June 2020
Part Ill- Growth & Development: The identification questions we had
Part IV- Hematology Samplex (Samplex ( The exam was ACTUALLY SAMPLEX from several Hema exam -_- )
Part V- Acquired Heart Diseases Samplex ( The exam was ACTUALLY SAMPLEX from several Acquired heart Disease exam -_- )
Part VI- GROWTH & Development Charts -ALMOST NEVER NOT ASKED IN THE BOARDS -> FROM BRS PEDIA 2018 & NELSON 2020
P.S. in our exam which was basically samplex from Hema & acquired pedia, some can be seen in the past PGI exams, so check those out too in case they decide to
have an exam which is “ANYTHING UNDER PEDIA.”
Lg ~F SUPER DUPER P.S. This was rationalized by PGls. No correct answers were given, except Hema & Acquired Cardio. If ever you will be utilizing this- Good luck! SS
NOTICE ME FIRST SENPAI!

‘1. Based on EPI, a fully immunized AGE IN MONTHS


child should have received how many ace
doses of Hep B vaccines? HepB
(DTwP - Hib
At c.3 aera
-H B*) id

B.2 D.4 {sombinetions|


combinations

IPV/OPV*

Pedia Platinum: Admin of a 4th dose is > Ld

permitted when a combo vaccine Eee


containing Hepa B is used after birth aries]
dose ( as in the EPI)
-> Minimum age for final dose is 24 wks oa = a
old ( if3 dose series is used) DISCLAIMER: a
jor children and adolescents based on updated literature review, experience
-> if 3rd dose give nea rlier give Ath d ose ane comes current at the time of publication. The PPS, PIDSP an ‘PEV acknowledge that individual Secacesiances may warrant a decision differing from the

at | ea st 4 we ek: S from 3r d dose Soosnoy ol vaccines ont Menosnaebeations relaiina to Wastoemuatstnaton neniinee to develop one a vaccineSets Bonated. som sete ent

Vaccines
in the Phi E ).
The following vaccines are in the 2018 NIP:
* BCG, monovalent Hep B, Pentavalent vaccine (DTwP-Hib-HepB), bivalent OPV, IPV, PCV*, MMR, MR, Td, HPV", JE*
-> Childhood Immunization 2019-----> Recommended Vaccines
These are vaccines not included in the NIP which are recommended by the Philippines Pediatric Society (PPS), Pediatric Infectious Disease Society of the Philippines
(PIDSP) and the Philippine Foundation for Vaccination (PFV)

2. If a newborn is discharged <48 hours after delivery, a definitive appointment must be made for the infant to be
examined within ___ hours of discharge.
A. 24 C. 72
B. 48 D. 96
From: https://pediatrics.aappublications.org/content/125/2/405

A medical home for continuing medical care for the infant has been
identified and a plan for timely communication of pertinent clinical
information to the medical home is in place. For newborns discharged
less than 48 hours after delivery, an appointment should be made for
the infant to be examined by a licensed health care professional,
preferably within 48 hours of discharge based on risk factors but no
later than 72 hours in most cases. '011/15.28,29 |f this cannot be

(GO}FEU-N RMF PGI 2020 #WE-WILL-ALL-PASS! GREEN DRAGONS 5PEAT!


« PEDIATRICS BCA AUG 2019 + EXIT EXAM+PRACS (MAY 2020):

3. A 10-year old patient came in because of diarrhea. The organism causing diarrhea by invading the lamina propia
and setting up an inflammatory process in the intestine is:
A. Salmonella non-typhi
B. V. cholera
C. Clostridium difficile
D. Campylobacter jejuni
Table 306-10 | Mechanisms of Diarrhea
PRIMARY
MECHANISM DEFECT STOOL EXAMINATION EXAMPLES COMMENT

Secretory Decreased absorption, Watery, normal osmolality Cholera, toxigenic Escherichia Persists during fasting; bile
increased secretion, with ion gap < coli; carcinoid, VIP, salt malabsorption can also
electrolyte transport 100 mOsm/kg neuroblastoma, congenital increase intestinal water
chloride diarrhea, Clostridium secretion; no stool
abe cryptosporidiosis leukocytes

Osmotic Maldigestion, transport Watery, acidic, and Lactase deficiency, glucose- Stops with fasting; increased
defects ingestion of reducing substances; galactose malabsorption, breath hydrogen with
unabsorbable substances — increased osmolality with lactulose, laxative abuse carbohydrate malabsorption;
ion gap > 100 mOsm/kg no stool leukocytes
Increased motility Decreased transit time Loose to normal- Irritable bowel syndrome, Infection can also contribute
appearing stool, thyrotoxicosis, postvagotomy to increased motility
stimulated by gastrocolic dumping syndrome
reflex
Decreased motility Defect in neuromuscular Loose to normal- Pseudoobstruction, blind loop Possible bacterial overgrowth
unit(s) stasis (bacterial appearing stool
overgrowth)
Decreased surface Decreased functional Watery Short bowel syndrome, celiac Might require elemental diet
area (osmotic, capacity disease, rotavirus enteritis plus parenteral alimentation
motility)
Mucosal invasion Inflammation, decreased Blood and increased Salmonella, Shigella infection; Dysentery evident in blood,
colonic reabsorption, WECs in stool amebiasis; Yersinia, mucus, and WBCs
increased motility Campylobacter infection

Growth indicators
. . os . . Z-score | Length/height- | Weight-for- | Weight-for- 7
__B_4.A 3-year old came in for a well child visit. His BMI-for-age is 2 for-age age length/height_| BM!-for-age
in the Z-score graph that means he is: — Soe nce ese nee
A. obese C. wasted Above 2 Seances Overweight Overweight
Possible risk Possible risk
B. overweight D. severely wasted Above 1 of overweight | of overweight
(See note 3) _| (See note 3)
0 (median)

5. The step-by-step performance in the essential intrapartum and Below ~1


newborn care is: Below -—2 iSownoks 4) Underweight | Wasted Wasted

. . fac C Severe! Severel)


1. early skin-to-skin contact 1 eee eeow=s | antes” underweight | Severely Severely
. (See note 4) (See note 5) | Wasted weeted
2. non-separation of newborn and mother for
i
early breastfeeding . cane
pele s
Measurements $ in the shaded ange.
boxes Sa are in the normal range.

3. immediate and thorough drying of the newborn, ane


. . . ur until the
4. properly-timed cord clamping and cutting soe Cte
matey
Ceca
DT TRL te
A 1 2 3 4 Cc 2 3 4 1 for Early Breastfeeding
-4,4,9, -4,9,,
B. 3, 4,1, 2 D.4,3,2,1

Give VitaminA Supplementation and Treatment 6. The recommended Vitamin A supplementation in infants 12 months is:
VITAMIN A SUPPLEMENTATION: A. 50, 000 IU C. 150,000 IU
= Give first dose any time after6 months of age to ALL CHILDREN B. 100,000 IU D. 200,000 IU
= Thereafter vitamin A every six months to ALL CHILDREN
‘VITAMIN A TREATMENT:
= Give an extra dose of Vitamin A (same dose as for supplementation) for treatmentif the child has MEASLES or PERSISTENT DIARRHEA. If the child has had a dose of vitaminA within the past month
or is on RUTF for treatment of severe acute malnutrition, DO NOT GIVE VITAMIN A.
= Always record the dose of Vitamin A given on the child's card.
AGE VITAMIN A DOSE

6 up to 12 months 100000 1U

One year and older 200.000 1U

(GO}FEU-N RMF PGI 2020 #WE-WILL-ALL-PASS! GREEN DRAGONS 5PEAT!


y= PEDIATRICS BCA AUG 2019 + EXIT EXAM+PRACS (MAY 2020

7. Routine blood pressure monitoring is recommended starting at age:


A. 1 year
B. 2 years D.5 years
Source: Begin routine blood pressure (BP) measurement at 3 years of age. Correct cuff size depends on
arm size. From Preventive Pediatric healthcare Handbook 2016
SMR Breast
8. A 15 year old girl on physical 1 Preadolescent
examination was found to have a 2 Breast and papilla elevated as small mound, diameter of
developing breast with a secondary areola increased
mound of the areola and nipple. By 3 Breast and areola enlarged, no contour separation
Tanner staging she should be in stage? -
Al Bing C3 8 4 Areola and papilla form secondary mound
B. 3 D4 5 Mature, nipple projects, areola part of general breast
, contour

GIVE ZINC (age 2 months up to 5 years) 9. Zinc supplementation in a 2 year old child
« TELL THE MOTHER HOW MUCH ZINC TO GIVE (20 mg tab): . . an
- with diarrhea is given at a dose of:
2 months up to 6 months | 1/2 tablet daily for 14 days
6 months or more 1 tablet daily for 14 days A. 10 mg x 7 days
« SHOW THE MOTHER HOW TO GIVE ZINC SUPPLEMENTS C. 10 mg x 14 days
* Infants - dissolve tablet in a small amount of expressed breast milk, ORS or clean water in a B. 20 mg x 7 days
cup.
* Older children - tablets can be chewed or dissolved in a small amount of water. D. 20 mg x 14 days

10. A 2 y/o boy was brought to the clinic because of sudden ee Le ea teh Le ee
onset of generalized petechiae. He had varicella Age at onset 2-6 yr 20-50 yr
immunization two weeks ago. On PE, he was playful, afebrile sex predilection None Famaleovennalaait
with generalized petechiae and no organomegalies. Your
segs . vo. Platelet count < 20,000/mcL 30,000- 80,000/mcL
initial diagnosis is:
Duration 2-6 weeks Months to years

B. Chronic ITP Spontaneous remission 90% of patients Uncommon

C. Acute Lymphocytic Leukemia Seasonal pattern Higher incidence in None


D. Acute Myelogenous Leukemia winter and spring

11. The classic presentation of Crohn disease in majority of children and adolescents is:
A. bloody diarrhea B. constipation

In Nelsons, it says Children an present with growth failure as the only manifestation of Crohn Disease. Dec height
velocity occurs in about 88% of prepubertal kids & it preedes GI symptoms.
From: kidshealth.org
The most common symptoms of Crohn's disease are belly pain and diarrhea. Other symptoms include:
e = blood in the toilet, on toilet paper, or in the stool (poop) , mouth sores, weight loss, low energy
® nausea or vomiting, fever
e = skin tags, sores, or drainage around the anus
INCREASED INTRACRANIAL PRESSURE
12. These are significant information in a patient with clinical
(IICP)
(Symptoms Of lICP Are Opposite Of Shock)
history in traumatic brain injury EXCEPT:

~
B. Loss of consciousness
ICP *
tB/e
» Shock
tere
«
Cc. Vomiting J Pulse t Pulse
D. Headache 1 Respirations
(Cushings Triad
t Reeplratione
The other choices are more important with regard to assessing traumatic brain injury. . " )

(GO}FEU-N RMF PGI 2020 #WE-WILL-Al


PEDIATRICS BCA AUG 2019 + EXIT EXAM+PRACS (MAY 2020):

C_13. How much Sodium is present in an reduced ORS solution according to the WHO?
A. 20 meq Cc. 75 meq
B. 90 meq D. 111meq

Formulation:

Reduced osmolarity ORS grams/litre Reduced osmolarity ORS mmol/litre


Sodium chloride 2.6 Sodium 75
Glucose, anhydrous 13.5 Chloride 65
Potassium chloride 1.5 Glucose, anhydrous 75
Trisodium citrate, dihydrate 2.9 Potassium 20

Citrate 10
Total Osmolarity 245

Composition of standard and reduced osmolarity ORS solutions

Standard ORS solution Reduced Osmolarity ORS solutions


(mEq or mmol/l) (mEq or mmol/l) (mEq or mmol/l} (mEq or mmol/l)
(21) (6, 14, 22-27) (13, 15-18, 28-29)
Glucose 111 111 75-90 FS

Sodium 90 50 60-70 fo

Chloride 80 40 60-70 65

Potassium 20 20 20 20
Citrate 10 30 10 10

Osmolarity 311 251 210-260 245

Why “reduced osmolarity”?

Studies have shown that the efficacy of ORS for treatment of children with acute diarrhoea is improved by reducing its sodium
concentration to 75 mEq/I, its glucose concentration to 75 mmol/l, and its total osmolarity to 245 mOsm/I. This compares to the
original solution which contained 90 mEq/I of sodium with a total osmolarity of 311 mOsm/I. There has been a concern that the
original solution, which is slightly “hyperosmolar” when compared with plasma, may risk hypernatraemia (high plasma sodium
concentration) or an increase in stool output, especially in infants and young children.

The study results clearly describe the advantages of this new reduced osmolarity ORS solution in treating children with acute
diarrhea:
1. Itreduces stool output or stool volume by about 25% when compared to the original WHO-UNICEF ORS solution,
2. It reduces vomiting by almost 30%,
3. It reduces the need for unscheduled IV therapy by more than 30%.
This last advantage is particularly important because this means less hospitalisation, and therefore less risk of hospital acquired
infections, less disruption of breastfeeding, decreased use of needles (which remains a strong advantage especially in high HIV
prevalence contexts), less cost, and in areas where IV therapy is not readily available less risk of dying of diarrhoea.

__B__14. The presence of hypoactive bowel sounds is/are associated with:


A. Hyponatremia C. Hypocalcemia
B. Hypokalemia D. All of the above

From Pedia Platinum:


Hypokalemia: Cardiac and Skeletal muscles-> vulnerable. But presents with muscle weakness, cramps, constipation,
ileus, urinary retention ( constipation-> hypoactive bowel sounds)
HYPERkalemia: can also present with muscle weakness and ascending paralysis, but cardiac toxicity precedes these.

(GO}FEU-N RMF PGI 2020 #WE-WILL-ALL-PASS! GREEN DRAGONS 5PEAT!


Hypocalcemia->often asymptomatic. Most Common Presentation: tetany and seizures
HYPERcalcemia-> is the one presenting with hypoactive bowel sounds { abdominal pain, constipation)
Hyponatremia: mostly CNS manifestations -> clouding of consciousness, confusion, stupor, or coma , but can
cause abdominal cramping ( hyperactive bowelsounds)

15. LL. 1 year old has 30% second degree burns. The pathophysiology of burn includes:
Depressed vasoactive mediators
GOP>

Increased anabolic hormones such as insulin like growth factor


Hypermetabolic response
. Myocardial depression in less than 10% TBSA burn
Correct: C???
The hypermetabolic response to injury is characterized by increased blood pressure and heart rate, peripheral insulin
resistance, and increased protein and lipid catabolism, which lead to increased resting energy expenditure, increased
body temperature, total body protein loss, muscle wasting, and stimulated synthesis

Not a? Large burns release vasoactive mediators that result in systemic capillary leakage. Cardiac output is decreased by
circulating factors that depress myocardial function, which may lead to shock.

Pathophysiology
1. Cell damage and death causes vasoactive mediator release:
a. Histamine,thromboxane,cytokine
2. Increasing capillary permeability causes edema,third spacing and dehydration
3. Possible obstruction to circulation(compartment syndrome) and/or airway
Deep Partial Thickness Burns: Pathogenesis and clinical findings
Author and Illustrator: 2 s — 5
Amanda Eslinger Sunlight, x-ray, Flash fire or Hot solid Hot liquid via Strong acid, Contact with
Reviewers: nuclear emission/ direct contact objects immersion, alkali or exposed electrical
Alexander Arnold explosion with flame spill or splash irritant gas wiring/appliances
Duncan Nickerson* + 4 t
* MD at time of publication
Radiation Fire Contact Scald Chemical Electrical
Note: _
Burns are not always
uniform in depth. Burn Direct transfer of heat energy
Injuries take time to Transfer of heat energy
Derere: SOS eves Specific to sunlight radiation, UV Coagulation necrosis is & direct injury to
superficial burns may rays damage keratinocytes induced cellular membranes
evolve deeper over time \ J

ff gx So. Deep Partial Thickness Burn


g 2 Injury to the epidermal layer and both the papillary and a portion of the reticular layer of the dermis
=S
| vascular 4 SS
Edema permeability A majority of Sub- Some healthy dermal
Epidermis ¥ Vasodilation in somato- cutaneous appendages
Fluid leak results fascia sensory tissue surrounded by islands
a1 avedene underlying structures containing of undamaged
between dermal subcutaneous are Pacinian epithelial cells
es
Papillary dermis and epidermal tissue completely corpuscles
i les! 1 injured remains
Reticular dermis intact
M4 Red OR Waxy | 1 2S
Blisters White Analgesia re-epithelialization
+ a
aa inin 2-92-9 weeks
week:
Sub Thin epidermal layer Cutaneous -
cutaneous forming fluid-filled capillary bed is of
Tissue vesicle breaks open destroyed ressure Hypertrophic
< + scarring due to
Moist Wound Non-blanchable injury to\dermis
Legend: Pathophysiology Mechanism Sign/Symptom/Lab Finding Complications | Published December 2, 2013 on www.thecalgaryguide.com (OSS)

__D__16. A3 month-old infant, born at home, came in because of high-grade fever of 3 days. He developed
generalized tonic-clonic seizure few hours PTA. He has not received any vaccination since birth. You are thinking of
bacterial meningitis. You need to request for:
A. CBC C. RBS
B. LP D. All of the above

(GO}FEU-N RMF PGI 2020 #WE-WILL-ALL-PASS! GREEN DRAGONS 5PEAT!


ure
Gra m astaistanl : and cult
on Gra
. May see; orgorg ani5 sms F
e Ifantibiotics $7" 2 cultures because pleocytosis, high CSF protein
Lumbar can be made despite negatv" ict for several days even after antibiotics
logic results because th ‘
on bacteriologic
juncture
(iP) and CSF nue LP is traumatic, it is prudent to rely
5) cf
- s lee level
le maya not t bebe influenced by a traumatic LP
gram stain, culture,and glucose
@s/cs gitis 5 ¢
Ny tients with suspected menin
« Should be performed ee ‘ia in up to 80-90% of cases of metningitis
res pon sib le bact er'
.
« Reveals the
aight PnP, es of meningitis
ial from viral caus
Differentiates bacter
ESR, and

_ B17. Ulcers at the uvulopalatoglossal junction among infants with roseola is known as
A. Koplick spots C. Forsheimer’s spot
D. Café au lait spot

Hello, B. Nagayama spots ito. Koplik ay sa measles (rubeola). Forsheimer ay sa rubella. Cafe au lait ay sa NF sa pagkaka
alala ko. Anyway, ratio from Nelsons:
a L 7 -

cipital nodes. In Asian countries, ulcers at the uvulopalatoglossal


junction (Nagayama spots) are commonly reported in infants with
roseola.
Roseola Infantum: Uvulopalatoglossal spots also referred to as Nagayama spots, are erythematous papules found on the
soft palate and uvula that are seen in two-thirds of patients
neurofibromatosis type 1:
Although these colored spots on the skin can be harmless, having six or more café au lait spots with freckles under the
arm or around the groin could indicate an underlying genetic problem
Café au lait spots, or café au lait macules, are flat, pigmented birthmarks.

__D__18. A 5-year-old child had a low-moderate grade fever of 3 days. On the 5th day of illness, he developed lace-
like erythematous rashes on the arms and slapped-cheeks rashes. Your most likely diagnosis is
A. Roseola C. Erythema infectiosum
B. Measles
Erythema infectiosum (also known as fifth disease) is usually a benign childhood condition characterized by a classic
slapped-cheek appearance (see the image below) and lacy exanthem. It results from infection with human parvovirus
(PV) B19, an erythrovirus. Classic slapped-cheek appearance of fifth disease.

___B__19. Ground itch results from the invasion of the skin with the larvae of
A. Strongyloides stercoralis C. Toxocara canis
D. Enterobius vermicularis
Infection via human hookworms is known as “ground itch.” Hookworms are intestinal nematodes excreted in fecal matter
by infected hosts (usually animals, less commonly humans). The larval form is able to penetrate the epidermis where the infection is usually
confined.

Strongyloidiasis is transmitted through direct penetration of human skin by infective larvae when in contact with soil; walking barefoot
is therefore a major risk factor for acquiring the infection. But it is not called ground itch.

Toxocara canis : Humans are accidental hosts who become infected by ingesting infective eggs in contaminated soil. After ingestion,
the eggs hatch and larvae penetrate the intestinal wall and are carried by the circulation to a wide variety of tissues (liver, heart, lungs,
brain, muscle, eyes).

Pinworms are transmitted by direct transfer of infected eggs by hand from the anus to the mouth of the same or another person. They
can also be transmitted indirectly through bedding, clothing, food or other articles. Spread is facilitated by overcrowding.

(GO}FEU-N RMF PGI 2020 #WE-WILL-ALL-PASS! GREEN DRAGONS 5PEAT!


20. 18-year old male consulted a physician because of vague abdominal discomfort and passage of motile
proglottids per anus. Taeniasis was highly entertained for which the drug of choice is:
A. Niclosamide C. Albendazole
B. Praziquantel D.Pyrimethamine
Could be : A or B but the drug of choice is B
From ANNE Trans: (Doc MICU)
Taeniasis (adult tapeworm)
* Praziquantel: 5-10 mg/kg PO once; Drug of choice
* Niclosamide: 1 gm PO for those 11-34 Ibs; 1.5 gm PO for 35 Ibs
and up

Niclosamide is the drug of choice for treatment of T. saginata and Taenia solium (pork tapeworm) infection; cure rates are approximately 90%. It is not absorbable
and thus is nontoxic. Niclosamide belongs to the family of medicines called anthelmintics. Anthelmintics are medicines used in the treatment of worm
infections. Niclosamide is used to treat broad or fish tapeworm, dwarf tapeworm, and beef tapeworm infections

The drug praziquantel (PZQ) is an essential drug that has had a huge impact in tropical medicine. This is because PZQ is the key therapy for schistosomiasis (Bilharzia),
as well as several other infections caused by parasitic flatworms.

Albendazole, also known as albendazolum, is a medication used for the treatment of a variety of parasitic worm infestations. It is useful for giardiasis, trichuriasis,
filariasis, neurocysticercosis, hydatid disease, pinworm disease, and ascariasis, among others.

Pyrimethamine is an oral antiparasitic drug used for treating Toxoplasma gondii and plasmodia infections. Toxoplasma gondii is a parasite that causes toxoplasmosis,
and plasmodia parasites cause malaria.

21. What are the main symptoms to look, listen and feel when classifying the patient as having pneumonia based on
IMCI?
A. stridor in a calm child severe pneumonia
B. patient presenting with convulsion severe

D. child unable to drink or breastfeed with stridor severe


Ratio: See photo below. source: IMCI
SDJHRI
THEN ASK ABOUT MAIN SYMPTOMS:
Does the child have cough or difficult breathing?
If yes, ask: Look, listen,
le For how long? = feel: © Any general danger Pink: = Give first dose of an appropriate antibiotic
Count the ee sign or SEVERE = Refer URGENTLY to hospital"
breaths in Classify © Stridor in calm child. PNEUMONIA
. COUGH or oR.
one DIFFICULT VERY SEVERE
minute BREATHING DISEASE
Look for
* chest * Chest indrawing or Yellow: = Give oral Amoxicillin for 5 days"
* Fast breathing. PNEUMONIA = If wheezing (or disappeared after
indrawing rapidly acting bronchodilator) give an
° MUST BE inhaled
Look = bronchodilator for 5 days****
andCALM If chest indrawing in HIV exposed/infected
listen for stridor. *™ child, give first dose of amoxicillin and refer.
Look and Soothe the throat and relieve the cough with
listen for "a safe remedy
wheezing. If coughing for more than 14 days or
If wheezing with either recurrent
fast breathing or chest wheeze, refer for possible TB or asthma
Indrawing: assessment
Give a trial of rapid acting Advise mother when to return immediately
inhaled bronchodilator for Follow-up in 3 days
If the child is: up to three times 15-20 * No signs of pneumonia or Green: = If wheezing (or disappeared after rapidly
2 months up to 12 months minutes apart. Count the very severe disease. ‘COUGH OR acting bronchodilator) give an inhaled
breaths and look for chest COLD bronchodilator for
42 Months up to 5 years indrawing again, and then. = Sdays****
classify Soothe the throat and relieve the cough with
Fast breathing is: = asafe remedy
50 breaths per minute or If coughing fer more than 14 days or
more recurrent
= wheezing, refer for possible TB or asthma
40 breaths per minute or = assessment
more Advise mother when to return immediately
“If pulse oximeter 1s available, determine oxygen saturation and refer if < 90%. Follow-up in 5 days if not improving
“* if referral is not possible, manage the child as described in the pneumonia section of the national referral guidelines or as in WHO Pocket Book for hospital care for children.
"™"Oral Amoxicillin for 3 days could be used in patients with fast breathing but no chest indrawing in low HIV settings.
“*** In settings where inhaled bronchodilator is not available, oral salbutamol may be tried but not recommended for treatement of severe acute wheeze.

22. If a 4 year old child has cough with wheezing, classified as having cough, which disappeared after giving rapidly
acting bronchodilator, which of the following is the next step for its treatment?

B. advise to increase oral fluid intake


C. advise to follow up after 3 days 5 days
D. refer for possible TB assessment if coughing for more than 14 days or recurrent wheezing

(GO}FEU-N RMF PGI 2020 #WE-WILL-ALL-PASS! GREEN DRAGONS SpeATi


Ratio: see photo above from IMCI. if wheezing (or disappeared after rapidly acting bronchodilator), give an inhaled bronchodilator
from 5 days. SInce wala to sa choices, dun tayo sa **** which says: In settings where inhaled bronchodilator is not available, oral
salbutamol may be tried but not recommended for treatment of severe acute wheeze.

23. This is the clinical finding in a newborn with breastfeeding jaundice .


A. Fever Cc.
B. poor suck D. dark yellow stools
Physiologic in the newborn. Normally
happens after 24hrs of life.
Farmers
Onset
[resstedng snus [Sreasiklouce
3°-4th day of life
| Late- start to rise on day 4; may reach
20-30mg/dL on day 14 then decrease
slowly, Normal by 4-12 weeks

( the other choices has no face in them) Pathophysiology Decrease milk intake Unknown; probably due to B-
resulting to increase glucuronidase in breastmilk which
- Declines after 2nd week, it disappears
enterohepatic circulation increase enterohepatic circulation;
oppositely starting from the lower extremities Normall Liver Function Test, (-)
Hemolysis
and lastly on the face.
-Healthy baby with jaundice has a good cry,
Management Fluid and caloric If breastfeeding is stopped, rapid
good suck, no fever, and examination supplementation decrease in bilirubin level in 48 hours,
findings are normal. if resumed may rise to 2-4mg/dL but
not to previous level
- The only abnormal result is
HYPERBILIRUBINEMIA, B1 (Indirect bilirubin). Breast Milk Collection
- If POSITIVE for breastfeeding jaundice, v¥ Mother's work / illness
v Hygiene
mothers STOP breastfeeding for 2 days ¥ Breast pumps/ manual expression
TEMPORARILY. Breast Milk Storage
24. Stored, expressed breastmilk at room temperature ¥ Used thawed breast milk w/in 24 hours.
¥ Don't refreeze thawedor thawed breast milk.
lasts
C. 18 hours oom re fs
B. 12 hours D. 24 hours Insulated coolerwith ice packs
or

25. Which of the following statements is TRUE of oral Polio vaccine?


Attenuated vaccines — organism is alive, produced
. Polio vaccine — live attenuated type given orally and the
B. It can be given at birth. inactivated type given IM
C. It is given as a single dose. o Culture for live attenuated vaccines: virus grown in a
monkey kidney cell
D. It is prepared by heat-inactivation. Discovered by Albert Sabin; while the
a inactivated type
-B is wrong, coz it is given at 6 weeks. C is wrong because it is 3 doses was discovered by Jonas Salk
oO Pathogen is grown under adverse culture condition
and is passed through various host
Polio Vaccine
o Vaccine type that can revert to its wild, unattenuated
Bivalent: type 1 and type 3 (type 2 is already eradicated)
form
Oral Poliovirus Vaccine (OPV)
. Polio vaccine can cause paralytic
+ Live, attenuated
poliomyelitis
« Multiple doses (3 doses)
. Benefits still outweigh the risk, since a very small percentage only
e May revert to actual infection will exhibit this adverse effect
« WOF for vaccine-assoc. poliomyelitis
Inactivated Poliovirus Vaccine (IPV) Inactivated — treated with chemical or heat
e — Given intramuscularly (IM) . Cannot revert to a virulent form
« Given at a minimum age of 6 weeks with a minimum a Exception: in cases of subunit vaccines which contains
interval of 4 weeks antigen that needs to be boost via adjuvants, which
e The primary series consists of 3 doses could also contribute to the reactions following
« A booster dose should be given on or after the 4th immunization
birthday and at least 6 months from the previous dose
« No occurrence of vaccine-assoc. poliomyelitis Stability is related to the vaccine’s ability to withstand exposures to
« May be incorporated in other vaccines — hexavalent (DPT, extreme temperatures. (genetic > inactivated > attenuated)
Hib, inactivated polio)

(GO}FEU-N RMF PGI 2020 #WE-WILL-ALL-PASS! GREEN DRAGONS 5PEAT!


% eg,» PEDIATRICS BCA AUG 2019 +
EXIT EXAM+PRACS (MAY 2020

26. History taking in a patient with central nervous system involvement entails:
A. Presence of headache C. Presence of rashes
B. Presence of gait
At first you'll be like rashes? But then since A and B are very correct-> gait problems like in Multiple sclerosis, or in
headache (Increased ICP) so D is correct
Disorder Screening test | Confirmatory
27. The newborn screening for congenital test
adrenal hyperplasia will detect: CAH 17 OHP 717 OHP
A. 21-OH enzyme CH TSH 174 TTSH
B. cortisol Galactosemia Galactose LGALT activity
PKU Phenylalanine LPAH activity
D. DHEA G6PD G6PD activity LG6PD activity
MSUD Leucine Leucine

28. Eruption of permanent teeth usually begins at what age ?


A. 1 year old C. 4 years old
B. 2 years old

RATIO (from Pediatric trans on Growth & Development)


e Eruption usually occurs at 6 mos of age- mandibular central incisors
e Eruption of permanent teeth begins at around 6 years old and completed at around 18 years old.

__B___29. The ideal age for a successful toilet training is between:


A. 24-29 months C. 40-48 months
D. 49-55 months

¢ TOILET TRAINING
v Average age of successful toilet training: (1960’s) 27-28 e At 2-4 years old, the child is developmental ready to
months; (1990's) 35-39 months . . woe . :
¥ Early training (<2 years old) should be discouraged due to begin toilet training but the ideal age for toilet
its association with chronic stool retention and encopresis training is between 35-39 months (around 3 years
(fecal soiling)
¥ Key factor: READINESS OF THE CHILD
¥ Positive reinforcement and regular toilet times ¢ Girls typically acquire bladder control before boys
¥ Give calm and understanding support * Bowel control in general is usually achieved first
¥ NOCTURNAL ENURESIS
— Occurrence of involuntary voiding at night at 5 years old
before bladder control

__B___30. The most common extrapulmonary tuberculosis is:


A. Pott’s disease C. TB meningitis
D. Miliary Tuberculosis
Lymph Node Disease
Tuberculosis of the superficial lymph nodes, often referred to as scrof-
ula, is the most common form of extrapulmonary tuberculosis in
children (Fig. 215-12). Historically, scrofula was usually caused by
Arinkine wannactomrigad eeware mill ladan writh AT hewsic AAnet crarrant

C__ 31. A PICU patient in septic shock needs a vasoconstrictor. These drugs are vasoconstrictors EXCEPT:
A. Dopamine
B. Epinephrine

D. Norepinephrine

(GO}FEU-N RMF PGI 2020 #WE-WILL-ALL-PASS! GREEN DRAGONS out ly E


PEDIATRICS BCA AUG 2019 + EXIT EXAM+PRACS (MAY 2020

Table 70-13 | Cardiovascular Drug Treatment of Shock


DRUG EFFECT(S) DOSING RANGE COMMENT(S)
Dopamine T Cardiac contractility 3-20 pg/kg/min T Risk of arrhythmias at high doses
Significant peripheral vasoconstriction at
>10 pafkg/min
Epinephrine T Heart rate and T cardiac contractility 0.05-3.0 pa/kg/min May + renal perfusion at high doses
Potent vasoconstrictor T Myocardial ©; consumption
Risk of arrhythmia at high doses
Dobutamine T Cardiac contractility 1-10 pafka/min a
Peripheral vasodilator
Norepinephrine Potent vasoconstriction 0.05-1.5 pg/kg/min T Blood pressure secondary to T systemic vascular
resistance
No significant effect on cardiac contractility T Left ventricular afterload
Phenylephrine Potent vasoconstriction 0.5-2.0 pg/kg/min (Can cause sudden hypertension
T ©: consumption

32. Osmotic diarrhea is characterized by the following:


A. stool volume is more than 200 mL/day
B. stool pH is more than 6 D. none of the above

Osmotic diarrhea:
¢ Ingestion of poorly absorbed solute
e Fermented in the colon ANTIMICROBIAL AGENTS USED IN THE TREATMENT OF
. SPECIFIC CAUSES OF DIARRHEA:
¢ Production of SCFA
. ANTIBIOTIC OF
* t osmotic solute load CHOICE _ ALTERNATIVE
° Lesser volume than secretory diarrhea
i CHOLERA Tepacine perenne
z 1e / TMX
TMX-
¢ Stops with fasting —
e Ex, ingestion of lactulose, sorbitol, etc. SHIGELLA Morofioncl Nalidixic acid /
7 Ing , , DYSENTERY ene Ampicillin
. . . Dehyrdoemetine
33. GG has bloody diarrhea. Dysentery is best treated with: AMOEBIASIS Metronidazole HCl
A. cotrimoxazole C. nalidixic acid GIARDIASIS Metronidazole Quinacrine
B. ciprofloxacin D. amoxicillin SALMONELLA No antibiotic of
TYHPHOID choice
*E. coli - antibiotic only within the 1st 24 hrs; no function afterwards

Ask: Look:
34 This/These is/are the danger sign/s in IMCI: e Is the child able todrinkor e Seeif the child is lethargic
. . breastfeed? or unconscious.
Cc. cough e Does the child vomit e Is the child convulsing
everything? now?
B. fever D. all of the above]. tas the chit
had
convulsions?

35. This is TRUE of arm blood pressure compared with calf blood pressure :
A. arm systolic pressure is higher by 5 - 10 mmHg
B.
C. the arm and calf systolic pressures are the same
D. the calf systolic pressure is higher by 20 mmHg

RATIO (Nelson’s): Ordinarily, the pressure recorded in the legs with the cuff technique is approximately 10 mm Hg higher
than that in the arms.

_36. These clinical features are seen in spasmodic croup EXCEPT:


A. High grade fever
B. Barking metallic cough

(GO}FEU-N RMF PGI 2020 #WE-WILL-ALL-PASS! GREEN DRAGONS 5PEAT!


y= PEDIATRICS BCA AUG 2019 + EXIT EXAM+PRACS (MAY 2020

C. Hoarseness
D. Noisy inspiration

RATIO (Nelson’s): Most patients have an upper respiratory tract infection with some combination of rhinorrhea,
pharyngitis, mild cough, and low-grade fever for 1-3 days before the signs and symptoms of upper airway
obstruction become apparent. The child then develops the characteristic “barking” cough, hoarseness, and inspiratory
stridor. The lowgrade fever can persist, although temperatures may occasionally reach 39-40°C (102.2-104°F); some
children are afebrile.

37. Baby Penelope, 5 month old had her well baby check up. On physical examination her anterior fontanel is 1.5 cm x
1.5 cm and closed posterior fontanels. At what age did you expect the anterior fontanels to close?
A. 4-5 months C.6—12months
D. 3-5 months
Two fontanelles usually are present on a newborn's skull:
e@ On the top of the middle head, just forward of center (anterior fontanelle)
e Inthe back of the middle of the head (posterior fontanelle)
e@ The posterior fontanelle usually closes by age 1 or 2 months. It may already be closed at birth.
e@ The anterior fontanelle usually closes sometime between 9 months and 18 months
B 38. In nephrotic syndrome, the main event that leads to the cascade of clinical manifestations is;
A. formation of edema

C. tubular cell malfunction decreasing the amount of protein reabsorbed


D. the secretion of renin and aldosterone.
Of course you will not have edema unless you start losing protein in the urine.
EDEMA is a clinical hallmark, but it won’t happen unless you start losing protein thru urine
International Study of Kidney disease in Children (ISKDC)
Hyperalbuminuria Hypoalbunemis Lb Oncotic
« Criteria > must have all of these four: Pressure

o Massive proteinuria > 40 mg/m?/hr_


o Hypoalbuminemia < 2.5 g/dL
= N: 3.5-4.5 g/dl
o Hypercholesterolemia/ hyperlipidemia
co Edema formation (Clinical Hallmark)
ADH, Renin, Water Retention
Angiotensin, EDEMA!
Aldosterone
Release

Diagnosis
_C___ 39. Atwo-year old male is diagnosed with UTI. — « Consider UTI if the folowing:
The initial renal imaging that must be requested is: e Culture >50K colonies of a single pathogen (via suprapubic
og tap or catheter sample
A. plain film of the abdomen Or if there are 10K colonies and child is symptomatic,
B. IVP bag sample urinalysis result is (+), colony count is > 100K, and

e Urinalysis is suggestive of UTI; urine culture is used for final


D. CT scan diagnosis (gold standard).
KUB ® Urinalysis: take into consideration proper collection
e Suprapubic tap (3rd best)
e Kidney, urinary bladder e below 1 year old: to rule out fistula
» Pre and post void bladder ultrasound « used for super duper contaminated GUT
e First renal imaging study performed fora diagnosed case of UTI * not used for hematuria
* Normal kidney is vascular, same echogenicity with liver * not routine
. « : : . an e Catheterized urine is a good alternative (2nd best)
« Kidney whiter than liver, classified as hyperechoic, and indicates : . 9 f . )
. e Midstream urine collection for cooperative patients (best)
renal parenchymal disease e Wee bag is not used because it is prone to contamination.
e Hydronephrosis is hypoechoic than liver. Negativity - 70% of the time, there is no infection

(GO}FEU-N RMF PGI 2020 #WE-WILL-ALL-PASS! GREEN DRAGONS 5PEAT!


Latent Period (Asymptomatic)
® infection may have subsided, continued, on and off, or
40. The average latent period for post-streptococcal
infectious was already treated (most commen)
glomerulonephritis: ® Variable course:
A. 2-3 days « less than a week to 6 weeks (viral)
B. 4-7 days D. 21 days «4 weeks, or even as long as 2 months (bacterial)
* Average of 10-14 days (Post-strep)
GABS, Staph, Salmonella, Leptospirosis # Representative diseases:
BACTERIAL * Post-steptococcal glomerulonephritis
Group 6 Strep causes Gram(+) neonatal
® throat/skin infection
sepsis. If Gram (-), E. coli.
« Group A Beta Hemolytic Steptococeus (GABS)
VIRAL Mumps, measles, chicken pox ® typical strep serotypes in AGN / nephritogenic
strains include
PROTOZAN Malaria
® pharyngitis: 1, 3, 4, 12, 25, 49
* Post-infectious glomerulonephritis is characterized by a previous
® pyoderma: 2, 49, 55, 57, 60
infection prior to the onset of the nephritic syndrome.
* Immune complex reaction beading to nephritis
* 2-12 years old, usually; mo sexual predominance (M=F) Symptoms Percent «lf all s/sx are present but
* All events under the overview of post-infectious:
glomerulonephritis should be present. ‘Hematuia
(+) allpis. 100 Neem
NEPHRITIC.
® Infection -> Latent Period -> Nephritic syndrome (positive Proteinuria 80 « Gross or microscopic
hematuria and other manifestations of the mephritic syndrome} Edema 90 hematuria
ol el e Hematuria with proteinura
HIN 60-80 should lead you to
41. In a patient suspected of glomerulonephritis, the cardinal
Oliguria 10-50 considering GLOMERULAR
symptom is the presence of: Dyspnea, HF <5 DISEASE.
. sy « Edema starts at the
A. edema C. high blood pressure
Nephrotic proteinuria 4 perioribtal area, subsides,
D. congestive heart Azotemia 25-40 and goes to the lower
failure Early Mortality <] extremities, and may lead
to anasarca.
e Edema, HTN, dyspnea, HF are related to volume expansion due

42. Following a streptococcal infection, ASO titer starts to to loss of renal function.
e Oliguria and azotemia are also secondary to loss of renal
increase after: function.
C. three weeks
B. two weeks D. four weeks

In an infected individual, the Group A Streptococci produced Streptolysin O acts as a protein antigen and
causes the patient's immune system to mount a defensive response with Antistreptolysin O antibodies. Arise
infection and peaks 2-3 weeks later.

43. In which of the following gives you the highest possibility that UTI is present.
A. to numerous to count pus cells in the urine
B. if there is bacteriuria and pus cells on urinalysis yD
C. a positive test for WBC esterase and nitrite test
“In the Philippines the cutoff considered as significant in
diagnosing UTI is presence of WBC >5/hpf. It is only
D. Positive test for bacteria considered significant value to consider UTI, it does not
TEST SENSITIVITY SPECIFICITY always mean that patient already have UTI.”
WBC ESTERASE 83% 78% “Sensitivity increases as the tests are combined together.”
NITRITE 53% 98% “Specificity of 70%: if all of the tests are negative in 10
WBC ESTERASE 93% 72% patients, only 7patients are sure to be negative while the 3
AND patients are not sure to be negative to have UTI.”
NITRITE “4 parameter test used by old hospitals / clinics in
MICROSCOPY Philippines: pH, Specific gravity, Protein and Sugar”
WBC >5 / hpf 73% 81% “Even if the sensitivity of these tests combined: 99.8%, it
Bacteria 81% 83% cannot replace URINE CULTURE (gold standard) in
(+) WBC ESTERASE, 99.8% 70% diagnosing UTI.”
[hk NITRITE, MICROSCOPY

(GO}FEU-N RMF PGI 2020 #WE-WILL-ALL-PASS! GREEN DRAGONS 5PEAT!


$
PEDIATRICS BCA AUG 2019 + EXIT EXAM+PRACS (MAY 2020

44. A grade 3 male student is diagnosed with IgA nephropathy. Hematuria is defined as:
A. >5rbc/hpf
ina spun urine MICROSCOPIC HEMATURIA:
B. >5 rbc/hpf in an unspun urine - >5 RBCs/hpf on more than two occasions
C. >15 rbc/hpf in a spun urine —_— ‘
D - Significant Hematuria
>15 rbc/hpf in an unspun urine . . .
o RBCin the urine is > 5S (Other literature, >3)
Test strips can detect 5-10 intact RBCs/mul or 2-5 RBCs/hpf ' The implication in lower cut-off is that you will
© 2-5RBCs/hpf is a positive test but does not necessarily be screening for more people and getting work-
mean significant hematuria up for patient is that are normal
© POINT: test strips (qualitative) should be correlated ' The implication in higher cut-off (>5) is that the
with microscopic findings (quantitative) lesser people will be screened, possibly missing
patients who might have significant hematuria

What is the definition of hematuria?


Commonly thought to be greater than 5 rbc's per HPF on spun
urine. A dipstick test will detect red blood cells but also will

In quantitative terms, the difference between


invisible and visible hematuria is simply a question of
degree. As few as three to five RBCs/HPF qualify as
microscopic hematuria, but it won't appear to have
Etiologic Agents
45. The most common etiologic cause of urinary tract infection in « predominantly gram (-) [EKE]
children is: * E. coli
- 80%
;
A. Klebsiella pneumonia; ‘
C. Citrobacter sp. ¢ Klebsiell
eases Pheamoniaei
¢ Enterobacter
D. Pseudomonas sp. ¢ Pseudomonas from urine have relatively low virulence
unless
patient is immunocompromised
46. A15 year old/female has a chief complaint of cough for 2 weeks. You +S, aureus and Enterococcus are MC gram (+)
decided to perform Mantoux test to this patient. What type of
hypersensitivity reaction is the Mantoux test? >5mm
A. Anaphylactic Hypersensitivity reaction + HIV positive
B. Cytotoxic Hypersensitivity reaction 215mm
+ Recent contact with an active TB patient
+ Nodular or fibrotic changes on chest X-ray
C. Immune Complex Hypersensitivity reaction + Persons with no known risk factors for TB + Organ transplant

reaction
>10mm
Correct: This question should be a no-brainer at this level
+ Recent arrivals (< 5 yrs) from high-prevalence countries
+ IV drug users
+ Resident/employee of high-risk congregate settings
47. The hallmark of chest x-ray of childhood pulmonary tuberculosis is: « Niycbaclerology lob perdorinel
« Comorbid conditions
A. Hilar lymphadenopathy C. Pneumonic infiltrates ican tases
B. Cavitation D. Calcifications + Infants, children,
& adolescents exposed to high risk categories

Correct: Should be A. This is tricky. | was gonna answer B or D. but in Nelson’s :


The hallmark of primary tuberculosis in the lung is the relatively large size of the regional lymphadenitis compared with
the relatively small size of the initial lung focus. As delayed-type hypersensitivity develops, the hilar lymph nodes
continue to enlarge in some children, especially infants, compressing the regional bronchus and causing obstruction. The
usual sequence is hilar lymphadenopathy, focal hyperinflation, and then atelectasis. The resulting radiographic shadows
have been called collapse-consolidation or segmental tuberculosis.

(GO}FEU-N RMF PGI 2020 #WE-WILL-ALL-PASS! GREEN DRAGONS south EN


-(+) calcification implies lesion at least 6-12 mo.
-In adults, In active pulmonary TB, infiltrates or consolidations and/or cavities are often seen in the upper lungs with or
without mediastinal or hilar lymphadenopathy

48. Rolf, 7 years old is diagnosed with Dengue fever with warning signs. These statements are TRUE regarding the
Recovery Phase in Dengue EXCEPT:
A.B. ItThereoccursis improved
24-48 hoursappetite.
from the time the patient becomes afebrile.
C. It is characterized by diuresis.
D. Platelets start increasing while hematocrit start decreasing with stable vital signs.
Clinical Problems During the Recovery Phase
Criteria for Discharge
* Recognizing when to decrease or stop IVF is key to
¢ Afebrile for 72 hours
preventing fluid overload
Treatment of Fluid Overload * Good appetite
* 2 therapy s/b given immediately o If the patient ask for food they are in
. Stop IVF therapy during the recovery phase will allow recovery phase
fluid in pleural and peritoneal cavities to return to the * Visible clinical improvement
intravascular compartment
* — IVF should be discontinued or reduced to the * Good urine output
minimum rate when the following signs are present ° No bleeding
o Signs of cessation of plasma leakage * Rising platelet count 2100,000
o Stable BP, pulse, and penpheral perfusion * No respiratory distress
o Hematocrit decreases in the presence of a * No evidence of cardiac/CNS involvement, other
good pulse volume
complications
o ©Afebrile for more than 24-48 h (w/o use of
antipyretics)
o Resolving bowel/abdominal symptoms
o = Improving urine output
49. CC, 5 years old, male has primary dengue infection. His serologic test will not have a reactive:
A. NS1 antigen
B. Dengue IgM D. All of the above Laboratory Criteria for Dengue Confirmation
. . A 7 1. Serology
NS1 and IGM - Acute infection agree aD Blot + 24 fold change in IgG titer to one or more dengue
IgG - Past infection "primary. - IgM:lgG 512 virus antigens in paired samples
Oo 5 -
- + Anti-dengue IgM
9 Secondary IgM:IgG <1.2 2. Virologic confirmation
50. FF, 15 years old, female is being suspected + _ Viral isolation from serum
: : . . . . . + Demonstration of the dengue virus antigen by
of having Dengue infection. Chikungunya is one differential IFAT
diagnosis that has a more prominent symptom of: «RT-PCR

B. bleeding D. muscle pain

a |
Common
Middle East respiratory symptoms
syndrome
Made (MERS) and
carver ess Fa
severe acute respiratory e@ Fever
syndrome (SARS) are viral
respiratory illnesses A dry 2cough develops [MIRO
caused by a coronavirus. e after to 7 days Slay at ory ser
cecpco
ray uyae
Severe Y
symptoms PSS Ltr)
® High fever Mild breathing
(100.4°F or higher) e_—_ difficuities at
* Pneumonia ry 9
Ki ,
Kidney failure e@ Gastrointestinal
issues
Transmission be is characterised by internal
Coughs or @————_ Diarrhea Pee aU auton
Br TCE Cc ie) Te oe Ms =e}
infected from
sneezes person General
or touching body aches
contaminated objects.

ae org
ee aes bee _ |_|

Y=
Cd ee toe Wear long = Use mos:
C1 Ree that Reese Lt Gx:
Pe A ce Panay Gx:

Le Keep surroundings i @} Seeadoctor


if high-grade
y Ame ReNea 7] fever continues
for more
7 vostagnant water I aes -NRMEF PGI 2020 #WE-WILL-ALL-PASS! GREEN DRAGONS 5PEAT!
a 4 PEDIATRICS BCA AUG 2019 + EXIT EXAM+PRACS (MAY 2020)862) 3
Etiology: Winine rel

BECAUSE | WANT TO LEARN MORE, Etiology (MCC) RSV


pie
Pneumococcus
oe
M. pneumoniae

| WILL ANSWER THE EXTRA 5 QUESTIONS! SES:


yes thisi was iin the exam Course Indolent, usually Acute, progressive Indolent, prolonged
self-limiting

. . cBC
51. Which of the following pathogens WBC Normal, low or slightly Usually high Variable
. + elevated
causes atypical pneumonia? Diff count Lymphos predominate Neutros predominate Variable*
A Hemophilus influenzae Other labs ESR, CRP, Procalcitonin
. may be elevated
B. Staphylococcus aureus Imaging
-
C.
. . Treatment Supportive Antibiotics (usually Antibiotics (usually
D Klebsiella pneumoniae beta-lactams) Macrolides)

Clinical findings are often less severe than suggested by the patient chest radiograph, explaining why the term “walking
pneumonia” is often used to describe CAP caused by M. pneumoniae. - Nelson’s
RISK CLASSIFICATION FOR MORTALITY
. . . . Variables PCAP A PCAP B PCAP C PCAP D
52. A7 year
.
old child with community
. .
acquired . “
—— = None Present Present Present
pneumonia who presented with mild dehydration, jc Yes Yes No No
tachypnea and without cyanosis can be classified as:| Ab » Follow: Yes Yes No No
A PCAPA C.PCAPC Dehydration | None Mild Moderate Severe
Abilityto feed _ Yes Yes No No
D. PCAP D A je | >11 mos | >11 mos <11 mos <11 mos
Tachypnea i Yes Yes Yes Yes
Chest retractions © None None Present Present
‘Head g None None Present Present
Cc None None Present Present
r None None None Present
Apnea None None None Present
Sensorium Awake Awake Irritable Lethargic
Complications | None None Present Present
Action plan i OPD OPD Ward ICU

53. These otoscopic findings are indicative A diagnosis of AOM according to the 2013 guideline should be
of acute otitis media: made in children who present with:
. . * moderate to severe bulging of the TM or new-onset otorrhea not
A. presence of middle ear effusion caused by otitis externa
B resence of air bubbles in the mild bulging of the TM and recent (<48 hr) onset of ear pain or
. P intense ITM erythema
tympanic membrane
C. erythematous and bulging tympanic membrane
D. dry, perforated tympanic membrane
STAGES OF ACUTE OTITIS MEDIA
We Stage of Hyperemia/Retraction
: : ‘ od This is the onset of disease, which is characterized
Characterized
: '
by' signs ' and symptoms of middle' ear bymucoperiosteum.
a generalized hyperemia. of — the
inflammation with or without presence of effusion of * Symptoms will include mild earache, ear fullness
an ever.

<3 weeks iin children


i
(lecture update based on CPG «
co Se
An
taniaineeianed
erythematous and markedly retracted eardrum

of Philippine Society of OHNS 2016)

On otoscopy, the tympanic membrane appears


erythematous, sometimes bulging or retracted. On
pneumatic otoscopy, the tympanic membrane is
immobile.

(GO}FEU-N RMF PGI 2020 #WE-WILL-ALL-PASS! GREEN DRAGONS south OE


‘ eg,» PEDIATRICS BCA AUG 2019 + EXIT EXAM
+PRACS (MAY 2020

54. A2 year old, male, came in due to unilateral epistaxis with foul smelling mucopurulent discharge. What will be
your primary diagnosis?
A. Acute Sinusitis

Cc. Rhinosinusitis
D. Allergic Rhinitis

Foul smelling discharge = Foreign body

55. GG, 8 years old female, daughter of an allergologist develops acute urticaria. There is mild itch but otherwise, she
is active and has good appetite. She must be given:
A. Epinephrine subcutaneously
B.
C. Montelukast
D Steroids

Antihistamines have also been shown to be beneficial in the treatment of acute and chronic urticaria/angioedema. -
Nelson’s

END OF EXAMINATION
THANK YOU GUYS, STAY SAFE!

ran: eo axcreaLxmmmaroneanan
1.) At the 1st minute of life, a newborn was noted SCORE oO sl 2

to have a Cardiac Rate of 140, with active HR Absent =100 bpm 7100 bpm
. ‘ ‘ RR Absent, Slow, crying Good
movement, grimace to stimulation, irregular :
Some
acrocyanosis1 and good respiration
1 1
ect
MUSCLE
Limp
.
flexion of
-
Active
Active

A. What is the APGAR score? 8 REFLEX No a Cough or


B. What is the normal APGAR score? 7-10 IRRITABILITY | response ae
+ + age 4 ompLetely
C. What is the significance of APGAR score? COLOR Blue, pale | Acrocyanosis pink
. . . ‘ 7-10: NORMAL
Assess for need of immediate resuscitation 4-6 BORDERLINE
<3: RESUSCITATE
15’ MINUTE: ASSESS NEED FOR RESUSCITATION
5™ MINUTE: EFFECTIVENESS OF RESUSCITATION

2.) An irritable 3 y/o Male with history of vomiting and loose stools was seen at the OPD with pertinent physical
examination of: sunken eyeballs, dry lips, hyperactive bowel sounds and was noted to drink eagerly.

A. What is the level of dehydration? Some Dehydration


sunken eyeballs + dry lips + Drinks eagerly -> Moderate dehydration -> If with 2 or more signs in B-> SOME
DEHYDRATION
B. Give management (Drug/s and dosage) including home instruction. (see photos below. Some dehydration-
yellow and plan B)

(GO}FEU-N RMF PGI 2020 #WE-WILL-ALL-PASS! GREEN DRAGONS south OS


Does the child have diarrhoea?

Two of the following signs: Pink: . If child has no other severe


® Lethargic or SEVERE © classification: Give fluid for severe
if yes, ask: Look and feel: © unconscious DEHYDRATION dehydration (Plan C)
© For how long? . Look at the child's ® Sunken eyes OR
Is there blood in the general condition. Is for DEHYDRATION Not able to drink or W child also has another severe
nn © drinking poorly © classification:
stool? © the child: Lethargic or
unconscious?
Skinipinch{gossibecklvery Refer URGENTLY to hospital with
Classify slowy. © mother giving frequent sips of ORS on
© Restless and DIARRHOEA the w:
° irritable? Look for . Advise the mother to continue
. sunken eyes. breastfeeding
Offer the child fluid. Is If child is 2 years or older and there is.
© the child: cholera in your area, give antibiotic for
Not able to drink or cholera
drinking poorly?
°

Two of the following signs: Yellow: Give fluid, zinc supplements, and food for
Drinking eagerly, * Restless, irritable SOME some dehydration (Plan B)
thirsty? © Sunken eyes DEHYDRATION If child also has a severe classification:
Pinch the skin of the * Drinks eagerly, thirsty Refer URGENTLY to hospital with
°
abdomen. Does it go © Skin pinch goes back mother giving frequent sips of ORS on
back: Very slowly slowly. the way
(longer than 2 Advise the mother to continue
seconds)? breastfeeding
Slowly? Advise mother when to return immediately
Follow-up in 5 days if not improving
Not enough signs to classify | Green: = Give fluid, zinc supplements, and food to
as some or severe NO treat diarrhoea at home (Plan A)
dehydration. DEHYDRATION = Advise mother when to return immediately
= Follow-up in 5 days if not improving

Dehydration present. Pink: = Treat dehydration before referral unless the


SEVERE child
and if diarrhoea 14 PERSISTENT = has another severe classification
days or more DIARRHOEA Refer to hospital

No dehydration. Yellow: = Advise the mother on feeding a child who


PERSISTENT has
DIARRHOEA = PERSISTENT DIARRHOEA
Give multivitamins and
= minerals (including zinc) for 14 days
Follow-up in 5 days
Blood in the stool. Yellow: = Give ciprofloxacin for 3 days
DYSENTERY = Follow-up in 3 days
and if blood in stool

PLAN B: TREAT SOME DEHYDRATION WITH OFS


in the clinic, give recommended amount of ORS over 4-hour period

= DETERMINE AMOUNT OF ORS TO GIVE DURING


FIRST 4 HOURS
WENGHT |= 6 kg 6-=<10 kg 10-—12 kg 12-19 kg
AGE* Lip to 4 4 months op to 12 12 months up to 2 2 years
up tho S
months moms years years
in ml 200 - 450 450 - B00 800 - 860 960- 1600
“Use 1S OMS age oni wen wou do mot know ine Beg. The spprowinmate amount of a7
rAaQuned fat Wan aio be CacuiSted Dy TU ie chi S weg: fin Ag) tines ra.
«© Hthe child warts more ORS than shown, gwe more
« Por infants under 6 months who are mot breasted, also give 100
- 200 mi clean water during thes.
Beriod tf you use standard OFS. This is not needed if you use mew bow comolarity ORS.
=» SHOW THE MOTHER HOW TO GIVE ORS SOLUTION.
« Give frequent small saps from @ cup.
« the child vomits, wait 10 minutes. Then continue, but mom showy.
+ (Gonminue breaste=ading whenever the child wants.
= APTER4 HOURS -
* eassess the chiéd and classify tree chiééd for dehydration.
= Select the i (Plan to contin. trestinent.
«+ Begin feeding
the child in clinic.
= IF THE MOTHER MUST LEAVE BEFORE COMPLETING TREATMENT:
« Show her how to prepare ORS solution at home.
« Show her Foow mrsch ORES to gre to fimesh 44howr treatment
at hom
« (Give her enough ORS packets to complete rehydration.
Also give ier? packets.
a5. recommended
in Plan AL
« Exopisen the 4 Rules of Homme Treatment:
PEDIATRICS BCA AUG 2019 + EXIT EXAM+PRACS (MAY 2020

25 428

342
* mech“ile
95
S High Risk Zone
=
= KZ fhe os7
E is a
edt
=
[eeeesl =
2 ope as aah Ss
= a| EE
A)
2 171
55° wr LL =
a es ey Low Risk Zone .

|
5 -

. t -O
° ° 12 24 36 48 60 72 84 96 108 120 132 144
Postnatal Age (hours)

3.) Baby Girl A is on her 58th hour of life. You noted that the jaundice was down to the abdomen. Bilirubin Levels
revealed Total Bilirubin 14.3 mg/dl,B1 12.2 mg/dl and B2 2.1 mg/dl.
Facts to consider: Physiologic jaundice: appears after 24 hours -> so this case is Physiologic jaundice Peaks: Day
2-3, Disappears 5th day of life
Jaundice is level of abdomen: so estimate is 15 mg/dl -> DECREASED BIOTRANSFORMATION (CONJUGATION)

Under normal circumstances, the level of indirect bilirubin in umbilical cord


serum is 1-3 mg/dL and rises at a rate of <5 mg/dL/24 hr; thus, jaundice
becomes visible on the 2nd or 3rd day, usually peaking between the 2nd and 4th
days at 5-6 mg/dL and decreasing to <2 mg/dL between the 5th and 7th days
after birth.
In general, a
search to determine the cause of jaundice should be made if (1) it appears in the
1st 24-36 hr after birth, (2) serum bilirubin is rising at a rate faster than 5
mg/dL/24 hr, (3) serum bilirubin is >12 mg/dL in a full-term infant (especially in
the absence of risk factors) or 10-14 mg/dL in a preterm infant, (4) jaundice
persists after 10-14 days after birth, or (5) direct bilirubin fraction is >2 mg/dL at
any time.

a.Using the Bhutani chart at what zone will the bilirubin levels fall?
High-intermediate Risk zone { because baby is on the 58th hour of life with a total serum bilirubin of 12)

B. Using Kramer’s Chart what will be the approximated bilirubin levels of this patient?

jaundice was down to the abdomen -> 14 mg/dl (|


can’t find this sa Nelson’s )
— r
Table 1. Visual Assessment of Neonatal Jaundice (Kramer's rule)

C. What will be your plan of management? Area of the Body Level


of Serum Bilirubin

-Firstly, phototherapy is indicated in High-


trunk (above umbilicus) 2 85 - 204 5-12
intermediate risk zone. (below umbilicus)
-Explain that it resolves in 1 week in full term infants
-Interview patient to assess for need to do COOMB’s
Test.
-Educate about breastfeeding and how it may ause
jaundice

(GO}FEU-N RMF PGI 2020 #WE-WILL-ALL-PASS! GREEN DRAGONS south OS


y= PEDIATRICS BCA AUG 2019 + EXIT EXAM+PRACS (MAY 2020

4.) Baby Zoe a 7 month old female came in the OPD for well baby check up.

What is her IBW? 7 x 500 + 3000 grams = 6500 grams


7-12 months: Age {in mos) x 500 + BW
* Average weight at birth: 3000 grams Age Growth in HC / month
Birth — 3 months 2.cm
3-6 months 1cm
What iis the expected HC? ? 44.5 cm 612 months OScm
At birth head circumference is 35 cm ( normal babies
Should be monitored routinely during the first 3 years of life 13 years 0.24 cm
¥ 8 y 4-6 years 1icm/year
o Especially in the first 2 year
Measured over the most prominent part of the occiput and just above the supraorbital ridges

Assuming Birth Head Circumference is 35 cm -> She is 7 months:


35 cm ( birth ) + (6 cm -> 1st 3 months has 2 cm for the first month) + (3 cm -> 2nd 3 months has 1 cm per month) + ( 0.5 cm -> for the 7th month)
=44.5cem

Compute for the number of teeth/s expected for this age group?
7-months old: so 2 centralincisors from mandibular area (5-7 months) +2 central incisors on maxillary area { 6-
8months) +2 lateral incisors from mandibular area { 7-10 months) = 6 teeth-> It is expected for her to have 6 teeth

Cinco trans: Chronology of Human Dentition of Primary (Deciduous)


Eruption usually occurs at 6 months mandibular and Secondary (Permanent) Teeth
central Incisors
0 You can wait until 15 months of age CALCIFICATION AGEATERUPTION AGE AT SHEDDING
- Eruption of permanent teeth begins at around 6 y/o Begins at Completeat Maxillary Mandibular Maxillary Mandibular
and completed around 18 y/o PRIMARY TEETH
- Visit to the dentist could be done as early as the first | Gemtrel incisors. sthfetalmo | ia-2¢mo | &8mo_ | 5-7 mo TS yr | GT yr
tooth erupts Lateral imcisors 5th fetal mo 18-24 mo 8-limo | 7-10mo0 6-3 yw 7-8 90
Nelsons: Cuspids (canines) Gthfetalmo | 30-36mo | 16-20mo|1620mo | 11-19 yr | S41,
~ . , . First molars Sihfealmo | 2430mo | i0-l6me|i0-iema |i0i2y | 10129
Initial mineralization begins as early as the Second molars Gthfetalmo | emo 20-30 mo | 20-30mo | i0-i2y | 1i-i3yr
2nd trimester
oo.
(mean age for .
SECONDARY
Central incisors
TEETH 34 m0 §-10 yr 7-8 yr 6-7 yw
central incisors, 14 wk) and continues Lateral incisors Max, 10-12 mol 10-11 yr 8-5 yr 7-2 yr
through 3 yr of age for the
. Bnsy 8 P Y
primar Cuspids (canines)
Mand, 3-4 mo
4-5 mo i-isy | ii-2@y |[S-liy
(deciduous) teeth and 25 yr of age for First premolars (bicuspids) | 18-21 mo T2-igyr | i@ily | idi2y
Second premolars (bicuspidsy 24-20 mo 12-14 yr 12 yes{ 11-13 yr
the secondary (permanent) teeth. ee snp sie oe te
Eruption begins with the central incisors Second molars a0-d6 mo THity|ifisy | i=liy
‘Third molars Max,
and progresses laterally. Mand, 7-5i077
yr 18-25 yr 17-22 yr | 17-22 yr

Nelsons:
Central incisors: 5-7 months ( mandibular), and 6-8 months ( maxillary).
Lateral incisors 7-10 months (mandibular), and 8-11 months { maxillary)

5.) Identify the lesion. Milia


Is it Physiologic or Pathologic? Physiologic

Milia are tiny white spots due to accumulation of sweat in blocked pores. About 50% of infants have milia on the face,
most resolving within the first 4 weeks of life. Milia in newborns may also occur on the hard palate (Bohn's nodules) or
on the gum margins (Epstein's pearls). These also resolve spontaneously.

(GO}FEU-N RMF PGI 2020 #WE-WILL-ALL-PASS! GREEN DRAGONS 5PEAT!


« PEDIATRICS BCA AUG 2019 + EXIT EXAM+PRACS (MAY 2020):

5.) During a physical examination to an adolescent you noted that


. . . . Table 137.4 | Adolescent Psychosocial Assessment:
6.) there is breast bud with elevation of breast and papilla. [| MIS heen
a.) Identify the SMR. Tanner Stage 2
b.) In getting the Personal and Social History of this patient usin Home. Space, privacy, frequent geographic moves, neighborhood
)Ing & . . Y P 8 Education/School. Frequent school changes, repetition ofa grade/
HEADSSS FIRST, what is letter E? Education in each subject, teachers’ reports, vocational goals, after-school
c.) In interviewing the adolescent, what principle/s must you educational clubs (e.g., language, speech, math), learning
?
disabilities ; ‘ Ree
remember: Abuse. Physical, sexual, emotional, verbal abuse; parental discipline
Although the rules for confidentiality are the same for new and continuing patients, the Drugs. Tobacco, electronic cigarettes or vaping devices, alcohol,
change in the physician—patient relationship, allowing more privacy during the visit and more marijuana, inhalants, “club drugs,” “rave" parties, others; drug of
autonomy in the health process, may be threatening for the parent as well as the adolescent. choice, age at initiation, frequency, mode of intake, rituals, alone
For new patients, the initial phases of the interview are more challenging given the need to Br Peo, stro, (net States
Safety. Seat belts, helmets, sports safety measures, hazardous
establish rapport rapidly with the patient in order to meet the goals of the encounter. Issues activities, driving while intoxicated
of confidentiality and privacy should be explicitly stated along with the conditions under Sexuality/Sexual Identity. Reproductive health (use of
which that confidentiality may need to be altered, that is, in life- or safety-threatening contraceptives, presence of sexually transmitted infections,
situations. feelings, pregnancy)
Family and Friends
- Family: Family constellation; genogram; single/married/
Tanner Stages of Development Mnemonics separated/divorced/blended family; family occupations and
shifts; history of addiction in first- and second-degree relatives;
Nobody ElevalCs 2 mouniams “ Not parental attitude toward alcohol and drugs; parental rules;
Agu tthood CAT chronically ill, physically or mentally challenged parent
Friends: Peer cliques and configuration ("preppies," "jocks,"
No gondsiag "nerds," “computer geeks," cheerleaders), gang or cult
fi \ f\| affiliation
NO haw, Image. Height and weight perceptions, body musculature and
physique, appearance (including dress, jewelry, tattoos, body
piercing as fashion trends or other statement)
Breast bud fo2ms mal Small amount of
Recreation. Sleep, exercise, organized or unstructured sports,
downy ham
+ small dondwlaa TSive ll le : | |
+ arcda widens
iqmectohion recreational activities (television, video games, computer games,
om? internet and chat rooms, church or community youth group
Breast > Elevated a e coarse & Rhy activities [e.g., Boy (BSA)/Girl Scouts; Big Brother/Sister groups,
+ eftends payer’ bodes Il is e i \ | + ontonds tatteally campus groups]). How many hours per day, days per week
involved?
tecola + ppl = —— , Aault= hike naire, Spirituality and Connectedness. Use HOPE® or FICA’ acronym;
Seeondary round > ° adherence, rituals, occult practices, community service or
> size , > elevation fer + spares thighs involvement
: SS Ss Threats and Violence. Self-harm or harm to others, running away,
Final ; J ) Thighs rot cruelty to animals, guns, fights, arrests, stealing, fire setting,
AduIT © 6) KK) | spared GID fights in school
a =a NAME OF LOCATION
FONTANELLES
7.) A. Identify Structure A? Anterior fontanelle Anterior * This is found at the junction of
sagittal, coronal and frontal
B. When do you expect Structure B to close?
Fontanelle/Bregma} seures
2-3 months after birth * It is broad kite/diamond shaped
and it measures 3-4 cm long
and 1.5-2 cm wide and
normally closes by 18 month.
= : Posterior * This is situated at the
junction of the lambdoidal
Fontanelle/Lambda and sagittal sutures.
(shape like nr) ¢ It is small triangular in
8.) Aone year old boy with unrecalled vaccinations shape. It closes by 6 weeks
9.) given developed fever. On his 3rd day of illness, he was oCuee
10.) brought to the OPD. Upon oral examination, you saw:
Koplik’s spots

A. Identify the pointed lesion. Kopliks spot


B. This is pathognomonic of what disease? Measles

(GO}FEU-N RMF PGI 2020 #WE-WILL-ALL-PASS! GREEN DRAGONS 5PEAT!


j
PEDIATRICS BCA AUG 2019 + EXIT EXAM+PRACS (MAY 2020):

11.) PP, 5 years old was admitted because of 5 days fever. On the 8th hospital day ,
fever lysed and you noted the appearance of these skin changes in the legs.
A. What do you call the skin lesion? Hermans rash
B. In what disease is this rash seen? Dengue
C. In what phase of the disease it is typically seen? Recovery phase

10. 12 month old, male, came in the clinic for a well baby visit
A. Expected Developmental milestones
Motor: walks with one hand held; rises independently, takes several steps (Knobloch)
Adaptive: Picks up raisin with unassisted pincer movement of forefinger and thumb, releases object to other person on request or
gesture
Language: Says a few words besides “mama,” “dada”
Social: Plays simple ball game, makes postural adjustment to dressing

Childhood Immuniza
B. Based on EPI, what vaccine
should he receive? acein se 8 oe: . monmes
MMR vaccine
C. What other vaccines will Crear He
you recommend? combinations
PCV vaccine booster{ if ewsory
compete First 3 doses of a
PCV 6 months prior) “Measles
- Varicella =H
-HepaA Hep A
HPV
-Influenza ( if not yet given)

Part a: Identify the Patterns of Development: SS


The pediatrician or the parent may recognize concerning patterns of development, such as delay, dissociation, deviancy or deviation, or
regression.
a.) Developmental delay: when development is occurring in its usual sequence but at a slower rate, with milestones achieved later than the normal
range. Delay can occur in a single area of development or across several streams
ALL SKILLS-still achieved ---------------------------- straight line
b.) Developmental dissociation: delay in a single stream with typical development in other streams. A child with autism may have delays in verbal
or social language but normal motor skills.
-Atypical-> Lags in some domain
c.) Deviancy or deviation: development occurring out of sequence, as when a child stands before sitting (as in diplegic cerebral palsy) or has
better expressive vocabulary than receptive understanding of words (language and autism spectrum disorders).
Ex: skip 1 part; not learn language, but can learn alphabets.
d.) Developmental Regression: a loss of skills. It may also be identified earlier or more subtly by a slowing or lack of advancement in skills. Although
uncommon, regression is described in as many as 1 in 4 children with autism and is also seen in rarer neurologic disorders, such as Rett syndrome
and Duchenne muscular dystrophy
e.) Typical Development-> most normal, by order-> follows a line ------------------------------------------------

Developmental surveillance: tracking a child’s achievement of milestones, which represent key readily recognizable skills that usually occur ina
predictable sequence and at predictable age ranges during childhood. The developmental skill areas can be divided into gross motor, fine motor,
verbal speech and language (expressive and receptive), social language, and self-help. Tracking milestones will reveal that most children achieve
the milestones in a typical pattern and within typical age ranges.

(GO}FEU-N RMF PGI 2020 #WE-WILL-ALL-PASS! GREEN DRAGONS 5PEAT!


PEDIATRICS BCA AUG 2019 + EXIT EXAM+PRACS (MAY 2020):

4 Domains: Motor/ Language / Social / Personal

Part b: Identification:
Write the developmental milestone ( fine/motor/gross?) next to the the age group.
For the Milestones: These were the ones Dr. Mallari had emphasized( or the ones | was able to write only. Sorry !) . It is absolutely similar to the
one in Nelson’s. We had attached it at the back part of this ratio + the one in BRS Pedia
Fetal:
wk 13-14: breathing and 4 months: Toddler Period: Older kids: Milestones shorter,
swallowing motions appear. -Turns to sound/voice Rapid growth and development established, just improving
grasp reflex : 17 wk and is well -Rolls over 2 years old: 4 yrs old: stand and can hop on 1
developed by 27 wk. -transfers objects Runs well foot, copies a square
Eye opening: around 26-28 wk. 6-8 months: Phrases: 2 word, 50% of words -100% words spoken are clear
-Sits without support at 6 months clear ( expected) 3-step commands
Neonatal Period: -Holds bottle ->> simple tasks, play parallel -Group play
-Able to Hear -finger feeds
-Point of Clearest Vision: 20-30 -Responds playfully at the mirror 3yrs old: Tricycle, 75% words 5 years old:
cm -holds objects and put in mouth Copies circle (no corners) -Can now skip
Touch- ist & most developed 9 months: -> pretends plays with coll -shares stories
-pulls to stand, better leg control -starts to plays with other kids -Quiet when playing hide and
3-months: Good head control -pincer grasp seek
-Hand open/grasp -Does pat-a-cake or peek-a-boo Preschool: Test the limits,
-Coos & says “Aaaa” 1 year: expands social sphere School Age:
-Alert to Human voice -Walks alone ( wobbly walk) -Explore emotional separation 6-12
-Eats using spoon —> inc cognizant of limited -> Plays with friends of the same
abilities and constraints imposed gender
by adults -> Acdemic skills

7 years oled: does chores


9: copies cylinder

Part C: not sure if there was a identify if ADHD/ Autism/ Down Syndrome

(GO}FEU-N RMF PGI 2020 #WE-WILL-ALL-PASS! GREEN DRAGONS 5PEAT!


© PEDIATRICS BCA AUG 2019 + EXIT EXAM+PRACS (MAY 2020

&
3. Camitta’s criteria for severe aplastic anemia is
characterized by:
a. ANC 500, platelet 20,000, retic count 1%, bone marrow
( The Hema part of the exam was samplex :D — that is, cellularity of >25%
if you can find the ones she wil use xD ) b. ANC > 500, platelet >20,000, retic count >1%, bone
marrow cellularity of >25%
1. A10 year old boy was brought to the clinic because of c. ANC <500, platelet >20,000, retic count > 1%, bone
insidious onset of petichiae. On PE, he was playful, marrow cellularity of >25%
afebrile with occasional petechiae and no d. ANC <500, platelet <20,000, retic count <1%, bone
organomegalies. Your initial diagnosis is: marrow cellularity of 25%
a. Acute ITP Correct ans: D (no. 4in midterms) Camitta's criteria for
b. Chronic ITP severe aplastic anemia is characterized by: ANC <500,
c. Acute Lymphocytic Leukemia platelet <20,000, retic count <1%, bone marrow cellularity
d. Acute Myelogenous Leukemia of 257% CAMITTA CRITERIA (SAA = severe plastic anemia)
Correct ans: B ( slight modification from the midterm
* BLOOD (Peripheral)
question)
Predictors of Chronic ITP o ABSOLUTE NEUTROPHIL COUNT = <500
o Purpura 2-4 wks before diagnosis o PLATELETS = <20,000
o Female o RETICULOCYTE = <1%
o Age >10 years old
* BONE MARROW
o Higher platelet count
source: DEON 2017 p.11 o SEVERE HYPOCELLULARITY (25%)

FEATURES OF CHRONIC AND ACUTE ITP


o MODERATE HYPOCELLULARITY
with <30%
ae aig cel tee hematopoietic cells
cam <20,000/UL 00
mmm children,
2:5 y/o | Adults, 2040 y/o | ne 4. Acute myelogenous leukemia, FAB M5-M5 will have a
Sex predilection fitit FM 3i1 {8000/0 positive cytochemical staining with:
|__| (20yofemole) | SETS Common fate a. non-specific esterase ALL — periodic acid Schiff
ecm Common 1-3 wks | Unusual Mss b. periodic acid schiff AML, FAB M1 — myeloperoxidase
c. sudan black -
as) (preceded by d. myeloperoxidase AML, FAB M4-5— non specific esterase
infxn) TCL 2-0 weeks (short) | Months
or years
Correct ans: B (no. 9 in midterms) Question was asking
Betore (insidious) for: Acute myelogenous leukemia, FAB M5-M5- M5 is
uu a Abrupt | Insidious
Sy Occurin 0 of "common 24) | “monoblastic” so NSE applies. This was a tricky question.
aie (?) In severe Usually absent But it was in EJG. xD
au cases | US a 0908
: 3. BONE MARROW CYTOCHEMISTRY
2. This type of alpha thalassemia presents with low grade When patient can’t afford immunotyping above, then do this
anemia and low A2 on electrophoresis: PERIODIC Cytoplasmic glycogen > red
a. one gene deletion Pla R ella g 80% lymphoblasts; fine staining in
b. three gene deletion
(PAS) myeloid
c. two gene deletion
d. four gene deletion (+) in ALL
Correct ans: C MYELO- Cytoplasmic granular enzymes> brown-
In the midterms, one gene deletion: with normal ateycay black
hemoglobin and low A2 on electrophoresis. Notice how
the hemoglobin here is labelled as low grade anemia.
75% myeloblasts
The deletion of 1 a-globin gene allele (silent trait) is not 1 O) ey 1 * Cytoplasmic lipids >black
identifiable hematologically. Specifically, no alterations BLACK « Myeloblasts
are noted in the MCV and mean corpuscular
NON ¢ Stain granules and Auer rods; monocytes
hemoglobin. Alpha Thalassemia occur most commonly in people A) ede and monoblasts (differentiates
from southeast Asia and China 7) 3 monoblasts from myeloblasts)
* Inherited impairment of A chain synthesis
(NSE)
. Functional abnormality in one or more of the fourA
globin genes on chromosome 16 *So if you want to differentiate lymphoblasts from
- Deletion of a gene or genes from alpha globin chain myeloblasts, send it for PAS, myeloperoxidase, and NSE.
@ 1Gene Deletion Silent Carrier
@ 2 Gene Deletion Thalassemia Trait or Minor
oO 3 Gene Deletion Hemoglobin H
°

GOTT years
oocee. » ~. 2020 #WE-WILL-ALL-PASS!
GREEN DRAGONS 5PEAT!
© PEDIATRICS BCA AUG 2019 + EXIT EXAM+PRACS (MAY 2020

5. This type of alpha thalassemia is characterized by 8. one of the following conditions is NOT classified as
transfusion dependency, low A2 and Bart's hemoglobin extracorpuscular hemolytic anemia:
on electrophoresis: a. hemologytic disease of the newborn
a. one gene deletion b. AIHA
b. three gene deletion c. Hemolytic transfusion reaction
c. two gene deletion d. Hereditary elliptocytosis
d. four gene deletion Correct Answer: D -> this was a midterm question that she
Correct ans: D (See ratio for no. 2) just reversed.
Corpuscular Hemolytic Anemia is anemia occurring
6. In this condition, severe infection can trigger acute WITHIN the RED BLOOD CELL usually congenital in origin
hemolytic crisis because of the deficiency of while NON-CORPUSCULAR HEMOLYTIC ANEMIA happens
cytoskeletons: OUTSIDE the RBC usually acquired conditions such as
a. G6PD deficiency HEMOLYTIC DISEASE OF THE NEWBORN (where antibodies
b. Thalassemia of the mother passes through the placenta(lIgG) and
c. Hereditary spherocytosis “attacks” the newborn - immune mediated hemolysis)
d. sickle cell anemia | Classification of Hemolytic anemia
Correct ans: C (in midterms, she added spectrin and Corpuscular
Ankyrin to the question) Booyah! Samplex: Membrane defect
>| Sperocytosis, clliptocytosis
Spherocytosis: spectrin/Ankyrin problem
Enzyme defect
HEREDITARY SPHEROCYTOSIS (HS) > Pyruvate kinase,
Hereditary spherocytosis is the most common defect
leading to anemia, affecting 1/5000 Europeans heme |= ss globin
Autosomal Dominant trait — — —
-| Quantitative ex. Thalassemia |
°o Hereditary ovalocytosis
. ———| Qualitative ex. Sickle cell
.- 7
Normal: >20% spherocytes in | Classification of Hemolytic anemia ]
smear in the feathery edge Extracorpuscular |
o Hereditary elliptocytosis { Immune
7 Usually does not produce anemia | Isoimmune i ~ Autoimmune

"Autosomal dominant, affecting | (Hamers disease oF NO [Idiopathic


1/2500 Europeans [incompatible BT | Warm antibody
o Hereditary pyropoikilocytosis Cold antibody
7. Usually results from double _| Non- Cold-warm
+, i mune
recessive or homozygous defect m
idiopathic || | a infxn. drugs, hemo |
fo, microangiopathic

Defect is in proteins of the membrane skeleton,


9. In acute promyelocytic leukemia, one of the ff.
o Usually spectrin or Ankyrin
markers will NOT show significant fluorescence in flow
t ti
7.A 12 y//o boy presenting with fever, significant weight Sepa.
loss, night sweats, and a solitary tumor in the left anterior b. HLA-DR
cervical area with occasional RS cells and numerous ‘
: oe c.CD 13
lymphocytes on histopatholgy is diagnosed to have: d. CD 33
a. Hodgkin's lymphoma, Stage IA
correct answer: A Please memorize the table by heart!
b. Hodgkin's lymphoma, Stage IB
She rotates here questions around here!
c. Non-Hodgkin's Lymphoma, Stage IA
BONE MARROW PHENOTYPES — can request specific markers but usu request standard leukemia panel;
d. Non-Hodgkin's Lymphoma, Stage IB
CD10 aka CALA: mixed = 2 markers of L, 2 of M
Correct Answer: B Not in the midterms, but this is how she
usually gives here bonus shiftings. Take note that RS CELLS- Bou | co5, 00%, ALADR
> that should ring a bell to you that this is HODGKIN”S EARLY PRE-B CD 19,
CD 20, CD 10, HLA-DR
LYMPHOMA. A\so, it was mentioned that the kid has PRE-B CD 19,
CD 20, CD 10, HLA-DR
fever, significant weight loss, night sweats -> these are B MATURE B-CELL | CD 19, CD 22, HLA-DR, KAPPA, LAMBDA
symptoms in the Ann-Arbor Staging. Please refer to the
Deon Trans for other ways to distinguish it from NHL.
Te ——_| (3,005,007, HLA-DR
MIXED LINEAGE | Lymphoid and myeloid markers
STAGING: Ann Arbor AUL CD 38, CD 7, HLA-DR
* A=absence MYELOID HLA-DR
* B= 10% weight loss, fever, night sweats
* — Eg: “Hodgkins IIB”
Single node region or single extranodal site
2 2 nodes/ sites on same side of the diaphragm
localized nodes/ sites both sides of the diaphragm
- F PGI 2020 #WE-WILL-ALL-PASS! GREEN DRAGONS 5PEAT!
diffuse spread (bone marrow) —
PEDIATRICS BCA AUG 2019 + EXIT EXAM+PRACS (MAY 2020

10. The acquired form of hypoplastic anemia can be Recovering nutritional deficiency on iron/folate/B12
differentiated from Diamond Blackfan syndrome by the therapy myelostimulant; initial response is to
following lab findings: increase reticulocytes
a. elevated ADA, elevated Hab F, elevated | antigen
Bone marrow failure syndrome- disorders may
b. elevated MCV, normal Hgb F, elevated ADA
c. elevated ADA, normal Hgb F, normal | antigen
manifest as single cytopenia or as pancytopenia
d. normal MCV, normal Hgb F, noromal ADA
Correct Ans: D Occurs in individuals who produce insufficient
TRANSIENT ERYTHROBLASTOPEMA OF CHILDHOOD (TEC) amount of RBC, WBC, platelets. It includes aplastic
Most common ACQUIRED pure red cell aplasia anemia, myelodysplastic syndromes and
(PRCA) in children
Sewere transient hypoplastic amemia im prewiously
paroxysmal nocturnal hemoglobinuria.
healthy children between 6 months and 3 years of

age As for Fe-Def Anemia, the reticulocytes are


Follows a viral infection expected to be low. Hence, D is the answer.
Transient immunologic suppression of erythropoiesis

Laboratory Findings:
13. In childhood non-hodgkins lymphoma, this
4 reticulocytes
+ bone marrow erythroid precursors
stage is associated with the worst prognosis.
A NHL, Stage |
ooooo0

MOY monmall
ADA monomal B NHL, Stage l
Heb F normal
C NHL, Stage Ill
Treatment: PREC
D NHL, Stage IV
Prognosis: recovery in 1-2 months (pood response)
Correct Ans: D
pee fe ee ed ee ee) PROGNOSIS
Congenital pure red cell hypoplasia * Stage | and Il 90-100% SR (survival)
Symptomatic im early infancy < 1 year old
¢ Stage Ill, IV 60-95% SR
T fetal Heb
« Lymphoma-leukemia worst prognosis
4 expression of | antigen
T ADA (erythrocyte adenosine deaminase activity)
o If >25%lymphoma cells
tran overload if > L000 — work for organ dysfunction: o Differentiate from NHL by BM problems:
Hemosiderosis (pancreas, liver, heart) § NHL has tumor CLUSTERS
14, Overwhelming infection can trigger acute hemolytic
11. This surface antigen confers good prognosis among crisis in this/these condition/s:
patients with low white count acute lymphocytic a. G6PD deficiency
leukemia b Hereditary spherocytosis
a. cd 20 c Thalassemia
b.cd 19 cd All of the above
c.cd10 Correct ans: D
d. cd 22 G6PD deficiency: HEMOLYSIS IN G6éPD
correct ans: C The problem of G6PD in RBC
CD 10 ="CALLA” (Common Acute Lymphocytic ¢ Inability of RBC to synthesize G6éPD than old red cells
Leukemia Antigen) indicates GOOD PROGNOSIS = « Age dependence of red cell G6PD.
“PRE” cell 0 Reticulocytes have about 5x more activity than RBCs
12. Anemia with reticulocytosis is seen in the * RBC GéPD t1/2 of 60 days
* Destroyed by spleen — acute hemolysis crisis
following conditions, EXCEPT:
Hereditary spherocytosis:
a. hemolytic diseases Diagnose by f osmotic fragility
b. occult or overt bleeding ¢ Aplastic crises with Parvovirus B19
c. recovering nutritional deficiency ¢ Will see soherocytes with feathery edges
d. iron deficiency anemia Thalassemia: Can't find anywhere in the trans, but hey,
correct ans: D since a and b are like correct, so all of the above baby!
BLOOD LOSS may possibly present with
reticulocytosis on the peripheral blood smear 15. This condition is an example of an immune mediated
Low Production hemolytic anemia:
a. cooley's anemia
Hemolytic disease/Occult /overt bleeding
b. sickle cell anemia
Possible hookworm infection c. hemolytic transfusion reaction
d. CCNSHA

(GO}FEU-N RMF PGI 2020 #WE-WILL-ALL-PASS! GREEN DRAGONS 5PEAT!


y= PEDIATRICS BCA AUG 2019 + EXIT EXAM+PRACS (MAY 2020

Al PCD (ACQUIRED PROTHROMBIN COMPLEX DEFICIEN


The risk factors for the development of acquired prothrombin complex deficiency
Correct ans: See chart for no. 8. The rest of the chocies
does NOT include: oral vitamin K intake (kasama yung: malabsorption of vitamin K,
are examples of non-immune anemia.
prolonged antibiotic intake, breastfeeding)
Corpuscular Hemolytic Anemia is anemia occurring
These are risk factors for the development of acquired prothrombin complex
WITHIN the RED BLOOD CELL usually congenital in origin
deficiency EXCEPT: aquamephyton (kasama yung: malabsorption of vitamin K,
while NON-CORPUSCULAR HEMOLYTIC ANEMIA happens prolonged antibiotic intake, breastfed infant)
OUTSIDE the RBC This is NOT a risk factor for the development of acquired prothrombin complex
16. A patient with severe classic hemophilia will benefit deficiency: oral vitamin K intake (kasama yung: malabsorption of Vitamin K,
from the following treatment modalities: prolonged antibiotic intake, breastfed infant)
a. FFP, Cryoprecipitate This/these is/are risk factors for the development of acquired prothrombin
b. FFP, Factor9 concentrate complex deficiency: Malabsorption of Vitamin K, Prolonged antibiotic use,
c. FFP, Factor8 concentrate Breastfed infant (AOTA)
d. FFP, cryoprecipitate, factor 8 concentrate
Correct ans: D This was tricky, but in EJG, it was shown
that you can give FFP to Classic Hemophilia, though it
Post neonatal Vit. K deficiency
was notin Deon Trans.
HEMOPHILA A: CLASSIC: FACTOR 8 rhymes with Breasted
cryoprecipitate
lack of oral intake of Vit. K, alterations in the GUT
HEMOPHILA B: Christmas Factor? You give them
something FRESH and FROZEN- it's cold in CHRISTMAS flora due to prolonged intake of broad spectrum
right? So fresh frozen* plasma!
antibiotics, malabsorption of Vit. K
A patient with severe classic hemophilia
will benefit from the following treatment
modalities EXCEPT: factor 9 concentrate (kasama yung: factor8 precipitate, fresh fresh frozen plasma until patient wakes
frozen plasma, cryoprecipitate) Change in sensorium, seizures
This type of clotting factor deficiency is also called Christmas disease: factor 9
deficiency 18.)Patients with DIC requiring transfusion should be
Severe hemophilia will have a factor activity of: < 1% transfused with the following blood products as follows:
This type of clotting factor deficiency is also called Classic hemophilia: factor & a. FFP-> Cryoprecipitate-> platelets
b. cryoprecipitate-> FFP->Plateletes
deficiency
c. platelets-> cryoprecipitate-> FFP
Continuous bleeding occurs in patients with Classical hemophilia with this severity: d. FFP-> platelets-> cryoprecipitate
severe
A patient with severe classic hemophilia will benefit from the following treatment Correct Answer: C. platelets-> cryoprecipitate-> FFP
modalities, EXCEPT: Factor 9 concentrate (kasama yung: Factor 8 concentrate, PCF
Fresh frozen plasma, Cryoprecipitate} It's like a rainbow.
PCFFP_ -> you start with P and end with FP, with C in
Hemophilia A Hemophilia B tury) a
between
(Classic) Cet (Autosomal)
* Deficiency of * Deficiency of * Deficiency of Thrombotic microangiopathy
Consumption of clotting factors, platelets and
factor VIII factor IX factor Xl anticoagulation proteins
Widespread deposition of fibrin tissue ischemia,
* 85% * 15% * Autosomal mecrosis, generalized hemorrhagic state, hemolytic
* Most common * Use FRESH * Mild form anemia
Triggers: Hypoxia, Acidosis, Tissue mecrosic, Shock,
sex-linked frozenplasma Endothelial damage
AClOM panies sewere systemic disease
coagulopathy Clinical manifestations:
= Seeding, petechiae, eochymoses, tissue
* Use
necrosis, anemia (microamgiopathic HA}
cryoprecpititate Laboratory findings:
co Low fibrinogen, prothrombin, Factors Vv and
wu
17.) These are risk factors for the development of e Prolonged PT, PTT, TT, low platelets
e Elevated FOP, O-dimer assay
acquired prothrombin complex deficiency: Treatment
a. malabsorption of vitamin K o Treat underying disorders trigger
o Restore nonmal hemostasis by correcting
b. prolonged antibiotic intake
shock, acidosis and hypoxia
c. breastfed infant Blood components
d. all of the above o Platelets (27 given), cryoprecipitate, FFP
Activated Protein C Concentrate (for DiC due to
Correct Answer: D. All of the above sepsis)

(GO}FEU-N RMF PGI 2020 #WE-WILL-ALL-PASS! GREEN DRAGONS 5PEAT!


19.) Splenectomy is indicated in patients with thalassemia major 2.) A diagnosis of inital attack of Rheumatic fever is made in the
with signs of hypersplenism and transfusion index of: presence of which of the following?
a. 400 cc/kg/year a.Carditis, pharyngitis and high ASO titer
b. 300 cc/kg/year b.Migratory polyarthritis and elevated acute phase reactants, high
c. 200 cc/kg/year ASO titer
d.100 cc/kg/year c.Arthralgia, prolonged PR interval on ECG, high ASO titer
Correct Answer: C. 200 cc/kg/year d.Erythema marginatum, arthralgia, elevated ESR and CRP, High ASO
SPLENECTOMY if symptoms; Belowis mandatory: titer
a Correct: D Five MAJOR criteria Four MINOR criterti
o Transfusion index 200 cc/kg/year ar . [ine omera [row cena
Jones Criteria (memorize!) oe 4 .
"Max limit for transfusions Initial attack years © arthralgia
. . . . Migratory Wy Fewer
0 Hypersplenism : major mannestations or Polyarthritis (tor? | ¥Flevated Acute
ign: Give
" — Sign: Gi 3 bags to correct to 40 = evidence of recent GAS infection jainits,‘es
an big joints,5] Phase CRP)
(ese Reactant
ants

.
Hematocrit still low
_ . .
vt
marginatum
dene,
ECG: Prolonged PR
20.) This type of alpha thalassemia is characterized by transfusion ¥ Subcutaneous interval
dependency, low A2 and Bart's hemoglobin on electrophoresis: nodule
a. one gene deletion v¥charea
b. three gene deletion
c. two gene deletion
d. four gene deletion 3.) A 10 y/o boy presented with recurrent pharyngitis associated
Correct Answer: D. four gene deletion swelling on the left knee and right ankle joint and holosystolic
murmur at the apex. Which major criteria in the diagnosis of ARF
Peal 2 are present in this patient?
A Carditis and subcutnaeous nodule
* — Alpha Thalassemia occur most commonly in people
B Carditis and migratory polyarthritis
from southeast Asia and China C Carditis and chorea
* — Inherited impairment of A chain synthesis D Carditis and erythema
Correct: B. Carditis and migratory polyarthritis
* Functional abnormality in one or more of the four A
BEC M CKO MMC eu ROU ECU Rommel em uC tome Cm Ala OCU MU se
1c Te
globin genes on chromosome 16 2015)'*
* Deletion of a gene or genes from alpha globin chain MAJOR SUPPORTING EVIDENCE OF ANTECEDENT
MANIFESTATIONS MINOR MANIFESTATIONS GROUP A STREPTOCOCCAL INFECTION
o 1Gene Deletion Silent Carrier
Carditis Clinical features: Positive throat culture or rapid streptococcal
Oo 2 Gene Deletion Thalassemia Trait or Minor Polyarthritis Arthralgia antigen test
Erythema marginatum Fever Elevated or increasing streptococcal antibody titer
oa 3Gene Deletion Hemoglobin H Subcutaneous nodules Laboratory features:
Chorea Elevated acute phase reactants:
Erythrocyte sedimentation rate
oO C-reactive protein
Prolonged P-R interval
From Guidelines for the diagnosis of rheumatic fever. Jones Criteria, 2015 update. Special Writing Group of the Committee on Rheumatic Fever, Endocarditis, and
Kawasaki Disease of the Council on Cardiovascular Disease in the Young of the American Heart Association (in press).
1, Initial attack: 2 major manifestations, or 1 major and 2 minor manifestations, plus evidence of recent GAS infection. Recurrent attack: 2 major, or 1 major and2
minor, or 3 minor manifestations (the latter only in the Moderate/High-Risk population), plus evidence of recent GAS infection (see text).
2. Low-Risk population is defined as ARF incidence <2 per 100,000 school-age children per year, or all-age RHD prevalence of <1 per 1000 population. Moderate/
High-Risk population is defined as ARF incidence >2 per 100,000 school-age children per year, or all-age RHD prevalence of >1 per 1000 population.
3. Carditis is now defined as clinical and/or subclinical (echocardiographic valvulitis). See Table 183-3,
( The Acquired Heart Disease part of the 4, Arthritis (major) refers only to polyarthritis in Low-Risk populations, but also to monoarthritis or polyarthralgia in Moderate/High-Risk populations.
5. Minor criteria for Moderate/High:Risk populations only include monoarthralgia (polyarthralgia for Low-Risk populations), fever of >36° C (>38.5° C in Low-Risk
exam was also samplex :D — that is, if populations), ESR >30 mm/hr (>60 mm/hr in Low-Risk populations).

you can find the ones she wilL use xD ) 4.) Which heart murmur suggests the presence of aortic
insufficiency?
1.) Which age group of patients will have a high mortality rate from Holosystolic murmur at the apex
viral myocarditis? Holosystolic murmur at the left lower sternal border
0-2 years Diastolic murmur at the apex
>2-5 years Diastolic murmur at the right upper sternal border
>5-10 years Correct: D. Diastolic murmur at the RUSB
10-15 years Mitral insufficiency- Holosystolic murmur at the apex
ans: A PROGNOSIS Mitral stenosis- Diastolic murmur at the apex
‘The prognosis of symptomatic acute myocarditis in newborns is poor, Aortic insufficiency- Diastolic murmur at the right upper sternal
and a 75% mortality has been reported. The prognosis is better for border
children and adolescents, although patients who have persistent evi- Aortic stenosis- Systolic murmur radiating to the neck
dence of DCM offen progress to need for cardiac transplantation.
Recovery of ventricular function has been reported in 10-50% of
patients, however.
(GO}FEU-N RMF PGI 2020 #WE-WILL-ALL-PASS! GREEN DRAGONS 5PEAT!
5.) In patients with ARF, what treatment is given to eradicate Rheumatic Heart Disease is the most common cause of acquired
streptococcal organisms in the body? heart disease among children 5-15 y/o in underdeveloped countries,
Erythromycin 250 mg two times a day crowded places, and low income group of the society.
Oral Penicillin 250 mg two times a day
Benzathine penicillin G single injection 9.) What will be the basis for the diagnosis of an initial attack of
Benzathine penicillin G every 21-28 days Rheumatic fever —- Rheumatic heart disease?
Correct: C. Benzathine Penicillin G single injection a. 2 major criteria
Antibiotic therapy, 10 days oral penicillin or amoxicillin or single IM
injection of long acting penicillin (BPG) or 10 days erythromycin, b. 2 major plus 2 minor criteria
azithromycin) (Sdays) or clindamycin, then long term antibiotic
prophylaxis. c. 1 major plus 2 minor criteria plus elevated ASO titer
Primary prevention - eradicate streptococcal organism still present
in the patient d. 2 minor criteria plus elevated ASO titer
Secondary prevention - continuous antimicrobial treatment to
prevent recurrence (Benzathine penicillin 1.2M units IM after cORRECT: c
negative skin test every 21-28 days) Why not A? 2 major manifestations? Previous simplexes had shown
C. is the answer. And someone had said that Dr. Latosa said C is the
6.) Which complication of Rheumatic Fever- Rheumatic Heart best choice.
Disease presents with cardiomegaly involving the left ventricle and
left atrium? Using the 2015 AHA revised Jones Criteria
Mitral regurgitation Initial attack - 2 major manifestations or
Aortic regurgitation
Mitral stenosis 1 major and 2 minor plus evidence of recent GAS infection
Aortic Stenosis Recurrent attack - more of Carditis, Erythema Marginatum and
Correct: Mitral Regurg Subcutaneous Nodules
One of the treatment for RHD is to closely monitor for evidence of
carditis which most commonly presents with a REGURGITANT 2 Major or 1 Minor and 2 minor or 3 minor manifestations plus
MITRAL VALVE producing a HOLOSYSTOLIC or PANSYSTOLIC
MURMUR at the apex radiating to AXILLA and BACK. evidence of recent GAS infection

7.) Among the major manifestations in the Jones criteria for 10.) A 10 year old boy presented with recurrent pharyngitis
diagnosing Acute Rheumtic fever, which one will present weeks to associated with pain on the left knee, swelling of the right ankle
months after a bout of pharyngitis and will be diagnostic of the and exertional dyspnea. Which major criteria in the diagnosis of
disease? acute rheumatic fever is present in this patient?
Arthritis Erythema a. Carditis and subcutaneous nodule
Carditis Chorea b. Carditis and migratory polyarthritis
c. Carditis and chorea
Ans: Chorea d. Carditis and erythema marginatum
Chorea may occur solely as a symptom, and manifest 1-2 months
after the throat infection. Often occurs as an isolated manifestation Correct Answer: b. Carditis and migratory polyarthritis
after the resolution of the acute phase of the disease and excluding
other possible causes like CNS conditions. (AJCP trans page 3) pain on the left knee, swelling of the right ankle > migratory
The latent period from acute GAS infection to chorea is usually polyarthritis
substantially longer than for arthritis or carditis and can be months.
Onset can be insidious, with symptoms being present for several exertional dyspnea > carditis (exertional dyspnea can also be
months before recognition. (Nelson’s 20th page 1334) associated with cardiac problem as the malfunction of the heart can
lead to inadequate oxygen supply)
8.) In underdeveloped countries, which is the most common cause
of acquired heart disease among children? 11.) What is the most commonly heard heart murmur in the initial
attack of Rheumatic fever?
a. Rheumatic fever— Rheumatic heart disease a. Holosystolic murmur at the apex
b. Kawasaki disease b. Holosystolic murmur at the left lower sternal border
c. Diastolic murmur at the apex
c. Infective endocarditis d. Diastolic murmur at the right upper sternal border
Correct: A
d. Myocarditis The most commonly associated valvular dysfunction in the initail
Correct answer: a. Rheumatic fever —- Rheumatic heart disease attack of RF is mitral regurgitation which is heard as Holosystolic
murmur at the apex radiating to the axilla.

(GO}FEU-N RMF PGI 2020 #WE-WILL-ALL-PASS! GREEN DRAGONS SPEAT


WE
Eg
i = PEDIATRICS BCA AUG 2019 + EXIT EXAM+PRACS (MAY 2020):

15.) Among the different causes of Infective Endocarditis, this


12. (congenital <3, but) JM, 6y/o, was noted to have a heart microorganism is often implicated among intravenous drug users:
murmur on routine PE. Pertinent findings showed an RV heave, A.Pseudomonas aeruginosa
= Staphylococcus endocarditis in
widely split and fixed $2 and systolic ejection murmur at LUSB. B.Group D Enterococcus patients with no underlying heart
C.Staphylococcus aureus disease
What is the most likely Diagnosis?
Streptococcus viridans after
A.PDA D.Streptococcus viridans
dental procedure
B.ASD Group D enterococcus after
bowel and GU manipulation
C.VSD * Viridans — type Streptococci and
Staphylococcus aureus > Pseudomonas aeruginosa in |V
D.AVSD leading causative agents in drug users
Pediatric patients
ANS: B. ASD = Blood cultures negative in 6% of
Fungal organisms after open
cases heart surgery
Coagulase negative Staph in
Incidental finding of murmur because patient is asymptomatic. patients with indwelling catheters
(usually during routine PE) kaya school age na nakikita.
*Pertinent findings: (+) RV Heave, systolic ejection murmur 16.) A 5-yr old male with a history of fever followed by chest pain
*Widely spilt and fixed $2 (Si PAT nag split) was rushed to the emergency department due to sudden collapse.
-PS He may be suffering from:
-ASD A.Kawasaki Disease
-TAPVR B.Rheumatic Fever
*Remember also: C.Acute Pericarditis
ASD- RV heaVe (Volume overload) D.Acute Myocarditis
PS - RV taP (Pressure overload) Correct: C
Normal pericardial fluid (PF) volume = 15 mL.
13.) The most common cyanotic heart leasion during infancy is: When the amount of fluid becomes excessive, pressure within the
A.TOF pericardium increases and is transmitted to the heart resulting to
B.TGA impaired filling ---> cardiac tamponade ---> shock and death
C.TAPVR If there is excessive accumulation of PF because the pericardium is
D.TVA inflamed ---> pericarditis. If mild = chest pain. If moderate = low CO.
Ans: B If severe, patient may go ona collapse due to tamponade.
CM: Depends on the amount of fluid in the pericaridial cavity.
Most common congenital heart defect in PREMATURE: PDA
Tamponade is a medical emergency. Aspirate pericardial fluid to
Most common congenital heart defect in NEONATAL period: relieve tamponande.
TGA
17.) A child previously diagnosed with heart disease, consulted a
Most common congenital heart defect BEYOND neonatal doctor with complaints of fatigue while playing. This was assoc.
period: TOF (TO inEinity and BEYOND) with fast heart rate and DOB. PE showed tachycardia, holosystolic
murmur at the apex and gallop rhythm. This child has s/sx
Most common acyanotic congenital heart defect: VSD
indicative of:
A.Constrictive Pericarditis
14.) What is the treatment given to patients with ARF that is B.Restrictive Cardiomyopathy
directed against recurrences of streptococcal throat infection? C.Rheumatic Heart Disease
a. Erythromycin for 10 days D.Heart Failure
b. Aqueous penicillin for 10 days Correct: D
c. Benzathine penicillin G single injection RATIO: A condition wherein the
d. Benzathine penicillin G every 21 — 28 days Heart failure is a clinical heart cannot deliver
Correct: D syndrome — a diagnosis is adequate cardiac output
made only through history (CO) to meet the metabolic
PREVENTION OF RF and physical examination.
needs of the body, for
Both initial and recurrent episodes of acute RF prevention depends children, including the
on controlling GAS infections of upper respiratory tract 3 most common causes of requirements for growth.
heart failure among
Primary prevention children:
HF occurs when the heart
1. Carditis
cannot deliver adequate CO
Treatment, preventing or eradicating streptococcal infection 2. Myocarditis
3. Dilated cardiomyopathy to meet the metabolic needs
Secondary prevention of the body.
Goal is to prevent recurrences and prevent further in developing
into Rheumatic Heart Disease 18.) Using the Dukes criteria, which of the following is diagnosis for
infective endocarditis in children?
*Penicillin G Benzathine 1.2 million units IM every 21 days and 28
days until 21 years of age or longer (40 years old) a. fever, chills, abdominal pain, chest pain
b. heart murmur, splenomegaly, seizure

(GO}FEU-N RMF PGI 2020 #WE-WILL-ALL-PASS! GREEN DRAGONS 5PEAT!


c. positive blood culture done twice and valve dysfunction on Infants <6months and children >5 years were at the
echocardiogram highest risk for CAA in the latest Japanese survey
d. infiltrates on chest xray and low voltage QRS complex of EKG ¢ The younger the patient = the greater risk of coronary artery
Correct Answer: c. positive blood culture done twice and valve complications
dysfunction on echocardiogram Aneurysm can be small and solitary to large or
2 major criteria were met: numerous
1) (+) blood culture — 2 separate cultures for a usual pathogen, 2 or ¢ Acute thrombosis in an aneurysmal or stenotic
more cultures for less typical pathogens coronary artery aneurysm
2) Evidence of endocarditis on echo ¢ Percutaneous transluminal coronary angioplasty
(PTCA) and stent placement is the treatment for
*Definite diagnosis can be made if 2 major criteria OR 1 major + 3 stenotic coronary artery.
minor OR 5 minor criteria are met
KAWASAKI DISEASE
DUKE CRITERIA + In Japan, 2010 survey had the highest recorded
Definite Diagnosis: + Classified under the rheumatic diseases of rate of 239.6/100,000 children ages 0-4 years with
= 2 major criteria childhood but discussed in acquired heart diseases —_ highest rate in very young children ages 6-11
| major (most likely Echo evidence of because complications in the particularly the months.
coronary arteries. + Infants <6months and children =5 years were at the
Endocarditis) and 3 minor
+ Adisease
of childhood, higher susceptibility
in highest risk for GAA in the latest Japanese survey
= $ minor criteria (not very sure that boys. * The younger the patient = the greater risk of
endocarditis but treat the patient * Who are at risk: Children <5 years old but there coronary artery complications
endocarditis) aré also children ages 5-10 years old (not the usual
¥ Major Criteria age group/uncommon) who May acquire KD.
3 (+) Blood culture 2 separate cultures for a + Children of Asian and Pacific Islander descent had
usual pathogen, 2 or more cultures for less highest hospitalization rates of 30.3/100,000
typical pathogens if patient received children, compared with 17.5/100,000 black, non-
antibiotic therapy before blood culture Hispanic children (more common in Asians than
may be low yield black race) ey
o Evidence of endocarditis on echo =
¥ Minor Criteria GROSS MOTOR MILESTONE:
2 Predisposing conditions (e.g. Heart Birth Turns the head side to side
Surgery) 2 months Lifts the head when lying prone
9 Fever Head lag when pulled from the supine position
o Embolic-vascular 4 months Rolls over

. ne a pense pee No head lag when pulled from the supine position
morn aees. 08 en noes. Pushes the chest up with the arms when yiag prone
CNS or ocular lesions, Janeway S th Sits al “
manths Sits alone
lesions)
Leads with the head when pulled trom the supine position
o Signs of immune complex phenomena
2 Single (+) blood culture or serologic 8-10 months Crawls
evidence of infection 9 nvonths Pulls to stand
o ©Echo signs not meeting major criteria Cruises

19.) Aside from gentamycin, which antibiotic is given to infectiv bis moniths Walks

endocarditis caused by streptococcus viridans? BASIC LANGUAGE MILESTONES:


a. Aqueous penicillin Age Milestone
b. Erythromycin
Birth Attunes to human voice
c. Cefuroxime
d. Vancomycin Develops differential recognition of parents’ voices
Correct: A 2-3 months Cooing (runs of vowels), musical sounds (e.g, oel-ooh, aalt-aalt)
Aqueous crystalline penicillin G is the best answer. 6 months Babbling (mixing vowels and consonants together) [e.g., ba-ba-ba, da-da-da|
Cetriaxone sodium or ceftriaxone sodium + 9-12 months Jargoning (e.¢., babbling with mixed consonants, inflection, and cadence)
gentamicin sulfate may also be given but not included in the
Begins using mania, dada (nonspecific)
choices.
Vancomycin hydrochloride is recommended only for patients 12 months 1-3 words, mani and dada (specilic)

unable to tolerate penicillin or ceftriaxone 18 months 4-20 words


20.) What age group is considered high risk for the 2 years Over 50 words
development of coronary artery aneurysm in KD?
['wo-word telegraphic sentences (e.g. monty conte)
a. 0-6 months -> Corret
25-50% of child's speech should be intelligible
b. >6 months — 2 years
c. >2—3 years 3 years Three-word sentences
d. >3 —4 years More than 79% of the child’s speech should be intelligible

(GO}FEU-N RMF PGI 2020 #WE-WILL-ALL-PASS! GREEN DRAGONS 5PEAT!


PRIMITIVE REFLEX & POSTURE REACTIONS:

Description af Reflex/Reaction Appears


Primitive Reflexes
More reflec Symmetric abduction and extension of arms with trunk extension, followed by Birth 4 months of
adduction of upper extremities age
Hand grasp: Reflex grasp of any object placed in the palm Hirth 1-3 months
oF age
Atonie meck refles: If the head is turned to ome side, arms and legs extend on the same side and | 2-4 weeks | 6 months of
flex on the oppesite side (“fencer position”) [of age [age
Rooting reflex: Turing of the head toward the same side as stimulus when a corner of the Birth 6 months of
infant's mouth is stimulated | 45°
Postural Reactions
Head righting: The ability to keep the head vertical despite the body being tilted 4-6 months | Persists
of age
Parachute: Qutstretched arms and legs when the body is abruptly moved head first in a 3-9 months | Persists
downward direction of age

FINE MOTOR MILESTONES


Age Milestone
Birth Keeps hands tightly fisted
3-4 months | Brings hands together to midline and then to the mouth
4-5 months | Reaches for objects
6-7 months | Rakes objects with the whole hand. Transfers the object from hand to hand
9 months _ | Uses immature pincer (the ability to hold small objects between the thumb and index finger)
12 months | Uses mature pincer (the ability to hold small objects between the thumb and tip of the index finger)

*Normal infants show significant variation in the attainment of these milestones.


FROM NELSON’S PEDIATRICS 2020
Table 22.1 | Developmental Milestones in 1st 2 Yr of Life
AVERAGE AGE OF
MILESTONE ATTAINMENT (MO) DEVELOPMENTAL IMPLICATIONS
GROSS MOTOR
Holds head steady while sitting Allows more visual interaction
& oo bon

Pulls to sit, with no head lag Muscle tone


Brings hands together in midline Self-discovery of hands
O&O

Asymmetric tonic neck reflex gone Can inspect hands in midline


Sits without support Increasing exploration
Ralls back to stomach Truncal flexion, risk of falls
wi

Walks alone Exploration, control of proximity to parents


Runs Supervision more difficult
FINE MOTOR
Grasps rattle Object use
fw

Reaches for objects Visuomotor coordination


Palmar grasp gone Voluntary release
Transfers object hand to hand Comparison of objects
Ou

Thumb-finger grasp Able to explore small objects


Turns pages of book Increasing autonomy during book time
Scribbles Visuomotor coordination
Builds tower of 2 cubes Uses objects in combination
Builds tower of 6 cubes Requires visual, gross, and fine moter coordination
COMMUNICATION ANID LANGUAGE
Smiles in cesponse bo face, voice Mare active social participant
Moncsyilabic babble Experimentation with sound, tactile sense
Inhibits to “no” Response te tone [nonverbal
Follows T-step command with gesture Blonverbal comm; .enicesticon
Follows l-step command without gesture Verbal receptive language (¢.9g.. “Give it te re")
Says “mama” or “dada* Expressive lanquage
Points to objects Interactive communicetion
Speaks first real word Beginning ef labeling
Speaks 4-4 words. Acquisition of abject and personal names
Speaks 104-15 wards Acquisition of object and personal names
Speaks 2word senterces fe.g.. “Mammy shoe") Beginning grarmnmatization, coresponds with S0-word wocabulary
COGHITIVE
Stares momentarily at spot where object Lack of abject permanence (out of sight, aut of mind; c.g.. yarn
disappeared ball empaprecth
Stares at own hand Self-diseovery, cause and etiect
Bangs 2 cubes Active comparison of objects
Unceowers toy (after secing it hidden) Object permancnes
Eqocentric symbolic play (e.g, pretends to drink Beginning syrribelic thought
HTL CUED)
Uses stick to reach toy Able to link actions to solve problems
Pretend play with doll (eog., gives dell battled Symbolic thaught
Table 22.2 | Emerging Patterns of Behavior During the 1st Yr of Life
NEONATAL PERIOD (1ST 4 Wk)
Prone: Lies in flexed attitude; turns head from side to side; head sags on ventral suspension
Supine: Generally flexed and a little stiff
Visual: May fixate face on light in line of vision; doll’s eye movement (oculocephalic reflex} of eyes on turning of the body
Reflex: Moro response active; stepping and placing reflexes; grasp reflex active
Social: Visual preference for human face
AT1MO
Prone: Legs more extended; holds chin up; turns head; head lifted momentarily to plane of body on ventral suspension
Supine: Tonic neck posture predominates; supple and relaxed; head lags when pulled to sitting position
Visual: Watches person; follows moving object
Social: Body movements in cadence with voice of other in sacial contact; beginning to smile
AT 2 MO
Prone: Raises head slightly farther; head sustained in plane of body on ventral suspension
Supine: Tonic neck posture predominates; head lags when pulled to sitting position
Visual: Follows moving object 180 degrees
Social: Smiles on social contact; listens to voice and coos
AT3 MO
Prone: Lifts head and chest with arms extended; head above plane of body on ventral suspension
Supine: Tonic neck posture predominates; reaches toward and misses objects; waves at toy
Sitting: Head lag partially compensated when pulled to sitting position; early head ited with bobbing motion; back rounded
Reflex: Typical Moro response has not persisted; makes defensive movements or selective withdrawal reactions
Social: Sustained social contact; listens to music; says “aah, ngah”
AT4 MO
Prana: Litts head ane chest, with head in aperovimately vertical axis: lage extencled
Supine: Symmetric posture predominates, hands in midline; reaches and arasos objects end brings them te mouth
Sittirig: Bo heed lag when pulled to sting position head steady, tipped forward; enjoys sitting with full truncal suppert
Standing: When held erect, pushes with feet
Avdapsties: Seas raisin, but rakes ne mows ta reach for it
Social: Laugh: out loud; may show displeasure if social contactis broken; excited at sight of food
AT? MO:
Prone: Ralls. over; Pivots; crawls or creep-crawis (Knobloch)
Supa; Litts hese; rolls over, squenre
Sitting: Sits briefly, with support of pelvis; leans forward on hands; back rounded
Standing: hay support most of weight bourrces actively
Adaptive: Reaches out for and grasas lange object; tranders objects trom hand to hand, grasp uses radial palm; rakes at raisin
Lanquage: Forms polysyllabes vowel sounds
Social: Prefers mother; babbles; enjoys mirror; responds to changes in emotional content of social contact
AT 10 MO
Sitting: Sits up alone and indefinitely without support, with back straight
Standing: Pulls to standing position; “crulses” or walks holokyg on to furniture
Motor: Crecps or cra
Adaptive: Grasps cbjects with thumb and forelinger; pokes at ching with forefinger picks ua pellet with assated pincer move rent,
uncovers hidden tay; attempts to ratrieve dropmed object; releases abjact grasped by other parson
Language: Ranatitive consonant souncs fimama,” “dada"}
Social: Resoonds to sound of name; plays peck-aboo of pat-acake; waves bye bye

AT 1¥R
Mater: Walks with ene hand held: rises independently, taxes several steps (Knobloch)
Adaptive: Picks up raisin with unassisted pincer movement of forefinger and thumb; releases object to other person on request or gesture
Language: Says a lew words besides “mama,” “dada*
Social: Plays simple ball game: makes postural adjustment to dressing
‘Data ara derived from those of Gesell (zs revised by Knobloch), Shirley, Provence, Wolf, Bailey, and others,
Data Irom Knubluch H, Stevens F, Malona AF: Manual of develupmertal diagnosis, Hagerstown, MD, 1980, Harpar & Row.
"[Zenitsu,] Go the distance, You can run, you can cry, but . aay
don't give up. | believe in you. You've endured a hellish Thanks you Green Dragons for helping complete the ratio!
training regimen every single day. You will definitely be ey Ys ane tie
rewarded. Strike until it's at it's limit." SS a
-ligoro Kuwajima to Zenitsu Agatsuma
SRET, TRACE} ‘=
Kimetsu no Yaiba SUPPORT >
1.) At the 1st minute of life, a newborn was noted to have a Cardiac Rate of 140, with active
movement, grimace to stimulation, acrocyanosis and good respiration

A. What is the APGAR score? 8


Cardiac Rate -140-2
Muscle tone-good-2
Reflex- Grimace- 1
Color-Acrocyanosis-1
Respiration-good-2

B. What is the normal APGAR score? 7-10


C. What is the significance of APGAR score?
To assess for need of immediate resuscitation

HR Absent <=100 bpm >100 bpm


Absent, .
RR irregular Slow, crying Good

MUSCLE Lim flee oF Active


TONE P extremities
motion
REFLEX No Grimace Cough or
IRRITABILITY response sneeze

COLOR Blue, pale Acrocyanosis acennaed

7-10: NORMAL
4-6 BORDERLINE
<3: RESUSCITATE
1°" MINUTE: ASSESS NEED FOR RESUSCITATION
57 MINUTE: EFFECTIVENESS OF RESUSCITATION

2.) An irritable 3 y/o Male with history of vomiting and loose stools was seen at the OPD
with pertinent physical examination of: sunken eyeballs, dry lips, hyperactive bowel
sounds and was noted to drink eagerly.

A. What is the level of dehydration? Some Dehydration


sunken eyeballs + dry lips + Drinks eagerly -> Moderate dehydration -> If with 2 or more signs
in B-> SOME DEHYDRATION
B. Give management (Drug/s and dosage) including home instruction. (see photos below. Some
dehydration-yellow and plan B)

Does the child have diarrhoea?

Hfyes, ask: Look and feel:


© For how long? . Look at the child's
* Is there blood in the general condition. Is for DEHYDRATION_
stool? the child: Lethargic or
°

unconscious? Classify
Restless and | DIARRHOEA
°

. irritable? Look for


sunken eyes.
Offer the child fluid. Is
the child:
°

Not able to drink or


drinking poorly?
°

Drinking eagerly,
thirsty?
Pinch the skin of the
abdomen. Does it go
°

9 back: Very slowly


(longer than 2
seconds)?
Slowly?

and if diarrhoea 14
days or more

and if blood in stool

a
P.S. binago ko Ing ung picture coz may part na di mabasa kanina pero same siya sa pic :)
PLAN 8: TREAT SOME DEHYDRATION WITH ORS
In the clinic, give recommended amount of ORS over 4-hour period
» DETERMINE
AMOUNT OF ORS TO GIVE DURING FIRST 4 HOURS
WENGHT|~ 6 kg 6--10 kg 10 - <12 kg 12-19 kg
AGE* Up to.4 4 months wp to 12 12 months up to 2 2 yearsup bo >
months months years years
in mil 200 - 460 450 - 500 800 - 860 960 - 1600
”" Use
the chilis age oniy when you do not know ihe weight, The approximate amount of OAS
fRequined
jim mi) can atso be calculated by muMipiying the child's weight (in xg) times 75.
« Bethe child wants more ORS than shown, giwe mane
« Por infants under @ months who are mot breastied, also give 100 - 300 mi clean water during thes.
Berod if you use standard OFS. This is not needed if you use mew blow camolarity ORS.
s SHOW
THE MOTHER HOW TO GIVE ORS SOLUTION.
:* Ge frequent small sips from a cup.
« the child vomits, wait 10 minutes. Then continue, but more showy.
* Continue breastie=ding
whenever the child wants.
= AFTER 4 HOURS:
« Reassess the child and classify the child for dehydration.
« Select the appropriate plan to continue treatment.
« Begin feeding
the child in clinic.
s IF THE MOTHER MUST LEAWE BEFORE COMPLETING TREATMENT:
« Show her how to prepare ORS solution at home.
« Show her how much
OFS te give to fmesh 4-hour treatment at home
« Give her enough ORS packets
to complete rehydration.
Also give her 2 packets
as. recommended
in Plan
AL
« Eoplain the 4 Rules. of Home Treatment:
1. GIVE EXTHA FLUID

25 - 428

20
L I.
Lf _Alighi Fask’Zene ———— —-
Serum Bilirubin (mg/dl)

Loess st carciaacesaaedsvave baatavtentchaaces | eanctamenastatssaal| Messier anemTad aeseescen ce : L_———] Be cccccseee saeViitats anetae aturivavitasiedetege apres araithncaaaecd Ss ceeniracuraineal ates

15 Test a 257
— seat bem
A ee ee
Nice se scel ats edt? Sere recites eteea nathan
ee : eee

pmol/L
moe
hs

[ aes es yo wa = eal
le ap ae soe
10 Woe a 471

5 85

9 eet pee peorfiitis 0


0 12 24 36 48 60 72 84 96 108 120 132 144
Postnatal Age (hours)
3.) Baby Girl Ais on her 58th hour of life. You noted that the jaundice was down to the
abdomen. Bilirubin Levels revealed Total Bilirubin 14.3 mg/dl,B1 12.2 mg/dl and B2 2.1
mg/dl.

a.Using the Bhutani chart at what zone will the bilirubin levels fall?
High-intermediate Risk zone ( because baby is on the 58th hour of life with a total serum
bilirubin of 12)

B. Using Kramer’s Chart what will be the approximated bilirubin levels of this patient?

jaundice was down to the abdomen -> 14 mg/dl ( | can’t find this sa Nelson’s )
Table 1. Visual Assessment of Neonatal Jaundice (Kramer's rule)

Range of Serum Bilirubin


Area of the Body
umoll, mg/dL
Head and neck 68 - 133 4-8
Upper trunk (above umbilicus) 85 - 204 5-12
Lower trunk and thighs 136 - 272 8-16
(below umbilicus)
Arms and lower legs 187 - 306 11-18
Palms and soles 2306 218

C. What will be your plan of management?


-Firstly, phototherapy is indicated in High-intermediate risk zone.
-Explain that it resolves in 1 week in full term infants
-Interview patient to assess for need to do COOMB’s Test.
-Educate about breastfeeding and how it may cause jaundice

4.) Baby Zoe a7 month old female came in the OPD for well baby check up.

What is her IBW? 7 x 500 + 3000 grams = 6500 grams


7-12 months: Age (in mos) x 500 + BW
*Average weight at birth: 3000 grams

Pwede po ba ito? 7months *9/2=11.5 kg

What is the expected HC? 44.5 cm


At birth head circumference is 35 cm ( normal babies
- Should be monitored routinely during the first 3 years of life
o Especially in the first 2 year
Measured over the most prominent part of the occiput and just above the supraorbital ridges
Age Growth in HC / month
Birth — 3 months 2cm
3-6 months icm
6-12 months 0.5 cm
1-3 years 0.24 cm
4-6 years lem/year
Assuming Birth Head Circumference is 35 cm -> She is 7 months:
35 cm ( birth ) + (6 cm -> 1st 3 months has 2 cm for the first month) + (3 cm -> 2nd 3 months has 1 cm per month) +
(0.5 cm -> for the 7th month) = 44.5 cm

Compute for the number of teeth/s expected for this age group?
7-months old: so 2 central incisors from mandibular area ( 5-7 months) + 2 central incisors
on maxillary area ( 6-8months) + 2 lateral incisors from mandibular area ( 7-10 months) = 6
teeth-> It is expected for her to have 6 teeth

Cinco trans:
Eruption usually occurs at 6 months mandibular central Incisors
o You can wait until 15 months of age
- Eruption of permanent teeth begins at around 6 y/o and completed around 18 y/o
- Visit to the dentist could be done as early as the first tooth erupts

Nelsons:
Initial mineralization begins as early as the 2nd trimester (mean age for
central incisors, 14 wk) and continues through 3 yr of age for the primary
(deciduous) teeth and 25 yr of age for the secondary (permanent) teeth.

Eruption begins with the central incisors and progresses laterally.


Nelsons:
Central incisors: 5-7 months ( mandibular), and 6-8 months ( maxillary).
Lateral incisors 7-10 months (mandibular), and 8-11 months ( maxillary)
Chronology of Human Dentition of Primary (Deciduous)
and Secondary (Permanent) Teeth

CALCIFICATION AGE ATERUPTION AGE AT SHEDDING


Begins at Cemplete at Manillary Mandibular Maxillary Mandibular
PRIMARY TEETH
Central incisors 5th fetal mo 18-24 mo 6-8 mo 5-7 mo 7-8 yr 6-7 yr
Latetal imcisors 5th fetal mo 18-24 mo 8-11 mo F-10 mo 8-3 yr 7-8 Fr

Cuspids (canines) 6th fetal mo 30-36 mo 16-20 mo | 16-20 mo 11-12 yr 9-11 yr


First molars. 5th fetal mo 24-30 mo 10-16 mo | 10-16 mo 10-12 yr 10-12 yr
Second molars 6th fetal mo 36 mo 20-30 mo | 20-30 mo 10-12 yr 11-13 yr
SECONDARY TEETH
Central incisors 3-41 mo 3-10 yr 7-8 yr 6-7 yr
Latetal imcisors Max, 10-12 mol 10-11 yw 8-9 yr 7-8 yr
Mand, 3-4 mo
Cuspids (canines) 4-5 mo 12-15 yr 11-12 yr 9-11 yr
First premolars (bicuspids) 18-271 mo 12-13 yr 10-11 yr 10-12 yr
Second premolars (bicuspidsy 24-20 mo 12-14 yr 10-12 yr 11-13 yr
First molars. Birth 9-10 yr 6-7 yr 6-7 yr
Second molars 30-26 moa 14-16 yr 12-13 yr 12-13 yr
Third molars Max, 7-9 yr 18-25 yo 17-22 yr 17-22 yo
Mand, 5-10 yr
5.) Identify the lesion. Milia
Is it Physiologic or Pathologic? Physiologic

6.) During a physical examination to an adolescent you noted that there is breast bud with
elevation of breast and papilla.
a.) Identify the SMR. Tanner Stage 2

b.) In getting the Personal and Social History of this patient using HEADSSS FIRST, what is
letter E? Education
c.) In interviewing the adolescent, what principle/s must you remember? Confidentiality? Lol

rs pad eae

7.) Identify Structure A? Anterior fontanelle

B. When do you expect Structure B to close?


2-3 months after birth

8.) Aone year old boy with unrecalled vaccinations given developed fever. On his 3rd day of
illness, he was brought to the OPD. Upon oral examination, you saw:
Koplik’s spots

A. Identify the pointed lesion. Kopliks spot


B. This is pathognomonic of what disease? Measles

UF

9.) PP, 5 years old was admitted because of 5 days fever. On the 8th hospital day , fever
lysed and you noted the appearance of these skin changes in the legs.
A. What do you call the skin lesion? Hermans rash
B. In what disease is this rash seen? Dengue
C. In what phase of the disease it is typically seen? Recovery phase
10. 12 month old, male, came in the clinic for a well baby visit
A. Expected Developmental milestones
Motor: walks with one hand held; rises independently, takes several steps (Knobloch)
Adaptive: Picks up raisin with unassisted pincer m in ovement of forefinger and thumb, releases object to
other person on request or gesture
Language: Says a few words besides “mama,” “dada”
Social: Plays simple ball game, makes postural adjustment to dressing

B. Based on EPI, what vaccine should he receive?


MMR vaccine
C. What other vaccines will you recommend?
PCV vaccine booster( if compete First 3 doses of PCV 6 months prior)
- Varicella
-HepaA
-Influenza ( if not yet given)

Childhood Immunization Schedule 2019


WEEKS. MONTHS
4 16 18 20 10 12 «#14 «16

(DTwP - Hib
- Hep B*) and
other DTaP
combinations

IPV/OPV*

Pcv"

||

Measles
JE Vaccine

Varicella
Hep A

Uy Ce rey fee ee
BERREREEC Cr
a
Ce ea

DISCLAIMER:
The Childhood Immunization Schedule presents recommendations for immunization for children and adolescents based on updated literature review, experience
and premises current at the time of publication. The PPS, PIDSP and PFV acknowledge that individual circumstances may warrant a decision differing from the
recommendations given here. Physicians must regularly update their knowledge about specific vaccines and their use because information about safety and
efficacy of vaccines and recommendations relative to their administration continue to develop alter a vaccine is licensed.

ines i ional
The following vaccines are in the 2018 NIP:
* BCG, monovalent Hep B, Pentavalent vaccine (DTwP-Hib-HepB), bivalent OPV, IPV, PCV*, MMA, MR, Td, HPV", JE*

Recommended Vaccines
These are vaccines not included in the NIP which are recommended by the Philippines Pediatric Society (PPS), Pediatric Infectious Disease Society of the Philippines
(PIDSP) and the Philippine Foundation for Vaccination (PFV).
Question ]
In kerosene poisoning, which of the following statement is correct:
Correct

Marked out of
1.00

Y Flag
question o- it may cause convulsions

b. Inhalation is the most common route

©: There is a pharmacologic antidote

oe Pulmonary complication is unlikely

The correct answer is: It may cause convulsions


Question 2
Which of the following viral infections is transmitted airborne and makes it easily transmissible?
Incorrect

Marked out of
1.00

Y Flag
question o- Rubella

‘Mumps

©: varicella

d. Parvovirus 19

The correct answer is: Varicella


Question 3
Which of the following statements is NOT TRUE about macronutrients?
Incorrect

Marked out of
1.00

Y Flag
question a. Carbohydrates with low glycemic index increases the risk of type 2 diabetes.

D. The most important factor affecting protein requirement is its amino acid composition.

°- Fats provide the flavor and texture of foods.

* @: infants should get 30-40% of fat from the total dietary energy requirement.

The correct answer is:


Carbohydrates with low glycemic index increases the risk of type 2 diabetes.
Question 4
After being given 4 doses of a diuretic, a 5 year old girl cried, complained of cramps and weakness
Correct
of both lower extremities and was unable to walk. The most likely electrolyte imbalance in this girl
Marked out of would be:
1.00

Y Flag
question

* O Hy pokalemia

b ‘Hyponatremia

c. Hypomagnesemia

el Hypocalcemia

The correct answer is: Hypokalemia


Question 5 EBV infectious mononucleosis is not likely IF there is:
Correct

Marked out of a. Appearance of maculopapular rashes after amoxicillin is given


1.00
b. Increased WBC with lymphocytic predominance and reactive lymphocytes
Y Flag
question c. Splenomegaly

* d. Clinical improvement with amoxycillin

The correct answer is: Clinical improvement with amoxycillin


Question 6
A 3 year old girl has 2 days abdominal pain and high grade fever. She has had no vomiting or
Correct
diarrhea but there is terminal dysuria. On examination, there is lower abdomen tenderness, and
Marked out of costo-vertebral angle tenderness on the left. What would confirm the diagnosis?
1.00

VY Flag
question

9: Ultrasound of the Kidneys and urinary bladder with a post void bladder ultrasound

©. Urine culture and sensitivity

c. Urinalysis

d. Renal function test

The correct answer is: Urine culture and sensitivity


Question 7
Which of the following statements about beriberi is NOT TRUE?
Incorrect

Marked out of
1.00

Y Flag
question o. Peripheral neuritis is not seen in infants.

D- Affected infants may present with convulsion.

») ©. Consumption of highly polished rice or wheat flour is a predisposing factor.

d. CNS Involvement is the most common cause of death.

The correct answer is:


CNS Involvement is the most common cause of death.
Question 8
Coby 4 y/o had recurrent nasal congestion with purulent nasal discharge, periorbital edema and
Correct
hyposmia for 3 weeks. At his age, the following sinuses are involved:
Marked out of
1.00

Y Flag
question
* : Ethmoid sinus, Maxillary sinus v

b. Frontal sinus, Ethmoid sinus

c- Maxillary sinus, sohenoid sinus

qd. Ethmoid, sphenoid sinus

The correct answer is: Ethmoid sinus, Maxillary sinus


Question 9 . . ; F °
Ulcers at the uvulopalatoglossal junction among infants with roseola is known as
Correct

Marked out of
1.00

Y Flag
question * O Nagayama spots

b. Koplick spots

°- Forsheimer’s spot

a. Café au lait spot

The correct answer is:. Nagayama spots

4 Pravinitic nage EEE Neayt pane b>


Question 10
A 4-year old girl was assessed to have hypernatremic dehydration. In such instance, the first
Correct
priority is to__?
Marked out of
1.00

\Y Flag
question
9. Lower down hypernatremia

b. Restore intravascular volume

©. Treat underlying cause of hypernatremia

d. institute dialysis for removal of excess sodium

The correct answer is: Restore intravascular volume


Question 11
Which of the following statements about bronchiolitis is false?
Incorrect

Marked out of
1.00

Y Flag * ©: There is an increased risk of developing bronchial asthma after a bronchiolitis episode.
question

O. The most prominent PE finding is wheeze.

°- Treatment of choice is nebulization with a beta-2 agonist.

a. it is most commonly caused by RSV.

The correct answer is:


Treatment of choice is nebulization with a beta-2 agonist.
Question 12
Unusual growth in a pediatric patient is usually due to:
Incorrect

Marked out of
1.00

Y Flag
question °: Genetics

* B. endocrine disorders

©: Nutrition

d. Chronic diseases

The correct answer is: Genetics


Question 13
One should suspect chronic ITP if a patient presents with the following, EXCEPT:
Correct

Marked out of
1.00

Y Flag
question o- Female

b. Age <10 years old

c. Purpura 2-4 weeks before diagnosis

d. Higher platelet count

The correct answer is: Age <10 years old


Question 14
A 3-day old baby was seen at the OPD because of bilateral, mucopurulent eye discharge of 1 day.
Incorrect
The baby was term, delivered at home by a hilot with good cry and good suck. PE was normal
Marked out of except for bilateral conjunctival injection and moderate eye discharge. Gram stain of eye
1.00
discharge showed many pus cells, few gram (+) cocci and gram negative intracellular diplococci.
Y Flag Which is the most appropriate antibiotic regimen ?
question

°: Ceftriaxone 125 mg IV/IM, single dose

©. Ceftriaxone 125 mg IV/IM single dose plus oral erythromycin at 50 mg/k/ day x14 days

©: Oral erythromycin at 50 mg/k/day x 14 days

" o O5% erythromycin eye ointment x 7 days

The correct answer is: Ceftriaxone 125 mg IV/IM, single dose


The Philippine National Immunization program mandates that a 3 1/2 old infant receive which of the
following vaccines?

oO. opy

ib. Pentavalent vaccine

e All choices are correct

— d. py
Question 16
Which of the following drugs DOES NOT cause adrenal insufficiency?
Correct

Marked out of
1.00

VY Flag
question o- Ketoconazole

b. Phenobarbital

c. Phenytoin

* d. isoniazid

The correct answer is: Isoniazid


Question 17
In developing countries, which is the most common cause of acquired heart disease among
Correct
children?
Marked out of
1.00

Y Flag
question
o- infective endocarditis

b. Rheumatic fever - Rheumatic heart disease

°- Kawasaki disease

2h Myocarditis

The correct answer is: Rheumatic fever - Rheumatic heart disease


Question 18
Which is the cornerstone in the management of patients with abcess?
Incorrect

Marked out of
1.00

Y Flag
question a. Drainage of Abcess

b. Topical mupirocin

c. Antistaphyloccocal antibiotics

a. Wound dressing

The correct answer is: Drainage of Abcess


Question ] 9 Tony, a high school graduate who is planning to take up a nursing course consults you because of a
Incorrect laboratory exam with this result: HBsAg: reactive , anti HBS: non-reactive, anti HBc :reactive. His
Marked out of mother doesn’t remember if he was vaccinated.B What will you tell Tony?
1.00

Y Flag a. He has an infection that may be acute or chronic


question
b. He is susceptible to hepatitis B

c. He is immune to Hepatitis B from a previous infection.

e d. He has an acute infection. x

The correct answer is: He has an infection that may be acute or chronic

4 Previous page
Question 20
Which is the most common vascultic process associated with meningitis?
Incorrect

Marked out of
1.00

VY Flag 9: Venous infarction


question

b. Arterial infarction

©- Venous thrombophlebitis

d. Arteritis

The correct answer is: Venous thrombophlebitis


Question 21
This enzyme is important to avoid ambiguous genitalia in males and virilization in females due to
Incorrect
excessive amounts of dehydroepiandrosterone:
Marked out of
1.00

Y Flag
question
o. Aldosterone synthase

>. 3 beta hydroxysteroid dehydrogenase

* ©-17 qlpha hydroxylase

o.170H pregnenolone

The correct answer is:


3 beta hydroxysteroid dehydrogenase
Question 22 oye . . . . '
The probability of transmission among the children of thalassemia trait parents will be:
Correct

Marked out of
1.00

Y Flag
question 950% major

b. 25 % trait

* © 50% trait

d. 50 % normal

The correct answer is: 50 % trait


Question 23 . .
This is TRUE about Beta Thalassemia:
Correct

Marked out of
1.00

Y Flag
question °. Occur most commonly in people from southeast Asia and China

* b. Functional abnormality on the globin gene on chromosome 11

©: Inherited impairment of A chain synthesis

a Functional abnormality on the globin gene on chromosome 16

The correct answer is: . Functional abnormality on the globin gene on chromosome 11
Question 24 In patients with systemic onset JRA, what do you call the phenomenon that is characterized by
Incorrect cutaneous hypersensitivity evoked by heat or trauma, manifesting as salmon colored rashes, in the
Marked out of absence of fever?
1.00
¥ Flag * a. Raynauds phenomenon x
question
b. Papular purpuric gloves and socks syndrome

c. Koebner phenomenon

d. Extremity claudication

The correct answer is: Koebner phenomenon

4 Previous page Next page >


Question 25 A 2 year old child was brought to the clinic because of fever, sore throat and poor appetite for 3
Correct days. PE: vital signs were normal except for temperature of 38 0 C. There were scattered vesicles
Marked out of and ulcers on the buccal mucosa and posterior pharynx, maculopapular to vesicular rashes on the
1.00 hands and feet,and maculopapular rashes on the buttocks. A playmate apparently had the same
Flag illness 5 days earlier. Which is the most commonly implicated virus?
question
* a. Coxsackie virus A v

b. Herpes simplex virus 1

c.. Echovirus

d. Ebstein barr virus

The correct answer is: Coxsackie virus A


Question 26
Which of the following would need lifelong treatment?
Correct

Marked out of
1.00

Y Flag a. Absence
question

D- Fifth day fits

* © Janz syndrome

- Benign myoclonus of infancy

The correct answer is: Janz syndrome


Question 27 . . . . . .
Which primary type of nephrotic syndrome is the most responsive to steroid treatment?
Correct

Marked out of
1.00

Y Flag
question °- Membranoproliferative GN

* B. Minimal Change disease

© Focal Segmental GN

a. Mesengial GN

The correct answer is: Minimal Change disease


Question 28
Which enteric pathogen is associated with a cholera-like diarrhea?
Incorrect

Marked out of
1.00

Y Flag
question Se Salmonella

* EIEC

CE. histolytica

d. Rotavirus

The correct answer is: Rotavirus


Question 29
Which of the following congenital heart diseases are characterized by the presence of cyanosi, with
Correct
increased pulmonary vascular markings?
Marked out of
1.00

Y Flag
question

o- TOF, PVA

B-ps AS CoA

C- ASD, VSD, PDA

° Oo. 176A TAPVR

The correct answer is: TGA, TAPVR


Question 3 0
Gonadotropin-dependent Precocious Puberty is considered in the following situation:
Correct

Marked out of
1.00

Y Flag
question 7 10 year old boy with pubic hair.

D ang year old girl with menarche.

°c AG year old girl with body odor.

dn 4 year old boy with penile enlargement and small testes.

The correct answer is: An 8 year old girl with menarche.


Question 31
A 6 month old infant was brought to you for reflux with no other manifestations. The mother said her
Incorrect
child must be having GERD. What will you do?
Marked out of
1.00

VY Flag
question

* © Positioning measures

O. Observe the patient

“Do esophageal tests

d. Modify dietary regimen

The correct answer is: Observe the patient


Question 32
Dating during the middle adolescent period may cause conflict between parents and adolescents.
Incorrect
This conflict stems from:
Marked out of
1.00

Y Flag
question

o- What they do during the dating activity

O. Who they are with

c. Separation from family

* d. privacy issues

The correct answer is: Separation from family


Question 33
Which of the following staphylococcal diseases is NOT primarily toxin-mediated?
Correct

Marked out of
1.00

Y Flag
question Staphylococcal scalded skin syndrome

“TSST

©: Cavernous sinus thrombosis.

a Staphylococcal food poisoning

The correct answer is: Cavernous sinus thrombosis.


Question 34
In Staphylococcal Scalded skin Syndrome, the toxin is produced in the following clinical
Incorrect
environment EXCEPT:
Marked out of
1.00

Y Flag
question

a. Presence of Abcess

=) High pCO2

©. nerobic environment

* o. Neutral pH

The correct answer is: Presence of Abcess


Question 35
Joel is a 2-year old boy who suddenly had bouts of severe colicky abdominal pain with distention,
Correct
and bilious vomitus with bloody mucoid stools. History revealed he developed infectious parotitis 1
Marked out of week prior to consult. Which of the following is true of his condition?
1.00

Y Flag
question

* 9 The abdominal ultrasound may show “pseudo kidney” sign in longitudinal view.

O- The triad of right upper quadrant mass, jaundice and fever is a hallmark

c- Majority of cases have an identifiable etiology

a Radiologic reduction is always attempted first as this is a lesser invasive procedure

The correct answer is: The abdominal ultrasound may show “pseudo kidney” sign in longitudinal
view.
Question 36
This age corresponds with decreases in nutritional requirements and appetite leading to picky
Correct
eating habits:
Marked out of
1.00

\V Flag
question
9- 12 to 15 months

O 9 t05 yrs old

©-12 months

d. 4 to 6 months

The correct answer is: 2 to 5 yrs old


Question 37
Cyanosis in this congenital heart disease is due to the mixing of saturated and desaturated blood
Correct
in the right atrium;
Marked out of
1.00

Y Flag
question
o. Transposition of great arteries

“TAPVR v

c Tetralogy of Fallot

d. Truncus arteriosus

The correct answer is: TAPVR


Question 38
A 1y/o infant was brought to the clinic because of sudden onset of petechiae. There are no other
Correct
pertinent information in the history. On PE, she was playful, afebrile with generalized petechiae and
Marked out of no lymphadenopathies and hepatosplenomegaly. Your initial diagnosis is:
1.00

Y Flag
question

o- Acute ITP, post-vaccination

D Acute ITP, idiopathic

©: Chronic ITP

a. Acute ITP, post infectious

The correct answer is: Acute ITP, idiopathic


Question 3 9 A 16 year old male came in with high grade fever and monoarticular arthritis involving the right
Correct knee. Synovial fluid analysis revealed highly elevated PMN’s with growth of staphylococcus aureus in
Marked out of culture, what do you call this form of arthritis?
1.00

Y Flag * a. Septic arthritis v


question
b. Postinfectious arthritis

c. Reactive arththritis

d. Transient synovitis

The correct answer is: Septic arthritis


Question 40
Which heart murmur suggests the presence of aortic insufficiency?
Correct

Marked out of
1.00

Y Flag
question Q. Diastolic murmur at the apex

2 Holosystolic murmur at the left lower sternal border

c. Holosystolic murmur at the apex

* d. Diastolic murmur at the right upper sternal border

The correct answer is: Diastolic murmur at the right upper sternal border
Question 4] Which among the following is the most common vasculitis in childhood characterized by
Correct immunoglobulin A deposition in the small vessels of the skin, joint, GIT and kidneys?
Marked out of
1.00 a. Hypersensitivity angiitis
Fl
i * b. Henoch — schonlein; purpura Vv
question

c. Kawasaki disease

d. Polyarteritis nodosa

The correct answer is: Henoch —- schonlein purpura

4 Previous page Next page >


Question 42
Which of the following does NOT characterize cognition in the preschool age?
Correct

Marked out of
1.00

Y Flag
question * Abstract thinking

2 Thinking dominated by perception

c- Magical thinking

S Egocentrism

The correct answer is: Abstract thinking


Question 43
Which of the following is NOT a common indoor allergen?
Correct

Marked out of
1.00

Y Flag
question a. Cigarette smoke

©. Cockroach

©: Housedust mite

‘Molds

The correct answer is: Cigarette smoke


Question 44
The chest findings in a 7-year old female showed asymmetrical chest expansion (left lagging
Incorrect
compared to the right) & intercostal retractions. The right lung was resonant with good air entry &
Marked out of crackles all over. The left lung had decreased vocal & tactile fremiti, & decreased breath sounds.
1.00
Which of the following conditions should be suspected?
Y Flag
question

9- Pleural effusion

b. Pneumothorax

©. Consolidation

d. Atelectasis

The correct answer is: Pleural effusion


Question 45
A 2 year old female had two episodes of cystitis in the past 4 months. Her latest Urine culture
Correct
revealed E. coli > 100,000 CFU/ml. What would be the next appropriate investigation?
Marked out of
1.00

Y Flag
question
a. Request for a urinalysis

* VCUG

* © KYB ultrasound

a. Renal biopsy

The correct answer is: KUB ultrasound


Question 46
A 1-year old boy was brought to the OPD due to recurrent abdominal pain accompanied by
Incorrect
irregular bowel movement for the past 9 months. His care-giver usually notices that he has foul-
Marked out of smelling dark brown Bristol 7 stools. PE revealed a wt <-2 SD, distended abdomen with multiple
1.00
slightly tender hard masses on both hemiabdomen with hypoactive bowel sounds. Your most likely
Y Flag diagnosis is:
question

* O- Intestinal Neuronal Dysplasia *

b. Hirschprung disease

°- Functional Constipation

e Paralytic ileus

The correct answer is: Hirschprung disease


Question 47 This medication is recommended to all patients with SLE to treat mild clinical manifestations such
Correct as rash and arthritis and is proven to be effective in preventing SLE flares.
Marked out of
1.00 ¢ a. Hydroxychloroquine v
VY Flag
b. Methotrexate
question
c. Azathioprine

d. Cyclophosphamide

The correct answer is: Hydroxychloroquine


Question 48
A 6 year old boy sustained a wound while playing 16 days PTC. 12 days PTC, the wound got infected
Incorrect
but no medication was taken. 2 days PTC, the child was noted to have puffy eyelids with pedal
Marked out of edema and gross hematuria. The latent period for this patient is;
1.00

Y Flag
question

A190 days

ae days

c- 6 days

* 4-19 days

The correct answer is: 10 days


Question 49
At 1 minute of life, a 38 week neonate delivered by forceps extraction, exhibited the following: heart
Correct
rate 90 beats/min, slow irregular respiratory effort, limp, grimace on suctioning and acrocyanosis.
Marked out of What is his APGAR SCORE?
1.00

Y Flag
question

The correct answer is: 4


Question 50
A 4-year old boy was noted to have +2 to +3 proteinuria on playing and standing; however, when
Correct
rested and asleep, the urinalysis taken just after resting showed no protein. This proteinuria is
Marked out of called___?
1.00

Y Flag
question

SE Tubular

2 Pathologic

©: Postural or orthostatic

d. Glomerular

The correct answer is:


Postural or orthostatic
Tantrums peak in prevalence at this age:

~ O 6-19 months

ib. 12 months

* © oto 4yrs old

dd. 0-6 months


Question 52
Considered as a hallmark of Henoch-Schonlein Purpura ( HSP ) is the presence of:
Correct

Marked out of
1.00

Y Flag
question o- Renal involvement

2 Palpable purpura

©. musculoskeletal involvement

Be Neurologic manifestations

The correct answer is: Palpable purpura


Question 53
A 5yo child weighing 22 kg was brought to the emergency room because of seizure. On PE he was
Incorrect
noted to have generalized edema, his BP = 140/100, CR= 130’s / min, RR = 30's / min. His last urine
Marked out of output was 10 cc for the past 16 hours prior to consultation. Which of the following should NOT be
1.00
given to the patient?
Y Flag
question

o- Anti- hypertensive

>. Furosemide at I-2mg/kg

©: anti-convulsant

* Fy bolus of 440 cc/ kg

The correct answer is: Anti- hypertensive


Question 54
Syphilis occurring in a newborn that is congenitally transmitted is:
Incorrect

Marked out of
1.00

Y Flag
question a. Tertiary syphilis

2 Primary syphilis

c. Secondary syphyllis

* | atent syphilis

The correct answer is: Secondary syphyllis


Question 55 Which of the following is NOT true in pyloric stenosis?
Incorrect

Marked out of a. Usually seen at birth


1.00
lb. More concordant in monozygotic twins
Y Flag
question « c. Unusual in still births

d. In association with neonates receiving erythromycin first 2 weeks of life

The correct answer is: Usually seen at birth


Question 5 6
A 5-year old boy with generalized swelling was diagnosed to have nephrotic syndrome. His CBC
Incorrect
showed Hgb 15, Hct 45%, WBC 13,000/mm*: platelet count 500,000/mm%, urine protein to creatinine
Marked out of ratio was 5. Which of the following complications should he be monitored for?
1.00

Y Flag
question

* O: deritonitis

b. Pneumonia

©: Thromboembolism

* Sepsis

The correct answer is: Thromboembolism


Question 57
A 9 year old male has been passing tea colored urine for 4 days, associated with cough, colds and
Incorrect
low grade fever. PE and vital signs were normal. Urinalysis showed TNTC RBC with +1 proteinuria.
Marked out of The parents noted that this has happened before when the patient had an upper respiratory tract
1.00
infection.This history is consistent with;
Y Flag
question

a. Hemorrhagic cystitis

b. Glomerulonephritis.

© uti

& IgA nephropathy

The correct answer is: IgA nephropathy


Question 58
In patients with midline tumors, what is the most common visual deficit?
Correct

Marked out of
1.00

Y Flag
question a. Bitemporal hemianopsia

D. Homonymous hemisanopsia

©: Binasal hemianopsia

d. Blindness

The correct answer is: Bitemporal hemianopsia

a ae
Question 5 9 Which of the following is/are TRUE of Flu?
Correct

Marked out of * a. It has more systemic manifestations than other respiratory viruses
1.00
b. All the choices are correct
Y Flag
question c. Acute myositis is usually seen with Serotype A

d. Antivirals should be started within 72 hours from the onset of illness.

The correct answer is: It has more systemic manifestations than other respiratory viruses
Question 60
Which of the following statements is false?
Correct

Marked out of
1.00

Y Flag
question a. Pulmonary blood vessels accompany the airways.

b. No new alveoli are formed after birth.

°- The larynx demarcates the upper from the lower airway.

a. The pores of Kohn and canals of Lambert help prevent atelectasis.

The correct answer is: No new alveoli are formed after birth.
Question 61
In the pathogenesis of Juvenile Dermatomyositis ( JDM ), children with genetic susceptibility may
Incorrect
have prolonged exposure to:
Marked out of
1.00

Y Flag
question

a. Chemical substances

> Allergens

©- Infectious agents

d. Maternal chimeric cells

The correct answer is: Maternal chimeric cells


Question 62
Precautionary measures in the administration of an inactivated vaccine should be done if:
Correct

Marked out of
1.00

VY Flag
question
o With history of encephalopathy

2 Pregnant women

© With allergy to vaccine component.

d. With Severe Combined Immunodeficiency

The correct answer is:


Pregnant women
Question 63
A 7y/o child has high grade fever, body malaise and sore throat. On PE, there was marked tonsillar
Correct
enlargement, left upper quadrant abdominal tenderness, with multiple cervical and epitrochlear
Marked out of lymphadenopathy. Your primary consideration is:
1.00

VY Flag
question

°. Coxsackievirus pharyngitis

e Epstein-Barr virus Pharyngitis

c Streptococcal Pharyngitis

. Adenoviral Pharyngitis

The correct answer is: Epstein-Barr virus Pharyngitis


Question 64
Language is linked to both cognitive and emotional development. Delays can be suggestive of an
Correct
intellectual disability, autism spectrum disorder or the child has been maltreated:
Marked out of
1.00

Y Flag
question
Select one:

- Truew

False

The correct answer is ‘True’.


Question 6 5
Upon examination of a 12 year old female, you noticed that the patient’s breasts are beginning to
Correct
enlarge. Which will be your anticipatory advise regarding her growth and development?
Marked out of
1.00

VY Flag
question
* O Menarche may be expected within 2 years.

©. She would have achieved her peak height velocity.

c- Dating will be very much part of her social activity.

d. She can handle some basic adult roles.

The correct answer is: Menarche may be expected within 2 years.


Question 66
A 34 week 2100 grams neonate was born vaginally to a GIP] mother, with history of insulin
Incorrect
dependent diabetes. The neuromuscular and physical examination scoring at 1 hour of life is
Marked out of compatible with 34 weeks AOG. This neonate is;
1.00

Y Flag
question

o Term large for gestational age

> Preterm appropriate for gestational age

©: Preterm large for gestational age

a. Term appropriate for gestational age

The correct answer is: Preterm large for gestational age


Question 67
Bone marrow failure syndrome that is associated with pancytopenia and skeletal anomalies:
Correct

Marked out of
1.00

Y Flag
question 9: Diamond Blackfan Syndrome

b. Fanconi Anemia

c- Dyskeratosis Congenita

a. Shwachman-Diamond Syndrome

The correct answers are: Fanconi Anemia, Shwachman-Diamond Syndrome


Question 68 . . . . ye page
Which enteroadherent organism produces enterotoxin that enhances secretion while inhibiting
Correct F ;
sodium absorption?
Marked out of
1.00

VY Flag
question
CERES

D. EIEC

C- v. cholera

* ETEC

The correct answer is: V. cholera


Question 69
The following statement/s is/are TRUE about vitamin A:
Incorrect

Marked out of
1.00

Y Flag A: it is found in abundance in skim milk.


question

b. All choices are correct

& Its deficiency may predispose to bony defects.

a. 410 month boy with uncomplicated measles should be give 3 doses of this vitamin.

The correct answer is: Its deficiency may predispose to bony defects.
Question 70 . . ; ; ;
The following event(s) are expected to occur during pubic hair stage IV in males:
Correct

Marked out of
1.00
Y Flag
question * G- All choices are correct

b * Peak of height velocity

©. Peak of penile growth

a. Peak of testicular growth

The correct answer is: All choices are correct


Question 71
.A 12 year old boy was brought in because of enlarged, non-tender, bilateral anterior cervical lymph
Correct
nodes, observed for a month, accompanied by low grade fever, decreased appetite and weight
Marked out of loss. There was no improvement with 10 days of amoxicillin. Chest-ray showed interstitial
1.00
pneumonitis and tuberculin test was 12 mm induration. There was no previous treatment for TB.
Y Flag What is the prescribed treatment regimen for this case?
question

O- SHRZE +10 HR

D. 5 HRZES + 1 HRZE + 9 HRE

C- 9 HREZS + 1 HRZE + 5 HRE

° oO. ouRZE + 4HR

The correct answer is: 2HRZE + 4 HR


tion 72
auestion A 5 year old presented with a 5 day history of fever, calf tenderness, headache and jaundice. On
Correct
physical examination, there was conjunctival suffussion and periorbital edema. He had a history of
Marked out of wading through the flood 1 week ago. Which is your most likely diagnosis?
1.00

Y Flag
question

o. Chikungunya Fever

D Hepatitis B Infection

* © leptospirosis

d. Dengue Fever

The correct answer is: Leptospirosis


Question 73 Aly/o infant had nasal congestion, rhinorrhea, low grade fever, sneezing and dry cough with
Incorrect irritative hack followed by machine gun burst of uninterrupted cough on a single exhalation. Toby
Marked out of probably has:
1.00

VY Flag «a. Allergic rhinitis x


question
b. Acute sinusitis

c. Pertussis

d. Streptococcosis

The correct answer is: Pertussis


Question 74 A 4 year old boy was admitted because of 4 days on and off fever and anorexia,.. Patient was
Correct weak-looking, flushed with the following vital signs: T:37.4 C, BP: 90/60 mmHg, PR: 90/min. CRT: <2
Marked out of secs .Breath sounds were clear . Abdomen was soft, non-tender without organomegaly. Torniquet
1.00 test was positive. CBC showed Hgb: 13.5g/dl Hct: 0.42 WBC: 2.6, neu: 0.30 lymphs: 0.70 platelets 150
VY Flag This is most likely?
question
« a. Dengue without warning signs v

b. Severe Dengue

c. Systemic viral infection

d. Dengue with warning signs

The correct answer is: Dengue without warning signs


Question 75
Deficiency of this enzyme leads to excessive androstenedione causing hyperandrogenism:
Incorrect

Marked out of
1.00

Y Flag
question °. 21 beta hydroxylase

0-17 0H pregnenolone

c 17 alpha hydroxylase

d. Aldosterone synthase

The correct answer is: 21 beta hydroxylase


Question 76
At birth a term neonate was noted to have a hoarse cry, large tongue, prolonged jaundice,
Incorrect
markedly globular abdomen with umbilical hernia, poor tone and large fontanel. What is the most
Marked out of likely diagnosis?
1.00

Y Flag
question

) a. Congenital Adrenal hyperplasia

PKU

©. Galactosemia

a. Congenital Hypothyroidism

The correct answer is: Congenital Hypothyroidism


Question 77
A 5-year-old child had a low-moderate grade fever of 3 days. On the 5" day of illness, he
Incorrect
developed lace-like erythematous rashes on the arms and slapped-cheeks rashes. Your most likely
Marked out of diagnosis is:
1.00

VY Flag
question

o- Roseola

b. Measles

* © parvovirus 19 Infection

e Erythema infectiosum

The correct answer is: Erythema infectiosum


Question 78
A diarrheal disease caused by this organism puts infants <3 months old and those with
Incorrect
immunodeficiency at risk for bacteremia, osteomyelitis, meningitis and other extra-intestinal focal
Marked out of infections.
1.00

V¥ Flag
question

OF Enterotoxigenic E.coli

b. Non-typhoidal salmonella

c- Shigella

* &. Campylobacter jejuni *

The correct answer is: Non-typhoidal salmonella


Question 7 9
Which of the following patients is a candidate for passive immunization?
Incorrect

Marked out of
1.00

VY Flag
question “ON 2y/o0 boy with 3-doses of DPT who sustained laceration.

D. A 2500g preterm newborn.

°- A newborn whose mother has varicella at the time of delivery.

dp baby born to a mother anti-HBs (+).

The correct answer is:


A newborn whose mother has varicella at the time of delivery.
Question 8 0
A 4 year old had intermittent nasal obstruction, clear watery nasal discharge and frequent upward
Correct
rubbing of the nose. You noted conjunctival edema, clear nasal secretion with pale boggy bluish
Marked out of swollen turbinates. What should be given?
1.00

VY Flag
question

o- Fluticasone nasal spray

b. Oxymetazoline nasal spray

c- Chlorphenamine

* d. 1 oratadine 4

The correct answer is: Loratadine


Question 81
A 6y/o child had high grade fever, sore throat, vomiting, right tender cervical lymphadenopathy,
Incorrect
and red enlarged tonsils with exudates . At the ER the initial test you need to perform on this child is:
Marked out of
1.00

Y Flag
question
* © anti streptolysin O titer

2. Rapid antigen detection test

a Throat cultures

& Heterophile antibody

The correct answer is: Rapid antigen detection test


Question 82 was ; ; Pea :
Preschool year cognition corresponds to Piaget's preoperational stage which is characterized by
Incorrect ; : : :
egocentrism which is defined as:
Marked out of
1.00

Y Flag
question
oO. Inability of a child to take another’s point of view

* B. Child's sense of right and wrong

°- Unrealistic beliefs about the power of wishes

a. an understanding that objects continue to exist even when not seen

The correct answer is: Inability of a child to take another's point of view
oS oo er AT ee SA ae SCO eS eS ew SS

Question 83
The most common tumors in children aged less than 5 years old is located in which area?
Incorrect

Marked out of
1.00

Y Flag
question o- Midline

b. Hemispheral

c Supratentorial

d. Infratentorial

The correct answer is: Infratentorial


& S 2 wwss Su = oe SF Sar 27 Bw ww Swe SEES SON WS Sw ee eS

Question 84
Pubertal delay due to absence of androstenedione and, hypertension due to excessive aldosterone
Incorrect
are seen in patients with deficiency of this enzyme:
Marked out of
1.00

Y Flag
question
O17 alpha hydroxylase

b
* 17 OH pregnenolone

© 3 beta hydroxysteroid dehydrogenase

a. aldosterone synthase

The correct answer is: 17 alpha hydroxylase


Question 85
This test is both diagnostic and therapeutic for foreign body aspiration:
Correct

Marked out of
1.00

Y Flag
question * O. Bronchoscopy

>. Chest x-ray

©- laryngoscopy

“CBC

The correct answer is: Bronchoscopy

Previous page
Question 8 6 A 14 year old girl, victim of prostitution, was newly admitted to a shelter. PE revealed maculopapular
Correct rashes on her extremeties including palms and soles with wart-like plaques around her vagina.
Marked out of Her VDRL test was reactive at 1:32 .What is the most likely diagnosis ?
1.00

VY Flag a. Tertiary syphilis


question
b. Latent syphilis

c. Primary syphilis

« d. Secondary syphilis v

The correct answer is: Secondary syphilis


Question 87
The nontreponemal tests like VDRL or RPR are NOT used for:
Correct

Marked out of
1.00

V Flag
question 2) Establishing the diagnosis of syphilis

= Rapid and easy screening

c. Detecting re-infection

& Monitoring response to treatment

The correct answer is: Establishing the diagnosis of syphilis


Question 8 8
Which complication of Rheumatic fever-Rheumatic heart disease presents with cardiomegaly
Incorrect
involving the left ventricle?
Marked out of
1.00

\V Flag
question

a. mitral stenosis

P- portic regurgitation

©. portic stenosis

* o- Mitral regurgitation

The correct answer is: Aortic regurgitation


Question 8 9
A diagnosis of initial attack of Rheumatic fever is made in the presence of:
Incorrect

Marked out of
1.00

Y Flag
question * ©: arthritis, pharyngitis and high ASO titer

2 Erythema marginatum and arthralgia

c. Arthralgia, prolonged PR interval on ECG, high ASO titer

d. Chorea alone

The correct answer is: Chorea alone


Question 90
A week after insertion of a ventriculo-peritoneal shunt, a patient developed ventriculitis. What is the
Correct
most likely cause?
Marked out of
1.00

Y Flag
question
Se S. aureus

a epidermidis v

cos. saprophyticus

obs pneumonia

The correct answer is: S. epidermidis


Question 91
Which of the following is NOT TRUE of adolescent development?
Incorrect

Marked out of
1.00

Y Flag
question 9 Growth pattern is central to peripheral.

O. onset of puberty has been noted to occur earlier than in previous


generations.

* © Males increase in height around 2 years earlier than girls.

a. There is a significant earlier onset of breast development in girls.

The correct answer is: Growth pattern is central to peripheral.


Question 92
An infant, who presents with cyanosis later in the first year of life, dyspnea on exertion, and a
Incorrect
systolic murmur on the 2nd left parasternal space has which of the following?
Marked out of
1.00

Y Flag
question
o- TAPVR

“TVA

Gh

* TOF

The correct answer is: TOF


Question 93
Poliomyelitis classically will present as asymmetric flaccid paralysis or paresis without
Incorrect

Marked out of
1.00

VY Flag
question o- Residual paralysis

b. Sensory disturbances

©: Absent deep tendon reflexes

q- Residual paralysis

The correct answer is:


Sensory disturbances
Question 94
A 3 year-old developed high fever and seizures. Lumbar tap showed wbc of 1000/cu mm with
Incorrect
neutrophil predominance and Gram-stain showed Gram-positive cocci. The most likely cause is:
Marked out of
1.00

Y Flag
question
a. Pneumococcus

DN. meningitidis tetanus

©: Tetanus

d. S. aureus

The correct answer is: Pheumococcus


Question 95
An 8 y/o child was diagnosed with persistent allergic rhinitis. She was not compliant with her
Correct
maintenance medications. Which of the following diseases is a common complication that can
Marked out of occur in this condition?
1.00

Y Flag
question

o- Asthma

©. Snoring

c- Conjunctivitis

* d. Chronic sinusitis

The correct answer is: Chronic Sinusitis


Question 96
Which of the following statements is true of measles vaccination?
Incorrect

Marked out of
1.00

Y Flag o- it can be given to a 4-month old breastfed infant.


question

D- it requires 4 weeks interval before oral Polio vaccine can be given.

cit requires at least 95% immunization coverage to achieve herd immunity.

a. it can elicit solely humoral mediated response.

The correct answer is:


It requires at least 95% immunization coverage to achieve herd immunity.
Question 97
A 4-year old female had 5 days dry cough & low grade fever. After 3 days, fever disappeared but
Incorrect
the cough became productive. On examination, his RR was 28min. He had symmetrical chest
Marked out of expansion, no retraction, with rhonchi & occasional wheezes on both lung fields. What is the most
1.00
probable etiology?
Y Flag
question

a. Chlamydia pneumonia

2 Hemophilus influenzae

* © streptococcus pneumoniae

“RSV

The correct answer is: RSV


Question 9 8
A 3 y/o non asthmatic child with sore throat and nasal obstruction followed by clear watery nasal
Correct
discharge was seen at the OPD. On PE, you noted swollen, erythematous nasal turbinates and clear
Marked out of breath sounds. Your plan of management includes:
1.00

VY Flag
question

o: saline nasal spray, Anti-leukotriene

>. saline spray, Cetirizine

©- Loratadine, phenylephrine

+) di. Chlorphenamine Maleate, Phenylephrine

The correct answer is: Chlorphenamine Maleate, Phenylephrine


Question 99
A 1-year old child was brought in with 3 days fever, colds and brassy cough. On the day of consult,
Incorrect
rashes were noted on the face and trunk. On PE, the patient had conjunctivitis, with maculopapular
Marked out of rashes on the trunk and face. Whitish enanthem were also noted on the buccal mucosa. Patient did
1.00
not receive any immunization. What would be your impression?
Y Flag
question

* & Rubella

b
* Zika Infection

© Rubeola

d. Roseola

The correct answer is: Rubeola


Question
100 A 12 year old male, had cough & colds with whitish discharge of 3 days. On the day of consultation,
Incorrect the discharge became mucoid accompanied by nasal congestion and decreased appetite. On
examination, his RR was 32/min. He had symmetrical chest expansion, no retraction, & clear breath
Marked out of
1.00 sounds. What is the most probable diagnosis?
Y Flag
question

9- Bheumonia

D- acute tonsillopharyngitis

©: acute rhinosinusitis

* d. Bronchitis

The correct answer is: Acute rhinosinusitis


1.) At the 1st minute of life, a newborn was noted to have a Cardiac Rate of 140, with active
movement, grimace to stimulation, acrocyanosis and good respiration

A. What is the APGAR score? ​8


Cardiac Rate -140-2
Muscle tone-good-2
Reflex- Grimace- 1
Color-Acrocyanosis-1
Respiration-good-2

B. What is the normal APGAR score? ​7-10


C. What is the significance of APGAR score?
To assess for need of immediate resuscitation

2.) An irritable 3 y/o Male with history of ​vomiting and loose stools​ was seen at the OPD
with pertinent physical examination of: ​sunken eyeballs, dry lips, hyperactive bowel
sound​s and was noted to drink eagerly.

A. What is the level of dehydration? ​Some Dehydration


sunken eyeballs + dry lips + Drinks eagerly -> Moderate dehydration -> If with 2 or more signs
in B-> SOME DEHYDRATION
B. Give management (Drug/s and dosage) including home instruction. (see photos
below. Some dehydration-yellow and plan B)

a
P.S. binago ko lng ung picture coz may part na di mabasa kanina pero same siya sa pic :)
3.) Baby Girl A is on her 58th hour of life. You noted that the j​aundice was down to the
abdomen. ​Bilirubin Levels revealed Total Bilirubin 14.3 mg/dl,B1 12.2 mg/dl and B2 2.1
mg/dl.
Facts to consider​: Physiologic jaundice: appears after 24 hours -> so this case is
Physiologic jaundice​ Peaks: Day 2-3, Disappears 5th day of life
Jaundice is level of abdomen: so estimate is 15 mg/dl -> DECREASED
BIOTRANSFORMATION (CONJUGATION)

Under normal circumstances, the level of indirect bilirubin in umbilical cord


serum is 1-3 mg/dL and rises at a rate of <5 mg/dL/24 hr; thus, jaundice
becomes visible on the 2nd or 3rd day, usually peaking between the 2nd and 4th
days at 5-6 mg/dL and decreasing to <2 mg/dL between the 5th and 7th days
after birth.
In general, a
search to determine the cause of jaundice should be made if (1) it appears in the
1st 24-36 hr after birth, (2) serum bilirubin is rising at a rate faster than 5
mg/dL/24 hr, (3) serum bilirubin is >12 mg/dL in a full-term infant (especially in
the absence of risk factors) or 10-14 mg/dL in a preterm infant, (4) jaundice
persists after 10-14 days after birth, or (5) direct bilirubin fraction is >2 mg/dL at
any time.

a.Using the Bhutani chart at what zone will the bilirubin levels fall?
High-intermediate Risk zone ​ ( because baby is on the 58th hour of life with a total serum
bilirubin of 12)

B. Using Kramer’s Chart what will be the approximated bilirubin levels of this patient?

j​aundice was down to the abdomen -> 14 mg/dl​ ( I can’t find this sa Nelson’s )
C. What will be your plan of management?
-Firstly, phototherapy is indicated in High-intermediate risk zone.
-Explain that it resolves in 1 week in full term infants
-Interview patient to assess for need to do COOMB’s Test.
-Educate about breastfeeding and how it may ause jaundice

4.) Baby Zoe a 7 month old female came in the OPD for well baby check up.

What is her IBW? ​ 7 x 500 + 3000 grams = ​6500 grams


7-12 months: Age (in mos) x 500 + BW
*Average weight at birth: 3000 grams

What is the expected HC? 44.5 cm


At birth head circumference is 35 cm ( normal babies
- Should be monitored routinely during the first 3 years of life
o Especially in the first 2 year
Measured over the most prominent part of the occiput and just above the supraorbital ridges
Assuming Birth Head Circumference is 35 cm -> She is 7 months:
35 cm ( birth ) + ( 6 cm -> 1st 3 months has 2 cm for the first month) + ( 3 cm -> 2nd 3 months has 1 cm per month) +
( 0.5 cm -> for the 7th month) = 44.5 cm

Compute for the number of teeth/s expected for this age group?
7-months old: ​so 2 central incisors from mandibular area ( 5-7 months) + 2 central incisors
on maxillary area ( 6-8months) + 2 lateral incisors from mandibular area ( 7-10 months) = ​6
teeth-> It is expected for her to have 6 teeth

Cinco trans:
Eruption usually occurs at 6 months mandibular central Incisors
o You can wait until 15 months of age
- Eruption of permanent teeth begins at around 6 y/o and completed around 18 y/o
- Visit to the dentist could be done as early as the first tooth erupts

Nelsons:
Initial mineralization begins as early as the​ 2nd trimester ​(mean age for
central incisors, 14 wk) and ​continues through 3 yr of age for the primary
(deciduous) teeth and 25 yr of age for the secondary (permanent) teeth​.

Eruption begins with the central incisors and progresses laterally.


Nelsons:
Central incisors: 5-7 months ( mandibular), and 6-8 months ( maxillary).
Lateral incisors 7-10 months (mandibular), and 8-11 months ( maxillary)
5.) Identify the lesion. ​Milia
Is it Physiologic or Pathologic? ​Physiologic

6.) During a physical examination to an adolescent you noted that there is breast bud with
elevation of breast and papilla.
a.) Identify the SMR. T ​ anner Stage 2

b.) In getting the Personal and Social History of this patient using HEADSSS FIRST, what is
letter E? ​Education
c.) In interviewing the adolescent, what principle/s must you remember? ​Confidentiality? Lol

7.)
A. Identify Structure A? ​Anterior fontanelle
B. When do you expect Structure B to close?
2-3 months after birth

8.) A one year old boy with unrecalled vaccinations given developed fever. On his 3rd day
of illness, he was brought to the OPD. Upon oral examination, you saw:

A. Identify the pointed lesion. ​Kopliks spot

​ easles
B. This is pathognomonic of what disease? M
9.) PP, 5 years old was admitted because of 5 days fever. On the 8th hospital day , fever
lysed and you noted the appearance of these skin changes in the legs.

A. What do you call the skin lesion? ​Hermans rash


B. In what disease is this rash seen? ​Dengue
​ ecovery phase
C. In what phase of the disease it is typically seen? R
10. 12 month old, male, came in the clinic for a well baby visit
A. Expected Developmental milestones
Motor: walks with one hand held; rises independently, takes several steps (Knobloch)
Adaptive: Picks up raisin with unassisted pincer movement of forefinger and thumb, releases object to
other person on request or gesture
Language: Says a few words besides “mama,” “dada”
Social: Plays simple ball game, makes postural adjustment to dressing

B. Based on EPI, what vaccine should he receive?


MMR vaccine
C. What other vaccines will you recommend?
PCV vaccine booster ( if compete First 3 doses of PCV 6 months prior)
- Varicella
-Hepa A
-Influenza ( if not yet given)
10/27/2020 Pediatrics 3B 2nd Prelim Exam: Attempt review

Dashboard My courses Pedia 3B 1st Sem SY 2020 MD3-I,J,K,L MIDTERM EXAM

Pediatrics 3B 2nd Prelim Exam

Started on Friday, 23 October 2020, 11:20 AM


State Finished
Completed on Friday, 23 October 2020, 1:29 PM
Time taken 2 hours 9 mins
Grade 59.00 out of 100.00

Question 1

Incorrect

Mark 0.00 out of 1.00

A 10 y/o girl experienced rapid weight gain from 23kg to 36kg, with an increase in height of only 3.5 cm in 2 years.
She had moon face and acne; experienced depression and poor school performance. Ht 122 cm, bone age 7 y/o.
Cortisol level increased in the morning and was suppressed by high dose dexamethasone. In the absence of
exogenous intake of steroids, what is the next most possible etiologic cause of this disorder?

a. Pituitary adenoma

b. Pituitary gland hyperplasia

c. Adrenal adenoma 

d. Adrenal tumor

The correct answer is: Pituitary adenoma

moodle.feu-nrmf.edu.ph/mod/quiz/review.php?attempt=419239&cmid=32627&showall=1 1/42
10/27/2020 Pediatrics 3B 2nd Prelim Exam: Attempt review

Question 2
Correct

Mark 1.00 out of 1.00

A 10 y/o girl experienced rapid weight gain from 23kg to 36kg, with an increase in height of only 3.5 cm in 2 years.
She had moon face and acne; experienced depression and poor school performance. Ht 122 cm, bone age 7 y/o.
Cortisol level increased in the morning and was suppressed by high dose dexamethasone. Select the treatment
of choice:

a. Transsphenoidal pituitary surgery 

b. Pasireotide

c. Cyproheptadine

d. Adrenalectomy

The correct answer is: Transsphenoidal pituitary surgery

Question 3

Incorrect

Mark 0.00 out of 1.00

A 12 y/o male is admitted due to repeated history of fall with difficulty of ambulation and climbing the stairs,
weakness and fatigue. An uncle died of the same illness at a young age. His IQ is normal. PE: obese, positive
Gower’s sign, (+1 to 2) motor strength. Creatine kinase is high. EMG suggests primary muscle disease. What is the
etiologic cause of the endocrine disorder?

a. deletion of XP21 contiguous gene

b. ABCD1 mutation 

c. disruption of SF-1

d. mutation of GL13 oncogene

The correct answer is: deletion of XP21 contiguous gene

moodle.feu-nrmf.edu.ph/mod/quiz/review.php?attempt=419239&cmid=32627&showall=1 2/42
10/27/2020 Pediatrics 3B 2nd Prelim Exam: Attempt review

Question 4
Incorrect

Mark 0.00 out of 1.00

A 13-month-old baby was seen because of high fever, poor oral intake and salivation. PE showed multiple vesico-
ulcers on the tonsils, tonsillar pillars, uvula and soft palate. The remainder of the throat looked normal. Which is
the most likely etiologic agent?

a. Adenovirus

b. Ebstein barr virus

c. Herpes simplex virus 1 

d. Coxsackie A

The correct answer is: Coxsackie A

Question 5

Incorrect

Mark 0.00 out of 1.00

A 13y/o female was at the Teen Center for catch up immunization. PE- wt and ht at the 50th percentile, breast and
papilla elevated as small mound, pubic hair sparse and lightly pigmented at the medial border. Menarche at 12
y/o. Which of the statements is CORRECT regarding the patient’s pubertal changes?

a. Her height and weight are normal, but menarche is early.

b. Her SMR is 2, height and weight normal and age of menarche normal at age 12 years.

c. Her SMR is 2 and menarche is early.

d. Her SMR is 2 but height and weight are abnormal. 

The correct answer is: Her SMR is 2, height and weight normal and age of menarche normal at age 12 years.

moodle.feu-nrmf.edu.ph/mod/quiz/review.php?attempt=419239&cmid=32627&showall=1 3/42
10/27/2020 Pediatrics 3B 2nd Prelim Exam: Attempt review

Question 6
Correct

Mark 1.00 out of 1.00

A 14 y/o male was brought to the Teen Center because he had no clinical signs of puberty. The subspecialty
resident was discussing the contemplated work up for hypogonadotropic hypogonadism with the patient and his
parents. Which of the following statements must be emphasized?

a. Testicular volume of less than 4ml by 12 years of age occurs in 13% of males.

b. Constitutional delay of puberty should be ruled out before a diagnosis is made and treatment is 
initiated.

c. Treatment of testosterone will not stimulate testicular growth and spermatogenesis.

d. A brief course of testosterone is recommended.

The correct answer is: Constitutional delay of puberty should be ruled out before a diagnosis is made and
treatment is initiated.

Question 7

Incorrect

Mark 0.00 out of 1.00

A 15 y/o male with a history of hypothyroidism was brought to the ER due to constipation, anorexia, weight loss,
abdominal pain of 1 month. At the ER, his BP suddenly dropped, given fluid resuscitation. CBG and Na were
decreased, K was increased. At the PICU, he had markedly elevated ACTH, undetectable cortisol level, increased
TSH, normal free T4 and aldosterone, and increased 21-OH antibodies. What is the diagnosis?

a. Isolated autoimmune disease

b. APS II

c. APECED I

d. Addison Disease 

The correct answer is: APS II

moodle.feu-nrmf.edu.ph/mod/quiz/review.php?attempt=419239&cmid=32627&showall=1 4/42
10/27/2020 Pediatrics 3B 2nd Prelim Exam: Attempt review

Question 8
Incorrect

Mark 0.00 out of 1.00

A 15-year-old female presented with short stature, widely-spaced nipples, and Tanner stage 1. Diagnostic work-
up should include:

a. IGF-1

b. FT3, TSH

c. All choices are correct 

d. Karyotyping

The correct answer is: Karyotyping

Question 9

Correct

Mark 1.00 out of 1.00

A 16-year-old girl came in with low-grade fever, tender cervical or post-auricular lymphadenopathy and
maculopapular rash which lasted for 3 days. There was no history of vaccination after 1 year of age. Which
among the following is the most likely etiologic agent?

a. Rubella 

b. Roseola

c. Erythema Infectiosum

d. Rubeola

The correct answer is: Rubella

moodle.feu-nrmf.edu.ph/mod/quiz/review.php?attempt=419239&cmid=32627&showall=1 5/42
10/27/2020 Pediatrics 3B 2nd Prelim Exam: Attempt review

Question 10
Incorrect

Mark 0.00 out of 1.00

A 17 y/o male who presented with eunuchoid body proportions, absence of facial, axillary and pubic hair,
micropenis and surgically corrected cryptorchidism. There was associated hyposmia. Karyotype was 46XY. Select
the first imaging step to perform:

a. MRI of adrenals

b. adrenal USG 

c. cranial MRI

d. cranial CT Scan

The correct answer is: cranial MRI

Question 11

Correct

Mark 1.00 out of 1.00

A 2-day old male had several bouts of vomiting but no fever, diarrhea since his hospital discharge. NBS showed
(++) 17-hydroxyprogesterone. Maternal history was unremarkable. PE: wt 2.8 kg, lethargic but responds to tactile
stimulation, sunken anterior fontanelle, CRT 4 sec; normal genitalia without signs of virilization. Choose the best
diagnostic panel:

a. CBC, 21-hyxylase antibody testing, serum Na, K, CBG

b. CBC, urinalysis, LP, serum Na, K, Cl, CBG, serum 17-hydroxyprogesterone, ABG 

c. Cranial CT-scan, CBC, urinalysis, chest X-ray serum Na, K, Ca, CBG

d. CBC, urinalysis, Chest X-ray, serum Na, K, Ca, Mg, Cl, CBG, serum 21-hydroxylase

The correct answer is: CBC, urinalysis, LP, serum Na, K, Cl, CBG, serum 17-hydroxyprogesterone, ABG

moodle.feu-nrmf.edu.ph/mod/quiz/review.php?attempt=419239&cmid=32627&showall=1 6/42
10/27/2020 Pediatrics 3B 2nd Prelim Exam: Attempt review

Question 12
Incorrect

Mark 0.00 out of 1.00

A 2-day old male had several bouts of vomiting but no fever, diarrhea since his hospital discharge. NBS showed
(++) 17-hydroxyprogesterone. Maternal history was unremarkable. PE: wt 2.8 kg, lethargic but responds to tactile
stimulation, sunken anterior fontanelle, CRT 4 sec; normal genitalia without signs of virilization. Select the most
appropriate initial therapy:

a. Give rapid bolus of D5NSS and may repeat twice.

b. Give maintenance D5 NSS and start IV hydrocortisone and kayexalate.

c. Give maintenance D5NSS and start IV hydrocortisone and fludrocortisone. 

d. Give a rapid bolus of plain NSS followed by maintenance fluid D5NSS and start IV hydrocortisone.

The correct answer is: Give a rapid bolus of plain NSS followed by maintenance fluid D5NSS and start IV
hydrocortisone.

Question 13

Incorrect

Mark 0.00 out of 1.00

A 2-month old baby presents with cleft palate, brief, recurrent seizures. Serum calcium is low. There is a soft
systolic murmur at the left sternal edge. Chest X-ray shows a narrow superior mediastinum. Select the most
appropriate laboratory test to arrive at the most likely consideration:

a. Parathormone and Vitamin D assay 

b. Determination of antiparathroid antibodies

c. FISH analysis

d. Karyotyping

The correct answer is: FISH analysis

moodle.feu-nrmf.edu.ph/mod/quiz/review.php?attempt=419239&cmid=32627&showall=1 7/42
10/27/2020 Pediatrics 3B 2nd Prelim Exam: Attempt review

Question 14
Correct

Mark 1.00 out of 1.00

A 6-month-old child came in with moderate grade fever and 5x vomiting. Later the patient developed watery,
non-bloody diarrhea. The patient was given 3 doses of DPT, HiB and OPV. This child’s gastroenteritis is most likely
caused by:

a. Measles

b. Rotavirus 

c. Norovirus

d. ETEC

The correct answer is: Rotavirus

Question 15

Incorrect

Mark 0.00 out of 1.00

A mother is concerned about the strong body odor of her 9-year-old heterozygous twins which she noted a year
earlier. She consults their pediatrician who explains this/these CORRECT statement/s:

a. Premature adrenarche coincides with increased levels of androgens before the age of 8 years in girls and 9
years in boys resulting to adult-type body odor, oily skin and hair and pubic hair growth.

b. Precocious puberty manifests with the development of secondary sexual characteristics including 
testicular growth and breast development.

c. All choices are correct.

d. The onset of puberty in girls is defined by thelarche and in boys by gonadarche.

The correct answer is: All choices are correct.

moodle.feu-nrmf.edu.ph/mod/quiz/review.php?attempt=419239&cmid=32627&showall=1 8/42
10/27/2020 Pediatrics 3B 2nd Prelim Exam: Attempt review

Question 16
Incorrect

Mark 0.00 out of 1.00

A newborn baby G has ambiguous genitalia, vomiting, dehydration with episodes of hypotension. Serum cortisol,
Na, androstenedione, testosterone and estradiol are low. ACTH, DHEA and renin are high. Select the most
appropriate treatment regimen:

a. Hydrocortisone, fludrocortisone, sex hormones

b. Hydrocortisone, fludrocortisone, NaCl supplementation, surgical correction of genitals at 2 months of age,


sex hormones

c. Hydrocortisone, fludrocortisone, NaCl for 6 months, genital repair at 6 months old 

d. Suppression with glucocorticoids and orchidopexy

The correct answer is: Hydrocortisone, fludrocortisone, NaCl supplementation, surgical correction of genitals at 2
months of age, sex hormones

Question 17

Correct

Mark 1.00 out of 1.00

A newborn was delivered FT, pink, with good cry and reflexes, but the external genitalia looks “odd”, described as
micropenis with empty scrotum. The rest of PE are normal. Select the next best step to undertake:

a. Consider doing CT-scan of adrenal glands.

b. Cortisol determination before and after administration of ACTH.

c. Perform NBS. 

d. Request for serum 17-OH progesterone.

The correct answer is: Perform NBS.

moodle.feu-nrmf.edu.ph/mod/quiz/review.php?attempt=419239&cmid=32627&showall=1 9/42
10/27/2020 Pediatrics 3B 2nd Prelim Exam: Attempt review

Question 18
Correct

Mark 1.00 out of 1.00

A patient with a bleeding dog bite was brought in. She had no history of rabies vaccine previously. The following
should be given to this patient:

a. Rabies Immunoglobulin

b. No need for Rabies Vaccine nor Immunoglobulin

c. Rabies Vaccine and Immunoglobulin 

d. Rabies Vaccine

The correct answer is: Rabies Vaccine and Immunoglobulin

Question 19

Correct

Mark 1.00 out of 1.00

A patient with short stature presented with growth velocity of > 5 cm per year, normal bone age and projected
height within mid-parental height. Secondary sexual characteristics at par with age. You should consider:

a. constitutional growth delay

b. idiopathic short stature

c. growth hormone deficiency

d. familial short stature 

The correct answer is: familial short stature

Question 20
Correct

Mark 1.00 out of 1.00

A preschooler, a known asthmatic is having an acute exacerbation. The most common viral trigger of asthma in
children:

a. Coronavirus

b. Rhinovirus 

c. Influenza

d. Respiratory syncitial virus

The correct answer is: Rhinovirus

moodle.feu-nrmf.edu.ph/mod/quiz/review.php?attempt=419239&cmid=32627&showall=1 10/42
10/27/2020 Pediatrics 3B 2nd Prelim Exam: Attempt review

Question 21
Incorrect

Mark 0.00 out of 1.00

A sick baby has a palpable gonad, no uterus, no skin pigmentation and no dysmorphic features. Karyotype 46,XY,
normal 17-hydroxyprogesterone, gonadal biopsy-normal testis, decreased testosterone response to hCG. What is
the most likely diagnosis?

a. Partial androgen insensitivity

b. Block in testosterone synthesis

c. Gonadal dysgenesis with Y chromosome 

d. Sex chromosome DSD

The correct answer is: Block in testosterone synthesis

Question 22

Correct

Mark 1.00 out of 1.00

About 90% of CMV congenitally infected babies are asymptomatic at birth but should be monitored for
___________, which is considered as a major sequela:

a. sensorineural deafness 

b. motor deficits

c. mental retardation

d. blindness

The correct answer is: sensorineural deafness

moodle.feu-nrmf.edu.ph/mod/quiz/review.php?attempt=419239&cmid=32627&showall=1 11/42
10/27/2020 Pediatrics 3B 2nd Prelim Exam: Attempt review

Question 23
Correct

Mark 1.00 out of 1.00

An 11-month old child was brought in due to difficulty of breathing. 3 days PTC, he had moderate fever
accompanied by cough. Few hours PTC, he was noted to have hoarseness with difficulty of breathing. Pertinent PE
showed subcostal and intercostal retractions, crackles and wheezes on all lung fields. Chest x-ray showed
hyperaeration of both lungs with minimal infiltrates. The most likely impression is:

a. Bronchiolitis 

b. Acute Bronchitis

c. Pneumonia

d. Tracheitis

The correct answer is: Bronchiolitis

Question 24

Incorrect

Mark 0.00 out of 1.00

An advanced bone age is NOT expected in this patient:

a. 15-year old female with symmetrically enlarged thyroid and hypertension

b. 14-year old male, with an upward slant to the eyes, large tongue, low set ears and a single, deep crease in
the middle of palm.

c. 8-year old female diagnosed with familial tall stature 

d. 9-year old male, height +3 SD, BMI between +2 and +3, tanner stage 2

The correct answer is: 14-year old male, with an upward slant to the eyes, large tongue, low set ears and a single,
deep crease in the middle of palm.

moodle.feu-nrmf.edu.ph/mod/quiz/review.php?attempt=419239&cmid=32627&showall=1 12/42
10/27/2020 Pediatrics 3B 2nd Prelim Exam: Attempt review

Question 25
Correct

Mark 1.00 out of 1.00

An eight-month-old boy was brought in due to fever of 3 days and rash a few hours PTC. The patient also
complained of cough and colds 2 days PTC. On PE, he has fever, conjunctival injection and maculopapular rashes
on the face. Whitish lesions were noted on the buccal mucosa. What is the most likely diagnosis?

a. Rubella

b. Measles 

c. Roseola Infantum

d. Fifth Disease

The correct answer is: Measles

Question 26

Correct

Mark 1.00 out of 1.00

An illness presenting with fever, rashes malaise, headache, conjunctivitis, and/or muscle/joint pains is NOT
characteristic of:

a. Japanese encephalitis 

b. Chikungunya

c. Zika

d. Dengue

The correct answer is: Japanese encephalitis

moodle.feu-nrmf.edu.ph/mod/quiz/review.php?attempt=419239&cmid=32627&showall=1 13/42
10/27/2020 Pediatrics 3B 2nd Prelim Exam: Attempt review

Question 27
Incorrect

Mark 0.00 out of 1.00

Angel, 10-years-old, came in for immunization update. She is asymptomatic with normal vital signs and
anthropometric measurements for age. Non-fasting non-HDL cholesterol was 146 mg/dL. Your next step in the
management is to do:

a. FBS or OGTT

b. fasting non-HDL cholesterol determination 

c. fasting lipid profile

d. update immunization and repeat test after 1 year

The correct answer is: fasting lipid profile

Question 28

Incorrect

Mark 0.00 out of 1.00

At the Oncology ward, an adolescent female has adrenocortical carcinoma receiving oral mitotane. Another
patient is for surgery and is to receive etomidate for general anesthesia. Another cancer patient receives
ketoconazole because of moderate oral thrush and chronic onychomycosis. All three drugs may cause adrenal
insufficiency because of _________:

a. inhibition of steroidogenesis

b. cytotoxicity 

c. no choice is correct

d. increased excretin of steroid metabolites

The correct answer is: inhibition of steroidogenesis

moodle.feu-nrmf.edu.ph/mod/quiz/review.php?attempt=419239&cmid=32627&showall=1 14/42
10/27/2020 Pediatrics 3B 2nd Prelim Exam: Attempt review

Question 29
Incorrect

Mark 0.00 out of 1.00

BB, 9y/o was brought to the OPD because of thickened nails of 6 months treated with antifungal ointment. 8 of the
toenails and 4 of the fingernails were consistent of chronic fungal infection. Rest of PE was normal. He had
intermittent leg pains and cramps regardless of activities relieved by rest and massage. The signs and
symptoms are suggestive of:

a. Chronic mucocutaneous candidiasis and Addison disease 

b. Chronic mucocutaneous fungal infection and hypoparathyroidism

c. Systemic fungal infection

d. Systemic candidiasis with adrenal insufficiency

The correct answer is: Chronic mucocutaneous fungal infection and hypoparathyroidism

Question 30

Incorrect

Mark 0.00 out of 1.00

Bong, 2 years old, came in for his annual flu shot. He was asymptomatic. Weight and length were between -2 and
-3 SD below the mean for age and sex. PE showed open, flat, 1x1 cm anterior fontanel with frontal bossing, high-
pitched voice, small hands, feet and genitalia, and undescended testes. He had a history of apnea during the
neonatal period. This is TRUE about his condition:

a. Skeletal age would be equivalent to a 2-year old.

b. Prolonged jaundice may have been present during infancy.

c. Height age would be equivalent to a 2 year old.

d. A delay in his neurodevelopmental milestones and mental development is expected. 

The correct answer is: Prolonged jaundice may have been present during infancy.

moodle.feu-nrmf.edu.ph/mod/quiz/review.php?attempt=419239&cmid=32627&showall=1 15/42
10/27/2020 Pediatrics 3B 2nd Prelim Exam: Attempt review

Question 31
Correct

Mark 1.00 out of 1.00

Cesar, 15-years-old, was diagnosed with central diabetes insipidus. This/These statement/s is/are TRUE in the
management:

a. Both oral and intranasal desmopressin have the same onset of action provided that the dose of oral
desmopressin is 10x that of the intranasal form.

b. Patient preference and desired length of anti-diuresis are factors that determine the appropriate dose 
and route of administration of desmopressin.

c. While on desmopressin, patients are advised to drink fluids every 1-2 hours during the day and 3x at night.

d. All choices are correct

The correct answer is: Patient preference and desired length of anti-diuresis are factors that determine the
appropriate dose and route of administration of desmopressin.

Question 32

Correct

Mark 1.00 out of 1.00

Children afflicted with this clinical disorder have no symptoms, the diagnosis being established after incidental
discovery of hypertension; others have concomitant headache, visual disturbances, muscle weakness and
intermittent paralysis which are primarily due to_________:

a. all choices are correct

b. hypokalemia 

c. hypocalcemia

d. hypomagnesemia

The correct answer is: hypokalemia

moodle.feu-nrmf.edu.ph/mod/quiz/review.php?attempt=419239&cmid=32627&showall=1 16/42
10/27/2020 Pediatrics 3B 2nd Prelim Exam: Attempt review

Question 33
Correct

Mark 1.00 out of 1.00

Claire, 14-years-old, was referred because of anemia despite intake of hematinics for 2 years. Upon history taking,
she also complained of constipation, cold intolerance, and sluggishness for the past 4 years. This is TRUE
regarding her case:

a. TSH and FT4 monitoring should be done every 3 months.

b. TSH should be monitored 8-12 weeks after changing the dosage

c. Earlier remissions are expected among males and those with higher BMI.

d. If euthyroid, L-thyroxine is not indicated, while monitoring of thyroid function tests should continue. 

The correct answer is: If euthyroid, L-thyroxine is not indicated, while monitoring of thyroid function tests should
continue.

Question 34

Incorrect

Mark 0.00 out of 1.00

Clyde, 10-years-old, male with a height of 125 cm ( z-score = -2 SD ) and a projected height of 162 cm. The mid-
parental height is 174 cm. The following is/are TRUE about this case:

a. All the options are correct 

b. Idiopathic short stature is a strong consideration.

c. He has short stature.

d. He is growing according to his genetic potential.

The correct answer is: He has short stature.

moodle.feu-nrmf.edu.ph/mod/quiz/review.php?attempt=419239&cmid=32627&showall=1 17/42
10/27/2020 Pediatrics 3B 2nd Prelim Exam: Attempt review

Question 35
Correct

Mark 1.00 out of 1.00

Dengue and chikungunya are both transmitted by Aedes mosquitoes and may present similarly. What
complication is associated with chikunguya?

a. encephalitis

b. shock

c. chronic arthritis 

d. bleeding

The correct answer is: chronic arthritis

Question 36

Incorrect

Mark 0.00 out of 1.00

Dianne, 6-years-old, came in for well child consult at the OPD. PE showed low hairline, webbed neck and strong
pulses in the upper extremities and lower extremities are weak. Her height age was that of 2 years old. Mother’s
height is 150 cm, father stands at 155 cm. This would be TRUE of her case:

a. sexual maturation is expected to be delayed

b. Mid-parental height would be attained after adequate treatment with growth hormone and LH agonist

c. Chromosomal analysis would reveal 46XO.

d. skeletal age may equivalent to that of a 2 year old 

The correct answer is: sexual maturation is expected to be delayed

Question 37
Correct

Mark 1.00 out of 1.00

High dose desmopressin may be combined with this drug in the management of congenital x-linked
nephrogenic diabetes insipidus:

a. indomethacin 

b. chlorthiazide

c. amiloride

d. pitressin

The correct answer is: indomethacin

moodle.feu-nrmf.edu.ph/mod/quiz/review.php?attempt=419239&cmid=32627&showall=1 18/42
10/27/2020 Pediatrics 3B 2nd Prelim Exam: Attempt review

Question 38
Correct

Mark 1.00 out of 1.00

Impaired glucose tolerance is defined as:

a. HgbA1C is 5.7-6.4%

b. FBS 100-125 mg/dL

c. FBS greater than or equal to 126 mg/dL

d. 2 hour OGTT is greater than or equal to 140 mg/dL but less than 200 mg/dL 

The correct answer is: 2 hour OGTT is greater than or equal to 140 mg/dL but less than 200 mg/dL

Question 39

Correct

Mark 1.00 out of 1.00

In Dengue infection, a decreasing hematocrit together with stable vital signs and good urine output indicate:

a. dehydration

b. reabsorption 

c. plasma leakage

d. bleeding

The correct answer is: reabsorption

Question 40
Incorrect

Mark 0.00 out of 1.00

In the management of Graves’ disease, the following statement/s is/are true:

a. During times of significant fever, pharyngitis and oral ulcers patients on methimazole should have their
WBC level determined while continuing the intake of the medication.

b. Treatment with 131I can be used in children 2 years old and above.

c. Radioactive iodine is the most cost-effective treatment.

d. Methimazole should be discontinued 7 days prior to doing radioiodine ablation with 131I. 

The correct answer is: Radioactive iodine is the most cost-effective treatment.

moodle.feu-nrmf.edu.ph/mod/quiz/review.php?attempt=419239&cmid=32627&showall=1 19/42
10/27/2020 Pediatrics 3B 2nd Prelim Exam: Attempt review

Question 41
Correct

Mark 1.00 out of 1.00

Jack, 3 years old, slipped and fell down a flight of stairs 3 days PTC. He was unconscious for a minute, had
headache and dizziness when he regained consciousness. There was note of contusion hematoma on this right
fronto-parietal area. Few hours PTC, there was headache, irritable and combative. BP 90/60, CR 75 cpm, RR 26
bpm. You noted a dry diaper that was last changed 15 hours prior. The main pathophysiology in this case
involves:

a. sodium 

b. potassium

c. calcium

d. phosphorus

The correct answer is: sodium

Question 42

Correct

Mark 1.00 out of 1.00

Jason, 9 years old, known case of T1DM, is scheduled to compete in the morning heats of the 800m and 1500m
events in an interschool athletics meet. The following statement/s is/are true:

a. All choices are correct. 

b. He should inject his insulin dose subcutaneously in his abdomen.

c. He should eat an open-faced whole wheat sandwich before the events

d. He should halve his total dose of insulin for the day.

The correct answer is: All choices are correct.

moodle.feu-nrmf.edu.ph/mod/quiz/review.php?attempt=419239&cmid=32627&showall=1 20/42
10/27/2020 Pediatrics 3B 2nd Prelim Exam: Attempt review

Question 43
Incorrect

Mark 0.00 out of 1.00

Jeb, 7 y/o, known case of T1DM, came in due to fever and difficulty in breathing of 1 day. PE showed BP 100/80, CR
130 bpm, RR 42 cpm, Temp 38.5C, weight =17kg. He had deep, rapid breathing, crackles both lung bases and a
fruity breath odor. ABG showed pH 7.14, pO2 98, pCO2 30, HCO3 9, O2 sat 96. The following statement/s is/are TRUE:

a. Give 340 cc of plain NSS as fast drip

b. all the options are correct 

c. Start insulin drip at 0.85 units/hr

d. Give 170 cc of plain LR to run for 1 hour.

The correct answer is: Give 340 cc of plain NSS as fast drip

Question 44

Correct

Mark 1.00 out of 1.00

JIM 12-year-old had malaise, myalgia & headache followed by fever for 3 days. This was accompanied by
pleuritic chest pain. This illness is most commonly associated with:

a. Coxsackie A

b. Numbered Enterovirus

c. Echovirus

d. Coxsackie B 

The correct answer is: Coxsackie B

moodle.feu-nrmf.edu.ph/mod/quiz/review.php?attempt=419239&cmid=32627&showall=1 21/42
10/27/2020 Pediatrics 3B 2nd Prelim Exam: Attempt review

Question 45
Correct

Mark 1.00 out of 1.00

Jim, a 10 y/o boy was seen at the clinic because of painful swelling in front of the right earlobe obscuring the right
angle of the jaw and pushing the earlobe upward and outward. Otoscopy was normal. His immunization from the
local health center was incomplete. What is the most common complication of his illness?

a. deafness

b. pancreatitis

c. orchitis

d. meningitis with or without encephalitis 

The correct answer is: meningitis with or without encephalitis

Question 46

Correct

Mark 1.00 out of 1.00

Jim, a 2 year old boy was seen at the clinic because of slight fever and rashes of two days. PE showed scattered
vesicles on the tongue and buccal mucosa together with maculopapular, vesiculo- pustular lesions on the arms
and legs. This child is most contagious within how many days of illness?

a. 14

b. 7 

c. 5

d. 10

The correct answer is: 7

moodle.feu-nrmf.edu.ph/mod/quiz/review.php?attempt=419239&cmid=32627&showall=1 22/42
10/27/2020 Pediatrics 3B 2nd Prelim Exam: Attempt review

Question 47
Incorrect

Mark 0.00 out of 1.00

Jose, 4 months old, was admitted due to poor breastfeeding. He prefers cold water instead of breastmilk. He is
warm to touch, cries incessantly unless given cold water and voids frequently and voluminously. Weight and
length were below -2SD. LABS: Pre-desmopressin serum osmolality 320 mOsm/kg and urine osmolality was 80
mOsm/kg. Post desmopressin serum osmolality 280 mOsm/kg and urine osmolality was 290 mOsm/kg. This is
TRUE about the main impression:

a. It is due to inactivating mutations of the vasopressin v2 receptors 

b. Can be a consequence of histiocytosis

c. Earlier and more severe symptoms are seen among males

d. All choices are correct.

The correct answer is: Can be a consequence of histiocytosis

Question 48

Incorrect

Mark 0.00 out of 1.00

Julius a 2-months old, presented with jaundice, purpura, hepatosplenomegaly, microcephaly, intracranial
calcification and chorioretinitis. Which test can establish the etiologic diagnosis?

a. CSF PCR DNA 

b. stool culture

c. IgM to viral capsid antigen

d. urine early antigen test or pcr

The correct answer is: urine early antigen test or pcr

moodle.feu-nrmf.edu.ph/mod/quiz/review.php?attempt=419239&cmid=32627&showall=1 23/42
10/27/2020 Pediatrics 3B 2nd Prelim Exam: Attempt review

Question 49
Incorrect

Mark 0.00 out of 1.00

Majority of neonatal infections with these viruses are asymptomatic EXCEPT:

a. Non-polio Enteroviruses

b. Herpes simplex virus 2

c. Hepatitis B virus 

d. CMV

The correct answer is: Herpes simplex virus 2

Question 50

Incorrect

Mark 0.00 out of 1.00

Mary, 11-years-old, Filipino, came in for immunization update. She is asymptomatic at the time of consult, with
normal vital signs but has a history of irregular menses. BMI was plotted between +1 to +2 SD. PE showed hirsutism
and diffused hyperpigmentation on the nape. Anticipatory management dictates that you do a:

a. none of the options is correct

b. fasting non-HDL cholesterol determination 

c. fasting blood sugar

d. fasting lipid profile

The correct answer is: fasting blood sugar

Question 51
Correct

Mark 1.00 out of 1.00

Neonatal infections with this virus ranges from an asymptomatic infection to a benign febrile illness. A minority of
infections though presents as severe multi systemic disease dominated by any combination of sepsis,
meningoencephalitis, myocarditis, hepatitis, coagulopathy and pneumonitis:

a. CMV

b. Herpes virus simplex 2

c. Enterovirus 

d. EBV

The correct answer is: Enterovirus

moodle.feu-nrmf.edu.ph/mod/quiz/review.php?attempt=419239&cmid=32627&showall=1 24/42
10/27/2020 Pediatrics 3B 2nd Prelim Exam: Attempt review

Question 52
Correct

Mark 1.00 out of 1.00

NN was brought to your clinic due to Increased drooling, refusal to eat or drink milk. On PE of the oral cavity, there
were scattered ulcers on the palate, tongue, buccal mucosa and inner lips. Gums were erythematous and
swollen. This is the most common manifestation of a primary infection with:

a. adenovirus

b. epstein barr virus

c. herpes simplex virus 1 

d. coxsackie A virus

The correct answer is: herpes simplex virus 1

Question 53

Incorrect

Mark 0.00 out of 1.00

Pao, 15 years old, male was diagnosed as a new onset T2DM and has been on metformin for 4 months. Insulin
should be started if:

a. FBS is greater than 300 mg/dL

b. All choices are correct 

c. HgbA1C is greater than or equal to 7%

d. OGTT is greater than 250 mg/dL

The correct answer is: HgbA1C is greater than or equal to 7%

Question 54
Incorrect

Mark 0.00 out of 1.00

Patient TT is diagnosed with the most common sex chromosomal disorder among males and the most common
cause of male hypogonadism. The following statements are true regarding this disorder EXCEPT:

a. Variants exist that are directly correlated with the severity of the symptoms.

b. It is associated neurocognitive, behavioral and psychiatric disorders.

c. Diagnosis is missed in 10% of adolescent patients.

d. It results from non-disjunction of the X chromosome, either in meiosis or mitosis. 

The correct answer is: Diagnosis is missed in 10% of adolescent patients.

moodle.feu-nrmf.edu.ph/mod/quiz/review.php?attempt=419239&cmid=32627&showall=1 25/42
10/27/2020 Pediatrics 3B 2nd Prelim Exam: Attempt review

Question 55
Correct

Mark 1.00 out of 1.00

Principles in the management of patients with Graves’ disease include:

a. Adequate control is evident within 3-4 weeks of methimazole treatment.

b. The duration of treatment is directly related to the relapse rate.

c. Relapses usually occur 2-3 years after therapy has been discontinued.

d. The dose of methimazole should be reduced if serum TSH is greater than normal. 

The correct answer is: The dose of methimazole should be reduced if serum TSH is greater than normal.

Question 56

Correct

Mark 1.00 out of 1.00

Resident is attending to a 9-month-old infant with measles. The routine management of Measles includes:

a. Antibiotics

b. antihistamines

c. Ribavirin

d. Vitamin A supplementation 

The correct answer is: Vitamin A supplementation

Question 57
Correct

Mark 1.00 out of 1.00

Residents, clerks and interns are discussing corona virus infections in the admission rounds. This is a true
statement regarding Corona Virus:

a. Transmission of SARS CoV-2 is mainly via respiratory droplet and to a lesser degree through fomites. 

b. Rapid Antibody test is accurate and may replace PCR for SARS CoV-2

c. MERS CoV has 65% mortality.

d. Isolation is a minimum of 5 days.from a positive SARS CoV-2 PCR

The correct answer is: Transmission of SARS CoV-2 is mainly via respiratory droplet and to a lesser degree
through fomites.

moodle.feu-nrmf.edu.ph/mod/quiz/review.php?attempt=419239&cmid=32627&showall=1 26/42
10/27/2020 Pediatrics 3B 2nd Prelim Exam: Attempt review

Question 58
Incorrect

Mark 0.00 out of 1.00

RG, 7 y/o was apparently well when he was rushed to the ER because of focal seizures. Neurological Exam was
normal. MRI showed progressive damage of the frontal lobe and white matter. 3 months later, there was
progressive cognitive, behavioral, psychomotor deterioration, including blindness, deafness. What mechanism
explains the progression of the illness?

a. impaired β-oxidation increases VLCFA that causes demyelination and CNS degeneration

b. DAX-1 gene mutation decreases cortisol causing the neuromuscular retardation 

c. significant hypoglycemia causes seizures and hypoxemia that bring about the clinical deterioration

d. SF-1 disruption causes defective steroidogenesis

The correct answer is: impaired β-oxidation increases VLCFA that causes demyelination and CNS degeneration

Question 59

Correct

Mark 1.00 out of 1.00

Ronn, a 7-year-old boy was admitted for high-grade fever for 4 days, poor intake and weakness. There was no
vomiting, LBM nor abdominal pain. He was awake with dry oral mucosa and flushed skin with last urine output
while at ER. Vital signs were as follows: CR 90/min, RR 28/min, Temp 38.5 OC, BP 90/60. CRT less than 2 sec with
flushed skin. Torniquet test was positive. The rest of the PE was unremarkable. CBC showed Hgb 13.5 g/L, HCT 0.39,
WBC count 3.5 x 109, segmenters 0.32, lymphocytes 0.68, platelet count 150,000 U/L. What is the most likely
diagnosis?

a. Severe Dengue

b. Dengue fever without warning signs 

c. Systemic viral infection

d. Dengue with warning signs

The correct answer is: Dengue fever without warning signs

moodle.feu-nrmf.edu.ph/mod/quiz/review.php?attempt=419239&cmid=32627&showall=1 27/42
10/27/2020 Pediatrics 3B 2nd Prelim Exam: Attempt review

Question 60
Incorrect

Mark 0.00 out of 1.00

Signs and symptoms of myalgia, fatigue and anorexia with findings of ketosis, eosinophilia and lymphocytosis in
a patient with adrenocortical insufficiency are mainly due to deficiency/deficiencies of:

a. Glucocorticoid

b. Glucocorticoid, mineralocorticoid, Na and glucose 

c. Glucocorticoid and mineralocorticoid

d. Mineralocorticoid

The correct answer is: Glucocorticoid

Question 61

Correct

Mark 1.00 out of 1.00

Tanya, a 12-year-old girl developed crops of painful vesicles at the left upper vermillion border of the lips. Which is
the most likely etiologic agent?

a. Herpes simplex virus 1 

b. Coxsackie A

c. Ebstein barr virus

d. Adenovirus

The correct answer is: Herpes simplex virus 1

moodle.feu-nrmf.edu.ph/mod/quiz/review.php?attempt=419239&cmid=32627&showall=1 28/42
10/27/2020 Pediatrics 3B 2nd Prelim Exam: Attempt review

Question 62
Correct

Mark 1.00 out of 1.00

Ted, a 15 y/o boy was seen at the clinic for 9 days of on and off fever, sore throat and fatigue despite Paracetamol
and Amoxycillin. PE: moderately enlarged and congested tonsils, bilateral non-tender cervical lymph nodes (1.5-2
cm), splenomegaly. CBC: Hgb 12.3, Hct 36, WBC 15,000 seg 0.24, lymph0 .73, eos 0.03, Plt 204 and with 30% atypical
lymphocytes. Which is the most common causative agent?

a. EBV 

b. CMV

c. Coxsackie A virus

d. Adenovirus

The correct answer is: EBV

Question 63

Incorrect

Mark 0.00 out of 1.00

The appropriate screening tool for a 6-year-old, Filipino, BMI z-score -2 SD, with polyuria, polydipsia and
polyphagia is:

a. OGTT

b. fasting non HDL-C 

c. FBS

d. urinalysis

The correct answer is: OGTT

moodle.feu-nrmf.edu.ph/mod/quiz/review.php?attempt=419239&cmid=32627&showall=1 29/42
10/27/2020 Pediatrics 3B 2nd Prelim Exam: Attempt review

Question 64
Correct

Mark 1.00 out of 1.00

The appropriate screening tool for an 8-year-old, Filipino, BMI z-score 1.2 SD, diagnosed with nephrotic syndrome
at 7 years old is:

a. FBS

b. non-fasting non HDL-C 

c. none of the options is correct

d. OGTT

The correct answer is: non-fasting non HDL-C

Question 65

Correct

Mark 1.00 out of 1.00

The diagnostic criteria in the diagnosis of thyroid storm DOES NOT include:

a. abdominal enlargement 

b. bipedal edema

c. extreme lethargy

d. atrial fibrillation

The correct answer is: abdominal enlargement

Question 66
Correct

Mark 1.00 out of 1.00

The first specific sign of rabies infection is:

a. Hydrophobia

b. Paresthesia at bite site 

c. Throat pain

d. Aerophobia

The correct answer is: Paresthesia at bite site

moodle.feu-nrmf.edu.ph/mod/quiz/review.php?attempt=419239&cmid=32627&showall=1 30/42
10/27/2020 Pediatrics 3B 2nd Prelim Exam: Attempt review

Question 67
Correct

Mark 1.00 out of 1.00

The following are AIDS defining illnesses EXCEPT:

a. Kaposi sarcoma

b. Esophageal candidiasis

c. Tuberculosis 

d. Pneumocystis pneumonia

The correct answer is: Tuberculosis

Question 68

Correct

Mark 1.00 out of 1.00

The following is/are TRUE of Oseltamivir in the management of Influenza:

a. All choices are correct 

b. It is best given within 48 hours from the onset of illness

c. It may be given to hospitalized patients after 48 hours from the onset of illness

d. It is given as prophylaxis

The correct answer is: All choices are correct

Question 69
Correct

Mark 1.00 out of 1.00

The following should undergo routine screening for Hepatitis C except for:

a. patients with Idiopathic liver disease

b. pregnant women 

c. patients on hemodialysis

d. patients with history of illegal drug use

The correct answer is: pregnant women

moodle.feu-nrmf.edu.ph/mod/quiz/review.php?attempt=419239&cmid=32627&showall=1 31/42
10/27/2020 Pediatrics 3B 2nd Prelim Exam: Attempt review

Question 70
Incorrect

Mark 0.00 out of 1.00

The highest risk of mother to child transmission in HIV is during:

a. breastfeeding

b. mother to fetus transmission/transplacental 

c. caesarian section

d. vaginal delivery

The correct answer is: vaginal delivery

Question 71

Correct

Mark 1.00 out of 1.00

The most common etiologic agent of Brochiolitis and Pneumonia in infants is:

a. S. pneumoniae

b. Rhinovirus

c. Influenza

d. Respiratory syncitial virus 

The correct answer is: Respiratory syncitial virus

Question 72
Correct

Mark 1.00 out of 1.00

The most common infectious cause of primary adrenal insufficiency resulting to the dreaded adrenal crisis is:

a. Tuberculous adrenalitis

b. Full-blown AIDS

c. Cryptococcal meningitis

d. Waterhouse-Friedrichsen syndrome 

The correct answer is: Waterhouse-Friedrichsen syndrome

moodle.feu-nrmf.edu.ph/mod/quiz/review.php?attempt=419239&cmid=32627&showall=1 32/42
10/27/2020 Pediatrics 3B 2nd Prelim Exam: Attempt review

Question 73
Correct

Mark 1.00 out of 1.00

The most common presenting sign among patients with acquired hypothyroidism is:

a. goiter 

b. weight gain

c. myxedema

d. slowing of growth

The correct answer is: goiter

Question 74

Incorrect

Mark 0.00 out of 1.00

The risk of Congenital Varicella is highest when maternal infection is between how many weeks of gestation:

a. 29-34

b. 21-28 

c. 4-12

d. 13-20

The correct answer is: 13-20

Question 75
Correct

Mark 1.00 out of 1.00

The triad of headache, sweating, and heart palpitations should raise a high index of suspicion of this clinical
disorder in a 10-year-old male especially when concurrent hypertension exists:

a. Paraganglionoma

b. Neuroblastoma

c. Pheochromocytoma 

d. All choices are correct

The correct answer is: Pheochromocytoma

moodle.feu-nrmf.edu.ph/mod/quiz/review.php?attempt=419239&cmid=32627&showall=1 33/42
10/27/2020 Pediatrics 3B 2nd Prelim Exam: Attempt review

Question 76
Correct

Mark 1.00 out of 1.00

These endocrine disorders will manifest with significant hypertension and signs and symptoms of
hyperandrogenism EXCEPT:

a. Cushing syndrome

b. Primary aldosteronism 

c. 17-α-hydroxylase/17,20-lyase deficiency

d. 11-β-hydroxylase deficiency

The correct answer is: Primary aldosteronism

Question 77

Correct

Mark 1.00 out of 1.00

This endocrine disorder has an increased incidence in adulthood of central adiposity, metabolic syndrome, and
breast cancer:

a. Klinefelter syndrome 

b. Late onset puberty

c. Turner syndrome

d. Kallman syndrome

The correct answer is: Klinefelter syndrome

Question 78
Incorrect

Mark 0.00 out of 1.00

This is the drug of choice in the management of patients with both growth hormone and prolactin over secretion:

a. bromcriptine

b. pegvisomant

c. octreotide 

d. cabergoline

The correct answer is: cabergoline

moodle.feu-nrmf.edu.ph/mod/quiz/review.php?attempt=419239&cmid=32627&showall=1 34/42
10/27/2020 Pediatrics 3B 2nd Prelim Exam: Attempt review

Question 79
Correct

Mark 1.00 out of 1.00

This type of Influenza is capable of initiating a pandemic:

a. B

b. C

c. A 

d. D

The correct answer is: A

Question 80

Correct

Mark 1.00 out of 1.00

This viral infection occurs in children 3 months to 3 years, presenting with moderate grade fever, seizures, rashes
on defervesence, and bulging fontanelle.

a. Roseola infantum 

b. Fifth Disease

c. Measles

d. Varicella

The correct answer is: Roseola infantum

Question 81
Correct

Mark 1.00 out of 1.00

This virus is a polytropic virus that can cause pharynconjunctivitis, gastroenteritis, cystitis and pneumonia:

a. Influenza

b. Rotavirus

c. Adenovirus 

d. Epstein barr virus

The correct answer is: Adenovirus

moodle.feu-nrmf.edu.ph/mod/quiz/review.php?attempt=419239&cmid=32627&showall=1 35/42
10/27/2020 Pediatrics 3B 2nd Prelim Exam: Attempt review

Question 82
Incorrect

Mark 0.00 out of 1.00

This/These statement/s is/are TRUE regarding Measles:

a. Tuberculin testing should be done after 4-6 weeks to avoid false positive results

b. TB may be reactivated by measles infection

c. All choices are correct

d. Vitamin A administration is repeated after 2 weeks in healthy individuals 

The correct answer is: All choices are correct

Question 83

Correct

Mark 1.00 out of 1.00

Ulcers at the uvulopalatoglossal junction among infants with roseola is known as:

a. Nagayama spots 

b. Forsheimer’s spot

c. Koplick spots

d. Café au lait spot

The correct answer is: Nagayama spots

Question 84
Correct

Mark 1.00 out of 1.00

Vaccination is an important preventive measure for these diseases, EXCEPT:

a. Hepatitis C 

b. Hepatitis A

c. Hepatitis D

d. Hepatitis B

The correct answer is: Hepatitis C

moodle.feu-nrmf.edu.ph/mod/quiz/review.php?attempt=419239&cmid=32627&showall=1 36/42
10/27/2020 Pediatrics 3B 2nd Prelim Exam: Attempt review

Question 85
Correct

Mark 1.00 out of 1.00

Vanessa, a 14 y/o who loves to eat street foods developed on and off fever, anorexia, with abdominal pain and
vomiting for 4 days. On the day of consult, she became afebrile but with note of generalized yellowish
discoloration of skin and dark yellow urine. If this were hepatitis A, she is considered to be contagious until how
many days after the jaundice?

a. 7 

b. 10

c. 14

d. 21

The correct answer is: 7

Question 86

Incorrect

Mark 0.00 out of 1.00

VV, 5 y/o is being worked up for autoimmune Addison disease. Which particular criterion must be present for a
definitive diagnosis of this endocrine disorder?

a. concomitant Hashimoto thyroiditis

b. exclusion of tuberculosis and intake of rifampicin

c. high titer of anticortex adrenal antibodies

d. low cortisol and high ACTH levels 

The correct answer is: exclusion of tuberculosis and intake of rifampicin

Question 87
Incorrect

Mark 0.00 out of 1.00

What is the most common opportunistic infection in children with HIV Infection?

a. Tuberculosis 

b. Oral Candidiasis

c. PCP Pneumonia

d. Recurrent Bacterial Parotitis

The correct answer is: PCP Pneumonia

moodle.feu-nrmf.edu.ph/mod/quiz/review.php?attempt=419239&cmid=32627&showall=1 37/42
10/27/2020 Pediatrics 3B 2nd Prelim Exam: Attempt review

Question 88
Incorrect

Mark 0.00 out of 1.00

Which is associated with fulminant hepatitis particularly among pregnant women with high fatality rate?

a. Hepatitis E

b. Hepatitis A

c. Hepatitis B

d. Hepatitis D 

The correct answer is: Hepatitis E

Question 89

Correct

Mark 1.00 out of 1.00

Which is considered the most valuable and specific serologic test for the diagnosis of acute EBV infection in
children?

a. IgG to viral capsid antigen (VCA)

b. Heterophil antibody test

c. antibody to early antigen(EA)

d. IgM to viral capsid antigen (VCA) 

The correct answer is: IgM to viral capsid antigen (VCA)

Question 90
Correct

Mark 1.00 out of 1.00

Which is considered the single diagnostic test of choice for susceptibility to hepatitis B?

a. Anti-HBeAg

b. Anti-HBs Ag

c. Anti-HBcAg 

d. HBsAg

The correct answer is: Anti-HBcAg

moodle.feu-nrmf.edu.ph/mod/quiz/review.php?attempt=419239&cmid=32627&showall=1 38/42
10/27/2020 Pediatrics 3B 2nd Prelim Exam: Attempt review

Question 91
Incorrect

Mark 0.00 out of 1.00

Which is NOT TRUE about the circulating vaccine-derived poliovirus (c-VDPV)?

a. It is transmissible.

b. It causes rare episodes of paralysis

c. It is identical to sabin vaccine virus.

d. It cannot cause a paralytic disease if population is fully immunized 

The correct answer is: It is identical to sabin vaccine virus.

Question 92

Correct

Mark 1.00 out of 1.00

Which of the following diarrhea-causing viruses is/are vaccine preventable?

a. All

b. Norovirus

c. Paranfluenza

d. Rotavirus 

The correct answer is: Rotavirus

Question 93
Incorrect

Mark 0.00 out of 1.00

Which of the following is a CORRECT statement regarding SSPE?

a. It occurs in children less than 2 years of age

b. It is a slowly progressive degenerative disease

c. It is the most common neurologic complication of measles. 

d. Measles antibodies are seen in the CSF but not in the blood

The correct answer is: It is a slowly progressive degenerative disease

moodle.feu-nrmf.edu.ph/mod/quiz/review.php?attempt=419239&cmid=32627&showall=1 39/42
10/27/2020 Pediatrics 3B 2nd Prelim Exam: Attempt review

Question 94
Correct

Mark 1.00 out of 1.00

Which of the following is an indication for Acyclovir among patients with Varicella?

a. A 13-year-old child who is apparently healthy before developing varicella. 

b. 3-year-old child who acquired the infection from a household member.

c. A 5-year-old child who is apparently healthy before developing varicella.

d. An 8-year-old whose rashes were noted 4 days prior to consult.

The correct answer is: A 13-year-old child who is apparently healthy before developing varicella.

Question 95

Incorrect

Mark 0.00 out of 1.00

Which of the following is indicative of a secondary acquired hypothyroidism?

a. high TSH, low FT4 

b. low TSH, low FT4

c. high TSH, high FT4

d. low TSH, high FT4

The correct answer is: low TSH, low FT4

Question 96
Correct

Mark 1.00 out of 1.00

Which of the following is TRUE in the management of Dog bite?

a. History of rabies preexposure immunization will reduce the number of doses of vaccine to be 
administered

b. Wound is washed with soap and water for 5 mins and iodine or alcohol is applied.

c. Suturing should be avoided however, if unavoidable, it may be done 1 hour after rabies vaccine
administration.

d. Antibiotics is not necessary

The correct answer is: History of rabies preexposure immunization will reduce the number of doses of vaccine to
be administered

moodle.feu-nrmf.edu.ph/mod/quiz/review.php?attempt=419239&cmid=32627&showall=1 40/42
10/27/2020 Pediatrics 3B 2nd Prelim Exam: Attempt review

Question 97
Incorrect

Mark 0.00 out of 1.00

Which of the following is true of SARS CoV-2 Infection in children?

a. Most common symptoms are fever and diarrhea. 

b. Remdesivir is licensed for the treatment in children.

c. Symptoms are less severe in children.

d. Incidence is higher in children than adults.

The correct answer is: Symptoms are less severe in children.

Question 98

Correct

Mark 1.00 out of 1.00

Which statement is NOT TRUE about the WHO acute flaccid paralysis surveillance?

a. Cases of Guillain-Barre syndrome, transverse myelitis, or traumatic paralysis need not be reported. 

b. Any child less than 15 years of age with acute flaccid paralysis should be considered a “suspect”.

c. There should be a 60-day follow-up by disease surveillance officer to check for residual paralysis.

d. Two stool specimens, taken 24- 48 hours apart within 14 days of onset of paralysis should be collected.

The correct answer is: Cases of Guillain-Barre syndrome, transverse myelitis, or traumatic paralysis need not be
reported.

Question 99
Correct

Mark 1.00 out of 1.00

Which virus is associated with microcephaly in fetal infections?

a. Dengue virus

b. Japanese encephalitis

c. Zika virus 

d. Chikungunya

The correct answer is: Zika virus

moodle.feu-nrmf.edu.ph/mod/quiz/review.php?attempt=419239&cmid=32627&showall=1 41/42
10/27/2020 Pediatrics 3B 2nd Prelim Exam: Attempt review

Question 100
Correct

Mark 1.00 out of 1.00

Which virus is most commonly associated with an illness characterized as acute fever, with hepatosplenomegaly,
rash, lymphadenopathy, neurologic symptoms, cytopenias, high serum ferritin and liver function abnormalities?

a. Varicella zoster

b. HSV 1 and 2

c. EBV 

d. CMV

The correct answer is: EBV

PREVIOUS ACTIVITY
◄ Endocrinology B video lecture (Dr. Naomi Nocheseda)

Jump to...

NEXT ACTIVITY
Midterms Pediatrics 3B ►

Stay in touch
 http://www.feu-nrmf.ph
 +632 8983 8338 / +639176272623
 info@feu-nrmf.ph

 Data retention summary

 Get the mobile app

moodle.feu-nrmf.edu.ph/mod/quiz/review.php?attempt=419239&cmid=32627&showall=1 42/42
2. An infant was seen due to seizures. The mother noticed that the child has an unpleasant urine odor. PPE: 
microcephaly, prominent maxillae and widely spaced teeth. Which of the following is the probable developmental 
outcome of this child?
a. Communication disorder 
b. Intellectual disability 
c. Attention-deficit/Hyperactivity disorder 
d. Autism spectrum disorder 

17. A 32-week AOG neonate delivered by Emergency CS because of eclampsia was tachypneic with cyanosis, 
tachycardic and was grunting at 4 hours of life. In spite of oxygen support and thermoregulation, there was further 
worsening of the condition. The most possible diagnosis is?
a. Transient tachypnea of the newborn 
b. Neonatal pneumonia 
c. Meconium Aspiration syndrome 
d. Hyaline membrane disease 

18. TIME-OUT is the removal of positive reinforcement for an unacceptable behavior. This is a method used to address 
temper tantrums and is computed at?
a. 2 minutes/year of age 
b. 1 minute/year of age 
c. 4 minutes/year of age 
d. 3 minutes/year of age 

22. An 18 month old girl had remittent moderate grade fever for 2 days. There were no associated signs and 
symptoms and she has normal appetite and is playful. Physical examination was insignificant. Your management 
option will be?
a. Give paracetamol and observe closely at home and return for any untoward symptoms  
b. Request for a CBC, urinalysis and blood culture 
c. Admit the patient 
d. Start oral amoxicillin only 

23. Which of the following physiologic responses lead to hypertension?


a. Polyuria 
b. Vasodilation 
c. Polycythemia 
d. Bradycardia 

31. Gabby a 1 year old boy has had 5 episodes of bacterial pneumonia. What is the most likely diagnosis
a. Humoral immunodeficiency 
b. Cellular immunodeficiency 
c. Complement immunodeficiency 
d. Phagocytic immunodeficiency  

34. A 9-year-old was brought to the ER because of ear pain of 3 days with colds, and occasional productive cough. 
PPE: yellowish discharge on the left ear with accompanying ear pain. Acute otitis media is highly entertained for 
which the recommended first line treatment

a. Dexamethasone (0.6-1 mg/kg)


B. Amoxicillin at 80-90mg/kg/day
C. Azithromycin 10 mg/kg/day
D. Clindamycin 7 mg/kg/dose for 10 days

36.  A  12  year-old  athlete had a chest radiograph as part of sports clearance CXR: cardiomegaly with RV prominence 


and increased pulmonary blood flow. These findings can be seen in this CHD?
a . VSF
B. PDA
C. Aortic Stenosis
D. ASD
35.  A  group  of  students  presented  with nausea, vomiting, and circumoral cyanosis after eating the noodles prepared 
by their teacher. What test is needed to identify the case of circumoral cyanosis
a. 2 panel drug testing 
b. Filter paper test 
c. Blood mercury determination 
d. 5 panel drug testing 

39. Which of the following is a risk factor for Late-onset Neonatal Sepsis
a. Indwelling venous or arterial catheters 
b. Maternal Fever 
c. Premature rupture of membranes of more than 18 hours 
d. Chorioamnionitis  

A  7  year  old  patient  presented  at  the  ER  complaining  of  dyspnea.  On  history,  he  had  URTI  2  weeks  ago.  On  PE you 
noted a grating to and fro sound on the left parasternal area. What is your diagnosis?
a. Restrictive cardiomyopathy 
b. Acute myocarditis 
c. Dilated Cardiomyopathy 
d. Acute pericarditis 

41. A 16 year old had febrile UTI. CT Scan revealed localized renal parenchymal mass without liquefaction. What 
should be the duration of treatment?
a. 5 days 
b. 10 days 
c. 7 days 
d. 14 days 

42. What is the daily caloric requirement of a 1 year old infant?


a. 500 cal 
b. 1000 cal 
c. 1500 cal 
d. 750 cal 

43. A child diagnosed with Attention Deficit Hyperactivity Disorder has the highest risk of having which of the following 
conditions?
a. Anxiety disorder 
b. Mood disorder 
c. Learning disorder 
d. Language disorder 

44. Prior to sending home the newborn with the mother, you will advise proper attachment during breastfeeding. 
Which of the following will you recommend?

a. More areola visible below than above the mouth 


b. Infant’s body is turned towards the mother 
c. Infant’s lower lip turned inward 
d. Infant’s neck is bent anteriorly 

45. Ace was hit by a car while crossing the street and sustained a severe traumatic brain injury (TBI). Which of the 
following is true in the management of TBI?
a. None of the choices are correct 
b. Maintain hyperthermic condition 
c. Use of hypotonic solution as maintenance fluids 
d. Head of bed elevation to 30 degrees 
46. Co-morbid of allergic rhinitis include/s?
a. Otitis media 
b. Sinusitis 
c. All of the choices are correct 
d. Bronchial asthma 

47. In a patient with anaphylaxis, the initial assessment should ensure an adequate airway, effective respiration, 
circulation and perfusion. The most important initial drug to administer without delay is?
a. Intramuscular H1 and H2 antihistamine antagonists 
b. Corticosteroid 
c. Epinephrine 1:1000 IV 
d. Epinephrine 1:10,000 IV 

48. Cushing’s triad is composed of?


a. Hypertension, bradycardia, bradypnea 
b. Hypertension, tachycardia, tachypnea 
c. Normotension, normal cardiac rate, normal respiratory rate 
d. Normotension, bradycardia, tachypnea 

49. An apparently well 1 year old infant was brought to the ER because of seizures a few minutes after consuming his 
milk formula. Which of the following agents is the LEAST likely cause of the seizures?
a. Methamphetamine 
b. Paracetamol 
c. Monosodium glutamate 
d. INH 

50. In a high quality CPR, if advanced airway is in place how many breaths should be given?
a. One breath in 2-3 seconds 
b. One breath in 6-8 seconds 
c. One breath in 8-10 seconds 
d. One breath in 3-5 seconds 

51. 

52.  Sandra developed pruritic erythematous ​papulovesicular rashes​ on  the ears after 2 weeks of ​wearing a new pair 
of earrings.​ What is the diagnostic test for the condition?
A. Intradermal skin test 
B. Allergy skin prick test 
C. Patch test 
D. In-vitro allergy test 

53. You were called by the OB resident for a possible imminent delivery. What would be the least important 
information you would ask?
A. Manner of delivery 
B. Is the pregnancy single or multiple? 
C. What is the age of gestation 
D. Status of cervical dilatation? 

54. What is the usual initial site of involvement of atopic march?


A. Lungs 
B. Nose 
C. Neck 
D. Skin 

55. A 14 year old girl was brought to the ER because of vomiting and abdominal pain. She told you that she ate the 
mushroom that was cooked by their neighbor. Which of the following information is ​LEAST​ helpful in identifying the 
type of mushroom?
A. Cap appearance 
B. Texture 
C. Stem appearance 
D. Color 

56. An​ 8 month old infant ​came in for 8x watery stools for the past 2 days. Assessment revealed ​dry mouth and lips​, 
breastfeeding ​more than the usual frequency ​and is ​fretful​. Most likely he has?
A. Some dehydration, mild 
B. Some dehydration 
C. No dehydration 
D. Some dehydration, moderate 

57. A 1 year old female had 3 days intermittent high-grade fever followed by generalized muculo-papular rashes after 
the fever subsided. It was accompanied by soft stools. There was no change in her appetite and activity. Diagnosis?
A. Exanthem subitem 
B. Rubella 
C. Rubeola 
D. Roseola 

58. A 2 day old had circumoral cyanosis when crying. The baby is comfortable, with an oxygen saturation of 84%, with 
grade 2/6 murmur appreciated at LUSB. His CXR showed decreased vascular markings. Which of the following is most 
likely anatomy you will find  on echo?
A. Truncus arteriosus 
B. TAPVR 
C. TGA 
D. PVA 

59. A ​6 month old​ was seen to be unresponsive. Where should you check the pulse?
a. Brachial  ​/ 
b. Popliteal 
c. Carotid 
d. Femoral  / 

60.  A febrile 6 month old female had a urinalysis result of:


● Yellow hazy, 1020, 6.5 (-) protein and glucose 
● RBC = 5-6/hpf 
● PC = 20/hpf + WBC esterase  
●(-) nitrite 
Among the laboratory tests, what will you prioritize?

A. UTZ of the kidneys and urinary bladder with post void bladder US 
B. CRP 
C. Urine culture with colony count and sensitivity 
D. Prolactin level 

61. What is a test for complement deficiency


a. Absolute neutrophil count 
b. CH50 
c. Respiratory burst assay 
d. CD16, CD56 

62.

63.

64. A 5 month old male had 9 days high fever and irritability. Labs: CRP: 5mg/dl, ESR: 50mm/hr, Hbg 90,
WBC 25. Platelet 650,000, urine WBC 12 wbc/hpf, albumin 3g/dl. 2D echo: normal sized coronary arteries, pericardial 
effusion, and mitral regurgitation. Based on AHA guidelines, what should be the next step in the management?
a. Treat with broad spectrum antibiotics 
b. Do serial clinical and laboratory evaluation 
c. Wait for typical peeling to develop before treatment 
d. Give IV immunoglobulin and salicylates 

65. A mother who gave birth via cesarean section is complaining of difficulty in breastfeeding because of incision site 
pain. She asked you for the most convenient and acceptable breastfeeding position. What will you recommend?
a. Cradle 
b. Cross-arm 
c. Lying down on side 
d. underarm 

66. 

67.

68. Gabby, a 1 year old boy has had 5 episodes of bacterial pneumonia. Aside from CBC, another laboratory exam 
to help diagnose his above condition?

a. Delayed hypersensitivity test like PPD candida 


b. Iso-hemagglutinin test 
c. T-cell count 
d. Serum immunoglobulin levels 

69. In diagnosing urinary tract infection which parameter is the most important in marking a diagnosis?

a. Odor 
b. Presence of nitrites 
c. Presence of ketones 
d. color 

70. The sensorium of a fretful 10-month-old infant with diarrhea for the past 3 days deteriorated. Hence you opted to 
administer an IV fluid to correct the dehydration. What would be the total amount?
a. 100 ml 
b. 800 ml 
c. 400 ml 
d. 600 ml 

71. What should you monitor daily in a child while on intravenous fluid?
a. Chest radiograph 
b. Weight 
c. Creatinine 
d. Urinalysis 

72. Which of the following is a recommended practice in the care of the newborn based on the WHO and DOH Early 
Essential Newborn Care?
a. Early bathing once the mother and baby dyad is at the rooming in area 
b. Immediate cord clamping 
c. Early vitamin K administration and eye prophylaxis after placing baby on skin to skin contact with mother 
d. Immediate and thorough drying after birth 

73. A 15-year old hypertensive was given spironolactone to lower his BP. What electrolytes imbalance is expected?
a. Hypocalcemia 
b. Hyperkalemia 
c. Hypernatremia 
d. hyperchloremia 

74. Diamond Blackfan syndrome can be differentiated from the acquired form of hypoplastic anemia by the 
following laboratory findings?
a. Elevated ADA, elevated HgbF, elevated I antigen 
b. Normal MVC, normal HgbF, normal ADA 
c. Elevated MCV, normal HgbF, elevated ADA 
d. Elevated ADA, normal HgbF, normal I antigen 

75.  Age- appropriate physical activities for children and adolescents are recommended at?
a. 45 minutes daily or on most days of the week 
b. 60 minutes daily or on most days of the week 
c. 15 minutes daily or on most days of the week 
d. 30 minutes daily or on most days of the week 

76. A 1 year old male, afebrile had a urine culture on a urine bag collection result of E. coli 100,000 cfu. The next thing 
you will do?
a. Do a congenital anomaly screening 
b. Do a kidney and urinary bladder with post void bladder ultrasound 
c. Repeat urine culture using a mid-stream catch urine sample 
d. Start cefuroxime at 50mg/kg/day 

77.  In acute primary peritonitis, the most commonly found organism is?
a. E . coli 
b. Pneumococci 
c. Staphylococci 
d. Group A streptococcus 

78. You are examining a 9 year old with CHD FDA. of the following findings, which signifies cardiomegaly?
a. Dynamic precordium 
b. Bounding pulses 
c. A gr 3/6 continuous murmur on 2nd ICSLUSB 
d. Apex beat at the 5th ICSLMCL 

79. A 3 week old breastfed infant has a good suck and is afebrile. The weight is 3.6 kg with a birth weight of 3 kg. The 
baby is fed every 2-3 hours with 6-7 diaper changes per day. Yellowish stool is passed daily. The infant has generalized 
jaundice. What is the most likely diagnosis?
a. Pneumonia 
b. Breastmilk jaundice? 
c. Breastfeeding jaundice 
d. Sepsis  

80. In Diphyllobothriasis, the adult worms use the vitamin B-12 of the host for the production of segments and eggs thus 
inhibiting vitamin B-12 uptake of the host. What will this lead to?
a. Aplastic anemia 
b. Iron deficiency anemia 
c. Megaloblastic anemia 
d. Hemolytic anemia 

85. The major manifestations of Rheumatic fever are the following EXCEPTt?
A. Carditis 
B. Chorea 
C. Fever 
D. Erythema Marginatum 
86. What other diagnostic procedure/s can be done to test for B cell function aside from immunoglobulin levels?
A. CD19, 20 
B. Iso-hemagglutinins in patients in blood type AB 
C. IgG subclass measurement 
D. Test for antibodies for diphtheria, tetanus, hemophilus influenza and pneumococcal antigens in patients less 
than 2 years old 

87. JD a 2 year old female has high grade fever with occasional episodes of vomiting. PE is unremarkable. Urine 
culture on a catheterized urine only showed E. Coli of 10,000 CFU. What will you do next?
A. Look for other focus of infection as UTI is ruled out 
B. Request for an ultrasound of the kidneys and urinary bladder 
C. Start antibiotics immediately 
D. Repeat urine culture from a suprapubic tap urine specimen  

88. Atopic individuals respond to allergen exposure with rapid expansion of TH2 cells that secrete cytokine favoring IgE 
synthesis and eosinophilia. What other statement/s is/are true regarding allergy?
A. The paternal history of atopy rather than maternal history have greater effect on development of allergies in 
a child 
B. Metabolites of arachidonic acid known to have potent anti-inflammatory activities are derived from 
eosinophils 
C. Allergens induce IgE production leading to Type I hypersensitivity responses 
D. TH17 cells are responsible for expression of allergic and autoimmune disease 

90. A 2 year old child was brought to your clinic with concern regarding her ​development​. Which of the following 
information should be elicited in the history?
A. Birth weight 
B. Intrauterine growth restriction 
C. Alcohol exposure 
D. All of the choices are correct 

91. Severe infection can trigger acute hemolytic crisis because of the deficiency of ​ankyrin and spectrin​ in this 
condition:
A. Thalassemia 
B. Hereditary spherocytosis 
C. Sickle cell anemia 
D. G6PD deficiency 

92. A 2 year old girl was brought in for consult due to ​regression in skills​ observed between​ 6-18 months​ of age. Which 
of the following conditions may be your differential diagnosis?

A. Angelman’s Syndrome 
B. Prader-Willi Syndrome 
C. Rett Syndrome 
D. Fragile X syndrome 

93. In the ​general assessment​ of a child brought to the Emergency Department, the following are evaluated except?
A. Strip of the dress ?? 
B. Muscle tone 
C. Skin color 
D. Breathing 

94. A 12-month old male was brought to the OPD due to​ multiple papulovesicular lesions on the face, trunk and 
extremities​. ​Varicella​ is highly considered for which the ​most common complication​ is?
A. Secondary infection of skin by streptococci or staphylococci 
B. Pneumonia 
C. Meningitis 
D. Encephalitis 

95.

96. These are risk factors for the development of ​acquired prothrombin complex deficiency​ EXCEPT:

A. Oral vitamin K intake? 


B. Malabsorption of vitamin K  
C. Prolonged antibiotic intake  
D. Breastfed infant 

97. The ​most common cause of mortality in patients with Trisomy 21​ is?
A. Duodenal atresia 
B. Pulmonary hypertension 
C. Leukemia 
D. Hypothyroidism 

98. A 4-year-old came in due to seizures with fever. The patient had 4 days cough and colds cough and colds. Past 
medical history showed that this is his 3rd episode. What would be your advice to the mother?
A. Start the patient with long-acting anti-seizure medication 
B.  Request for cranial MRI 
C. Expectedly there will be resolution of symptoms by 9 years old  
D. Get genetic testing since seizures are recurrent? 

99. A 7 year old had mild intermittent abdominal pain for 1 week associated with joint pains and multiple purpuric and 
petechial rashes over the buttocks and lower extremities. Your differential diagnosis should include?

A. All of the choices are correct 


B. Hypersensitivity vasculitis 
C. Juvenile rheumatoid arthritis 
D. Henoch Schoenlein purpura 

100. A 9 month-old, while being given immunization, developed circumoral cyanosis on crying.
PPE: central cyanosis, a grade 3/6 systolic ejection murmur at the 2nd LUSB radiating to the back. The most likely 
diagnosis is? 
A.  ​Total anomalous pulmonary venous return (TAPVR) 
B. Transposition of great arteries (TGA)  
C. Tetralogy of Fallot (TOF) 
D. d. Pulmonary valve atresia (PVA) 
PEDIA – PRACTICALS, Feb 2012

QUESTION ANSWER
1. At what age is the head circumference Silverman states different numbers depending on where in
smaller than the chest circumference the book. 6 months probably, but can be up to 12 months.
2. Define “low set ears” Silverman: 20% of ear above the horizontal line from the
medial canthus
3. How many fontanels does a newborn have? 6
4. How many teeth do you have at 10 months? I think = (Age – 6 months) = 4 teeth
5. Hyperdynamic heart is experienced in A. Fever I think.
Fever, B. Emphysema, C. both, D. neither
6. Narrow cuff leads to: A. higher BP, B. Lower Higher BP
BP, C. same BP
7. PERFORM Blood pressure on a child Make sure you grab the right cuff size
8. PERFORM Chest circumference Sternoxiphoid jxn
measurement on a dummy baby
9. PERFORM Chvostek’s sign? Flick the cheek of the patient. Check whether it should be
more towards the cheek, or more towards the mouth
10. PERFORM Jendrassek’s Have patient pull apart hands or tense something while
doing knee reflex
11. PERFORM Knee jerk
12. PERFORM Liver span on a real patient Remember to lift the shirt of the patient first.
13. PERFORM nasal patency on a dummy baby Plug one nostril at a time, and plug mouth too. (It helps to
explain to the resident what you are doing and why)
14. PERFORM the Ortolani test on a dummy Ask VERY clearly ahead of time what the pediatric
baby professors expect. Very few did it correct. You may need
to do one leg at a time
15. PERFORM tympanic membrane visualization Make sure to anchor your scope with a finger or knuckle
on a real patient on the patient
16. PERFORM vocal fremitus on a real patient. Confirm this one. They apparently expected us to use the
stethoscope, but I believe it should be palms only, right?
17. What is Chvostek’s sign assessing? Hypocalcemia (Silverman pg 229)
18. What is the most common congenital I think choanal atresia
anomaly of the nose?
19. What is the most common reason for non- Impacted cerumen?
visualization of the tympanic membrane?
20. What is the normal size of a 3yo liver? 4cm liver span
21. What is the Ortolani for? Assessing hip displacement
22. What is the test when you percuss the MacEwen’s
fontanels to assess high intracranial pres.?
23. What percussion does a pleural effusion Dull
make? A.? B.? C.dull D. resonant
24. When does the first tooth erupt? 6 months
25. Which have prominent ears? Trisomy 8, Silverman Pg 68. Not clear.
Trisomy 13, Both, or Neither
26. Which intercostal is the normal apex beat? 5th (4th in newborns)

ernie Page 1
Pracs last sem (2013):
1. babinski
2. Primary and Secondary skin lesion
3. Stereognosis
4. Short's manuever
5. Kernig's sign
6. Compute the IBW
7. How to check for skin turgor
8. Vocal fremitus/resonance
9.Prune belly syndrome
10. Dennie Morgan skin fold? and Allergic hand
salute
11.Tanner staging
12.Remitent fever

Vitamin A doses?
HEADSSFIRST (for adolescence)
H – home
E – education Zinc doses?
A – abuse
D – drugs
S – safety Signs of meningeal irritation
S – sexuality
x Nuchal rigidity
F – family & friends
x Brudzinski’s sign
I – image
x Kernig’s sign
R – recreation
S – spirituality
Review EPI vaccines (schedule & doses)
T – threats
Know the following procedures
(5) diseases of NB screening
1. congenital hypothyroidism x Ortolani sign
2. congenital adrenal hyperplasia x Chvosteks sign
3. PKU x Vocal & tactile fremitus
4. galactossemia
5, G6pd What disease in children will have low set ears?
x ANS:
VACTERL
V – vertebral Stimulation of NB
A – anus
C – cardiac Genu Valgum – knock knee
TE – transesophageal Genu Varum – bow legged
R – renal
L - limbs Reflexes
Superficial?
# of teeth = age in months – 6

Chest circumference Deep?


x Mid-inspiration at the xiphisternal
junction Pathologic reflex?

ernie Page 2
Cranial nerve test:
CN1
CN2
CN3
CN4
CN5
CN6
CN7
CN8
CN9
CN10
CN11
CN12

Abdominal Circumference? Umbilical level

Nasal patency assessment?

McEwan’s sign? Percuss fontanel to assess


increase ICP

BMI computation?

Jendrassic sign? Knee jerk reflex

Proper BF cuff?

Head Circumference measurement?

Cause of Nasal tumor/ anomaly?

How to check optic disc?

Where is the PMI of children?

Illicit patella reflex.

Review Trisomies.

ernie Page 3
ernie Page 4
ernie Page 5
ernie Page 6
 
         


  




 
     

      

     

    

       


 
             


  




 
  



 

  

    


 
         


  




 
             

            

        

             

   


           
 
                  

                
    


 


  

 

 

 

    


          


           

        
 
  

     

       


 
                   

             
             


 


             

    

 

   

       


 
         


  




 
        

      

              

         

   


        
 
             

           
  


 

      

    

    

   

       


      
           


  




 
    

  

  

    

     

      


 
              

  
  


 

   

   

     

       

      


 
        


  




                


       

         

       

   


        
 
         


  




 
  

   

 

  

    


 
            


  




 
  

    

     

   

       


 
                 

                  
               

             
         


      

               

        

        

         


               
 

 

  

  
    

  
 
         


  




 
  

 

 

  

    


 
             


  


 

  

      

  

 

         


 
          


  




 
    

  

  

  

      


                     
               
              


            

      

          

       

             

 
 
         


  




    




  

  

 

     


 
               

   
  


 

  

    

   

 

   


   
       
            


  




 
  

   

   

   

      


   
     


  




 
           

          

      

         

             
                 
              
     

      

      

  

  

     

  


                     
                 
                 
              
           

     

    

 

   

      


 
       


  




   




   

   

    

     


       
            


  




 
   

    

   

  

      


 
        


  




 
  

 

 

 

    


 
              

   
  


 


  

  

   

   

     


  
        


  




 
        

      

       

          

          
 
                  

        
  


 


   

   

  

   

      


 
            

   
  


 


       

    

   

   

      


 
         


  




 
    



   

   

      


 
            

 
  


 


   

  

  

   

      


 
                

             
               


 


            

             

       

             

             

 
            

 
  


 

    

    

 

    

        


 
               

   
  


 

    

 

   

  

    


 
                 

              
          


 


   

   

  

    

      


                   
              
           


      



  

  

  

     


 
        


  




 
      

        

     

         

           
              
              
  
   
  
      


  

  

       

  


 
           


  




 
   

    

  

 

     


 
         


  




 
   

 

  



     


 
             

                
                  

       
 


  

 

 

 

     


 
                  

            
  


 

    

 

   

  

     


 
               

               
                  

             
   


     

  

  

  

     


                
              
  
    
 
 

 

 

    


 
                  

                 
             


 


 

 

  

  

     


 
                

            
       


 


    


 
                 

              
   


 


 

 

   

 

   


  


      

   

  

    

  

 
 
             


  




 
   

  

  

  

     


 
                  

                    
                     

   
 


  

   

 

      

     


 
         


  




 
   

  

  

   

     


           


       



     
 
      

           

       


  
              

             
               


 


  

 

 

 

    


 
                  

               
                  

    
 


  

 

 

  

     


 
           


  




 
   

  

  

 

     

 
          


             

     
 
         

            

            
 
      


  




 
       

       

         

           

          
 
             

   
  


 


  



  

   

      


 
            


  




 

    

  

     

    

   


 
 
                 

          
       


 


  

   

  

  

      


 
              

          
  


 

 

 



    


  
                

           
  


 

        

        

          

       

          
 
                

             
          


 


     

     

     

     

        


 
           


  




 
    

  

  

  

       


       
         
  
 
  


 



 

  

 

     


 
       


  




          


    

       

              

          
      
             


  



 
      

     

    

    

       



               

             
               

              
          


   

        

        

    

       


 
                 

            
               


 


  

   

  

  

    


               
              
    


     



  

 

 

    


                    
                
               
                 
     

      

  

   

    

       


 
         


  




 
    

  

   

  

      


 
               

              
    


 


    



 

  

     


 
                 

            
    


 


 

 

    

  

     


 
              

          
   


 


  

  

 

    

     


  
          


  




 
          

    

           

       

   


          
  
              

              
        


 


   

   

 

    

    


 
             

                    
  


 

    

    

  

  

       


      
           
    
    
  


 

          

       

       

           

             
         


 
                


  




 
  

 

 

 

    


                  

 
            

        
  


 

  


   

    

  

      


 
          


  




 
   

  

 



    

 
                   
              
                


    



  

  

    

     


 
          


  




 
      

    

  

    

        


  
         

   
  


 


  

  

  

   

     


  
             


  




 
        

    

         

  

     


 
              

  
  


 

  

  

 

 

     


 
         


  




 
       

            


           

           

         
 
                 

            
  


 

 





 

    


 
          


  




 
   

  

    

  

   


 
 
               

           
  


 

 

  

 

  

    


 
              

            
      


 


 

 

 

   

     


 
         


  




            


            

          

       

   


          
 
                  

             
                  

 
 


  

  

   



    


  
                

               
         


 


    

   

  

     

      


 
                   

               
                   

        
 


  


  

 

 

    



                    
            
              
  
       
 


 

  

  

  

     

You might also like